You are on page 1of 196

CLT MAINS - 2018

Exam Title :
(Final)
Email : sasirekha314@gmail.com
Contact : 7708001877

QUESTION 1. MTQ3MStTQVNJUkVLSEEgUCtzYXNpcmVraGEzMTRAZ21haWwuY29tKzc3MDgwMDE4NzdRVU
VTVElPTiAw
PART A : Questions 1 - 50

True about superficial palmar arch is:

a) Direct continuation of radial artery


b) Lies deep to palmar aponeurosis
c) Integrity is tested by Adson’s test
d) Supplies the metacarpals
Correct Answer: B
Your Answer: A
Explanation

Ans. B. Lies deep to palmar aponeurosis

Superficial palmar arch Deep Palmar Arch


Features
Direct continuation of ulnar artery & Direct continuation of radial
Origin supported by superficial branch of radial artery & supported by deep
artery branch of ulnar artery

Superficial to long flexors of fingers & Deep to long flexors,


Relations deep to Palmaris brevis & palmar lumbricals & oblique head
aponeurosis of adductor pollicis

In the central part of palm on a line drawn At line drawn across palm at
Surface
across palm at distal border of fully proximal border of fully
marking
extended thumb extended thumb

3 palmar metacarpal
branches: these join the
digital branches of
4 digital branches supplying medial 3 & ½
Branches superficial palmar arch 3
fingers and lateral 3 digital branches
perforating arteries: these
anastomose with dorsal
metacarpal arteries.

QUESTION 2. MTQ3MStTQVNJUkVLSEEgUCtzYXNpcmVraGEzMTRAZ21haWwuY29tKzc3MDgwMDE4NzdRVU
VTVElPTiAx
All are Uncoupler of oxidative Phosphorylation, except:

a) Thyroxine
b) Thermogenin
c) 2,4 DNP
d) Cyanide

Score:
DBMCI CLT MAINS 2018 755.00 /
Page 1 1200
CLT MAINS - 2018
Exam Title :
(Final)
Email : sasirekha314@gmail.com
Contact : 7708001877
Correct Answer: D
Your Answer: D
Explanation

Ans. D. Cyanide

Uncouplers: Allow ETC to work normally but dissipate the proton gradient created by the ETC,
bypassing the ATP synthase complex, the Fo-F1 particle, also known as Complex V.

➢ Exogenous

● 2,4-Dinitrophenol

● Dicumarol

● FCCP (carbonyl cyanide-p-trifluorocarbonyl-cyanide methoxyphenol hydrazone)

➢ Endogenous:

● via UCP-1

▪ Thermogenin

● Via UCP-3

▪ Thyroxine

▪ Norepinephrine

▪ Epinephrine

▪ Leptin

● Via UCP-4

▪ Essential fatty acid deficiency

▪ Purine nucleotides

QUESTION 3. MTQ3MStTQVNJUkVLSEEgUCtzYXNpcmVraGEzMTRAZ21haWwuY29tKzc3MDgwMDE4NzdRVU
VTVElPTiAy
True statements about the kidney include:

a) The blood flow accounts for 75% of the total cardiac output.
b) The blood flow is greater in the renal medulla than in the cortex.
c) Over 99% of filtered water is reabsorbed by the kidney.
d) Antidiuretic hormone decreases the water permeability of the collecting duct.
Correct Answer: C
Your Answer: C
Explanation

Ans. C. Over 99% of filtered water is reabsorbed by the kidney

Score:
DBMCI CLT MAINS 2018 755.00 /
Page 2 1200
CLT MAINS - 2018
Exam Title :
(Final)
Email : sasirekha314@gmail.com
Contact : 7708001877

· The kidneys receive about 1.2 litres of blood per minute which is about 25% of the cardiac
output

· The blood flow is 20 times greater in the renal cortex than the renal medulla. Over 99% of the
filtered water is reabsorbed by the kidney.

· Antidiuretic hormone increases the water permeability of the collecting duct and hence
increases water retention.

· Kidney secrets erythropoietin which is essential for red blood cell production .

QUESTION 4. MTQ3MStTQVNJUkVLSEEgUCtzYXNpcmVraGEzMTRAZ21haWwuY29tKzc3MDgwMDE4NzdRVU
VTVElPTiAz
Choose the correct statement for the given histological section?

a) Duodenum with Brunner’s gland


b) Stomach with gastric pits
c) Jejunum with short villi
d) Appendix with distorted muscularis mucosa
Correct Answer: B
Your Answer: A
Explanation

Ans. B. Stomach with Gastric pits

Score:
DBMCI CLT MAINS 2018 755.00 /
Page 3 1200
CLT MAINS - 2018
Exam Title :
(Final)
Email : sasirekha314@gmail.com
Contact : 7708001877

QUESTION 5. MTQ3MStTQVNJUkVLSEEgUCtzYXNpcmVraGEzMTRAZ21haWwuY29tKzc3MDgwMDE4NzdRVU
VTVElPTiA0
Triangle of Koch is the landmark for :

a) SA node
b) Bundle of His
c) AV node
d) Purkinje fibres
Correct Answer: C
Your Answer: C
Explanation

Ans. C. AV node

Triangle of Koch

Triangular area in right atrium

· Marks the site of AV NODE

Boundaries

· Anterior : Base of septal leaflet of tricuspid valve

· Posterior : Anteromedial margin of opening of coronary sinus

· Superiorly : Tendon of Todaro

Score:
DBMCI CLT MAINS 2018 755.00 /
Page 4 1200
CLT MAINS - 2018
Exam Title :
(Final)
Email : sasirekha314@gmail.com
Contact : 7708001877

QUESTION 6. MTQ3MStTQVNJUkVLSEEgUCtzYXNpcmVraGEzMTRAZ21haWwuY29tKzc3MDgwMDE4NzdRVU
VTVElPTiA1
During moderate exercise, blood flow is maintained in which of the following organs?

a) Kidney
b) Skeletal muscle
c) Skin
d) Heart
Correct Answer: C
Your Answer: D
Explanation

Ans. C. Skin

During moderate exercise, blood flow is maintained in Brain and Skin

QUESTION 7. MTQ3MStTQVNJUkVLSEEgUCtzYXNpcmVraGEzMTRAZ21haWwuY29tKzc3MDgwMDE4NzdRVU
VTVElPTiA2
All of the following can be used as substrate for gluconeogenesis, except:

a) Pyruvate
b) Alanine
c) Acetyl Co A
d) lactate
Correct Answer: C
Your Answer: C
Explanation

Ans. C. Acetyl Co A

Substrates for gluconeogenesis are:

❖ Glucogenic amino acids (Alanine is most important precursor)

❖ Pyruvate

Score:
DBMCI CLT MAINS 2018 755.00 /
Page 5 1200
CLT MAINS - 2018
Exam Title :
(Final)
Email : sasirekha314@gmail.com
Contact : 7708001877

❖ Lactate

❖ Glycerol

❖ Propionate (from odd chain fatty acids)

QUESTION 8. MTQ3MStTQVNJUkVLSEEgUCtzYXNpcmVraGEzMTRAZ21haWwuY29tKzc3MDgwMDE4NzdRVU
VTVElPTiA3
Sciatic bed includes all the following Except:

a) Piriformis
b) Obturator internus
c) Adductor magnus
d) Superior gemellus
Correct Answer: A
Your Answer: C
Explanation

Ans. A. Piriformis

· Sciatic nerve comes below the piriformis.

· Thus not included in sciatic bed.

· The bed is contributed by- superior gemellus, obturator internus, inferior gemellus, Quadratus
femoris & adductor magnus.

QUESTION 9. MTQ3MStTQVNJUkVLSEEgUCtzYXNpcmVraGEzMTRAZ21haWwuY29tKzc3MDgwMDE4NzdRVU
VTVElPTiA4
CO 2 is primarily transported in the arterial blood as:

a) Dissolved CO 2
b) Carbonic Acid
c) Carbamino-hemoglobin
d) Bicarbonate
Correct Answer: D
Your Answer: D
Explanation

Ans. D. Bicarbonate

· CO2 is transported as dissolved CO2 (5%), as carbamino compounds (5%)and as bicarbonates


(90%)

Score:
DBMCI CLT MAINS 2018 755.00 /
Page 6 1200
CLT MAINS - 2018
Exam Title :
(Final)
Email : sasirekha314@gmail.com
Contact : 7708001877
QUESTION 10. MTQ3MStTQVNJUkVLSEEgUCtzYXNpcmVraGEzMTRAZ21haWwuY29tKzc3MDgwMDE4NzdRV
VTVElPTiA5
Which of the following apoprotein is found exclusively associated with chylomicrons?

a) Apo A
b) Apo B48
c) Apo CII
d) Apo D
Correct Answer: B
Your Answer: B
Explanation

Ans. B. Apo B48

· Apo B48 is found exclusively associated with chylomicrons and no other lipoprotein particle.
Apo B48 is synthesized from an mRNA that is transcribed from the Apo B100 gene.

· Following transcription, the mRNA is edited within the intestinal epithelium yielding the Apo
B48 transcript. Apo(a) is an apoprotein found disulfide bonded to Apo B100.

· This then forms a complex with LDL, generating a novel lipoprotein particle identified as
lipoprotein(a), Lp(a). Lp(a) has a strong resemblance to plasminogen and its presence in the
circulation is highly correlated with premature coronary artery disease.

· Apo CII is present in chylomicrons, VLDLs, LDLs, IDLs, and HDLs and is necessary for the
activation of endothelial cell LPL. Apo D is found exclusively with HDLs and is also associated
with cholesterol ester transfer protein (CETP) activity.

· Apo E is found in chylomicrons,VLDLs, LDLs, IDLs, and HDLs. It is necessary for interaction of
lipoprotein with the LDL-receptor (which is also referred to as the Apo B100/Apo E receptor).

QUESTION 11. MTQ3MStTQVNJUkVLSEEgUCtzYXNpcmVraGEzMTRAZ21haWwuY29tKzc3MDgwMDE4NzdRV


PTiAxMA==
The structures passing through optic canal include all of the following except:

a) Maxillary artery
b) Ophthalamic artery
c) Optic nerve
d) Central vein of retina
Correct Answer: A
Your Answer: A
Explanation

Ans. A. Maxillary artery

· The optic nerve which contains the central artery & vein of retina & the ophthalmic artery
pass through optic canal.

· The maxillary artery is a branch of external carotid & is divided into three parts by lateral
pterygoid muscle. It lies in the infratemporal fossa.

Score:
DBMCI CLT MAINS 2018 755.00 /
Page 7 1200
CLT MAINS - 2018
Exam Title :
(Final)
Email : sasirekha314@gmail.com
Contact : 7708001877

· The maxillary nerve passes through foramen rotundum.

QUESTION 12. MTQ3MStTQVNJUkVLSEEgUCtzYXNpcmVraGEzMTRAZ21haWwuY29tKzc3MDgwMDE4NzdRV


PTiAxMQ==
Which of the following are known as satellite DNA?

a) Centromere
b) Telomere
c) Both A & B
d) VNTRs
Correct Answer: C
Your Answer: B
Explanation

Ans. C. Both A & B

Organization of DNA:

QUESTION 13. MTQ3MStTQVNJUkVLSEEgUCtzYXNpcmVraGEzMTRAZ21haWwuY29tKzc3MDgwMDE4NzdRV


PTiAxMg==
A 53-year-old woman presents with tiredness. The results show: TSH 20.5 mU/l, free
thyroxine (FT4) 1.1 pmol/l, prolactin 1020 U/l. Which of the following is most likely
diagnosis?

a) Pituitary adenoma

Score:
DBMCI CLT MAINS 2018 755.00 /
Page 8 1200
CLT MAINS - 2018
Exam Title :
(Final)
Email : sasirekha314@gmail.com
Contact : 7708001877

b) Hypothyroidism
c) Sick euthyroid syndrome
d) Pregnancy
Correct Answer: B
Your Answer: B
Explanation

Ans. B. Hypothyroidism

· The patient has both symptoms and a biochemical profile consistent with hypothyroidism.

· A raised prolactin is a well-recognised association. The mechanism of this is not entirely clear
but probably involves an increased prolactin response to the rise in thyrotrophin-releasing
hormone (TRH).

· Apart from a pituitary prolactinoma, other pathological causes of a raised prolactin include:
hypothalamic/pituitary disease, renal impairment (reduced prolactin excretion) and drugs such
as dopamine antagonists (bromocriptine).

· Physiological causes are sleep, pregnancy, exercise, stress and puberty in girls.

QUESTION 14. MTQ3MStTQVNJUkVLSEEgUCtzYXNpcmVraGEzMTRAZ21haWwuY29tKzc3MDgwMDE4NzdRV


PTiAxMw==
Name the hormone that uses this type of receptor in Image?

a) Adrenaline
b) Nor adrenaline
c) Vitamin D
d) Dopamine

Score:
DBMCI CLT MAINS 2018 755.00 /
Page 9 1200
CLT MAINS - 2018
Exam Title :
(Final)
Email : sasirekha314@gmail.com
Contact : 7708001877
Correct Answer: C
Your Answer: C
Explanation

Ans. C. Vitamin D

Hormones that use nuclear receptor are,

· Aldosterone

· Cortisol

· Calcitriol

· Estrogen

· Progesterone

· Testosterone

· Thyroid hormones

Other than the thyroid hormones, all factors that use the nuclear receptor are steroid hormones

QUESTION 15. MTQ3MStTQVNJUkVLSEEgUCtzYXNpcmVraGEzMTRAZ21haWwuY29tKzc3MDgwMDE4NzdRV


PTiAxNA==
Most suitable test for metabolic disease?

a) Western Blot
b) Electrophoresis
c) Tandem MS
d) ELISA
Correct Answer: C
Your Answer: C
Explanation

Ans. C. Tandem MS

· Tandem MS involves use of two Mass spectrometers one after the other, i.e. in series. The first
MS will separate the various analytes as per q/m, while the second one will identify each
separated analyte. As many as 40 IEMs can be identified from a single sample, and it is the
preferred technique for neonatal screening for metabolic diseases.

· The ability of the other mentioned techniques to identify different proteins/analytes


simultaneously is limited.

QUESTION 16. MTQ3MStTQVNJUkVLSEEgUCtzYXNpcmVraGEzMTRAZ21haWwuY29tKzc3MDgwMDE4NzdRV


PTiAxNQ==
8th to 11th posterior intercostal veins of left side drains into:

Score:
DBMCI CLT MAINS 2018 755.00 /
Page 10 1200
CLT MAINS - 2018
Exam Title :
(Final)
Email : sasirekha314@gmail.com
Contact : 7708001877

a) Azygos vein
b) Hemiazygos vein
c) Accessory hemiazygos vein
d) Left brachiocephalic vein
Correct Answer: B
Your Answer: B
Explanation

Ans. B. Hemiazygos vein

HEMIAZYGOS VEIN :

· Formed by union of left subcostal & left ascending lumbar veins.

· Enters thorax by piercing the left crus of diaphragm.

· Drains into azygos vein opposite T8 vertebra.

· It may communicate with the left renal vein.

· Receives blood from lower 8th to 11th posterior intercostals veins.

QUESTION 17. MTQ3MStTQVNJUkVLSEEgUCtzYXNpcmVraGEzMTRAZ21haWwuY29tKzc3MDgwMDE4NzdRV


lPTiAxNg==
Purkinje cells of cerebellar cortex are excited by :

a) Stellate cells
b) Basket cells
c) Granule cells
d) Golgi cells
Correct Answer: C
Your Answer: C
Explanation

Ans. C. Granule cells

· Purkinje cells are inhibited by all other cortical interneurons (A, B, D).

· Purkinje neurons project to deep nuclei; they are only output from cerebellar cortex to deep
nuclei.

QUESTION 18. MTQ3MStTQVNJUkVLSEEgUCtzYXNpcmVraGEzMTRAZ21haWwuY29tKzc3MDgwMDE4NzdRV


PTiAxNw==
Glutathione reductase activity deficient in blood, with history of tingling and
numbness in both upper and lower limbs. Examination shows cheilosis, circumoral
lesions and painful red tongue. Deficient vitamin is:

a) Vit. B1

Score:
DBMCI CLT MAINS 2018 755.00 /
Page 11 1200
CLT MAINS - 2018
Exam Title :
(Final)
Email : sasirekha314@gmail.com
Contact : 7708001877

b) Vit. B2
c) Vit. B3
d) Vit. B12
Correct Answer: B
Your Answer: B
Explanation

Ans. B. Vit. B2

Riboflavin: Vitamin B2

· Dietary requirement is 0.5mg/1000kcal intake, not in terms of body mass.

· Biologically active form is FAD/FMN

· Deficiency is detected by measuring the activity of the enzyme glutathione reductase.

· Ariboflavinosis results in normochromic normocytic anemia and stomatitis, which presents as:

o Painful red tongue with sore throat

o Chapped and fissured lips (cheilosis)

o Inflammation of the corners of the mouth (angular stomatitis)

· Deficiency is sometimes called "pellagra sine pellagra", since the stomatitis of pellagra is
similar.

QUESTION 19. MTQ3MStTQVNJUkVLSEEgUCtzYXNpcmVraGEzMTRAZ21haWwuY29tKzc3MDgwMDE4NzdRV


lPTiAxOA==
Given three site of a typical injection for the anesthetic block of maxilla and mandible
are for which nerves?

a) A-Greater palatine nerve, B-Nasopalatine nerve, C-lingual nerve


b) A- IX nerve, B- Greater palatine nerve, C-Lingual nerve
c) A- Greater palatine nerve, B- Nasopalatine nerve, C- Inferior alveolar nerve
d) A- Palatine branch of VII nerve, B- IX nerve, C- inferior alveolar nerve
Correct Answer: C
Your Answer: B

Score:
DBMCI CLT MAINS 2018 755.00 /
Page 12 1200
CLT MAINS - 2018
Exam Title :
(Final)
Email : sasirekha314@gmail.com
Contact : 7708001877
Explanation

Ans. C. A- Greater palatine nerve, B- Nasopalatine nerve, C- Inferior alveolar nerve

QUESTION 20. MTQ3MStTQVNJUkVLSEEgUCtzYXNpcmVraGEzMTRAZ21haWwuY29tKzc3MDgwMDE4NzdRV


PTiAxOQ==
Posterior boundary of foramen of winslow is :

Score:
DBMCI CLT MAINS 2018 755.00 /
Page 13 1200
CLT MAINS - 2018
Exam Title :
(Final)
Email : sasirekha314@gmail.com
Contact : 7708001877

a) Aorta
b) Inferior vena cava
c) Portal vein
d) Liver
Correct Answer: B
Your Answer: B
Explanation

Ans. B. Inferior vena cava

· The other name for epiploic foramen is foramen of winslow.

· It is situated behind the free margin of the lesser omentum at the level of

· t12 vertebra.

Relationship:

· Anteriorly: Right free margin of the lesser omentum containing portal vein, hepatic artery and
bile duct.

· Posteriorly: The inferior vena cava, right suprarenal gland and t12 vertebra.

· Superiorly: The caudate process of liver.

· Inferiorly: 1st part of duodenum and horizontal part of hepatic artery.

QUESTION 21. MTQ3MStTQVNJUkVLSEEgUCtzYXNpcmVraGEzMTRAZ21haWwuY29tKzc3MDgwMDE4NzdRV


PTiAyMA==
Gap junctions are responsible for:

a) Cellular polarity
b) Transmission of action potentials from one fibre to another in skeletal muscle
c) Rapid transmission of action potentials by purkinje fibres
d) None
Correct Answer: C
Your Answer: C
Explanation

Ans. C. Rapid transmission of action potentials by purkinje fibres.

· The gap junction is a specialised channel of transport and communication for small molecules
and ions between the cytoplasm of two closely apposed cells.

· Electron microscopy reveals the two cell membranes to be separated by only 1–2nm at the
point where the gap junctions are sited.

· Each membrane has a large number of small protein channels termed connexons. Individual
connexons of both cells are joined to each other to create a communicating tunnel, which spans
the membrane gap.

· The internal diameter of the passage is about 2 nm.

Score:
DBMCI CLT MAINS 2018 755.00 /
Page 14 1200
CLT MAINS - 2018
Exam Title :
(Final)
Email : sasirekha314@gmail.com
Contact : 7708001877

· Each connexon on each membrane is made up of six polypeptide chain subunits, which span
the membrane.

· Electrophysiological studies have shown low electrical resistance for the cell when there are
abundant gap junctions. Injection of low molecular-weight dyes results in rapid spread between
cells.

· Therefore, gap junctions provide a pathway by which small molecules and ions can rapidly
move down concentration gradients between cells.

· Gap junctions are evident in epithelia and smooth muscle. They permit all of the cells to act as
a syncytium.

· However, if continued passive transport is detrimental to the syncytium, individual gap


junctions can be rapidly closed.

QUESTION 22. MTQ3MStTQVNJUkVLSEEgUCtzYXNpcmVraGEzMTRAZ21haWwuY29tKzc3MDgwMDE4NzdRV


PTiAyMQ==
Fluoride released from fluoroacetate inhibits which cell cycle?

a) Glycolysis
b) Krebs’ cycle
c) Oxidative Phosphorylation
d) Salvage Pathway
Correct Answer: C
Your Answer: B
Explanation

Ans. C. Oxidative Phosphorylation

· Fluoride released from fluoroacetate causes hypocalcemia. Calcium is responsible for


stimulating the ETC and oxidative Phosphorylation.

· Thus, hypocalcemia causes downregulation of these important pathways, besides causing the
regular physiological alterations related to muscle contraction, like tetany, etc.

· Fluoroacetate combines with oxaloacetate to form the reactive compound fluoro citrate which
binds tightly to the enzyme aconitase. This is the primary mode of toxicity of the molecule, but
secondary effects due to the free fluoride from both the precursor (fluoroacetate) and product
(fluoro citrate) by inducing hypocalcemia are significant; both physiological effects and
biochemical effects are seen.

· In addition, the action of fluoride in cytoplasm is well known. It Chelates the Mg++ ion, and
interferes with the action of enzyme Enolase, thereby inhibiting glycolysis.

QUESTION 23. MTQ3MStTQVNJUkVLSEEgUCtzYXNpcmVraGEzMTRAZ21haWwuY29tKzc3MDgwMDE4NzdRV


PTiAyMg==
The supraduodenal bile duct is chiefly supplied by –

Score:
DBMCI CLT MAINS 2018 755.00 /
Page 15 1200
CLT MAINS - 2018
Exam Title :
(Final)
Email : sasirekha314@gmail.com
Contact : 7708001877

a) Vessels that run upwards from gastroduodenal and retroduodenal arteries


b) Vessels that run downwards from the cystic artery
c) Vessels that arise from the hepatic artery as it courses up along the common bile duct and
supplies it in a non-axial fashion
d) Vessels that run downwards from the right hepatic artery
Correct Answer: A
Your Answer: Unanswered
Explanation

Ans. A. Vessels that run upwards from gastroduodenal and retroduodenal arteries

· The bile duct is sub classified into three segments hilar, supraduodenal & retropancreatic.

· The blood supply of supra-duodenal duct is essentially axial.

· It arises from the superior Pancreaticoduodenal artery, right branch of hepatic artery, cystic
artery, gastroduodenal artery and retroduodenal artery.

· The most important of these vessels run along the lateral border of ducts and are called 3 ‘o’
clock & 9 ‘o’ clock arteries.

· The hilar duct receives blood from surrounding vessel forming a rich network on surface of the
ducts in continuity with the plexus around supraduodenal duct.

· The retropancreatic common bile duct gets its blood supply from retroduodenal artery.

QUESTION 24. MTQ3MStTQVNJUkVLSEEgUCtzYXNpcmVraGEzMTRAZ21haWwuY29tKzc3MDgwMDE4NzdRV


PTiAyMw==
Incorrect statement regarding the given joint/marked structures is:

Score:
DBMCI CLT MAINS 2018 755.00 /
Page 16 1200
CLT MAINS - 2018
Exam Title :
(Final)
Email : sasirekha314@gmail.com
Contact : 7708001877

a) Synovial lining of elbow joint extends inferiorly deep to “A”


b) Radial fossa is extracapsular but olecranon fossa is intracapsular
c) “B” is the strangest band of ulnar collateral ligament
d) Brachioradialis is effective flexor in mid-prone position
Correct Answer: B
Your Answer: B
Explanation

Ans. B. Radial fossa is extracapsular but olecranon fossa is intracapsular

A. Synovial lining extends deep to annular ligament and continue with superior RU joint

B. Radial, coronoid and olecranon fossa fossae are intracapsular

C. B is the anterior band of ulnar collateral ligament and is the strongest of all three bands

D. Brachioradialis is most effective in flexion in midprone forearm.

QUESTION 25. MTQ3MStTQVNJUkVLSEEgUCtzYXNpcmVraGEzMTRAZ21haWwuY29tKzc3MDgwMDE4NzdRV


lPTiAyNA==
A neonate presents to emergency on 5th day of life with complaints of vomiting,
lethargy and poor feeding. The resident on duty suspects inborn error of metabolism.
Urine examination was negative by glucose-oxidase test, but positive in Benedict's test
for reducing substance. On physical examination, there is presence of metabolic
cataract in both eyes. Most likely IEM is:

a) Neonatal Diabetes Mellitus


b) Essential Fructosuria
c) Galactosemia
d) Hereditary Fructose Intolerance
Correct Answer: C
Your Answer: C
Explanation

Ans. C. Galactosemia

Galactose metabolism involves 3 enzymes. They are:

1. Galactokinase

2. Galactose-1-P uridyl Transferase (GALT)

3. Epimerase

Deficiency of any of these three enzymes can present with Galactosemia, however,
most commonly, the deficiency of GALT is encountered. The important presentations of
galactosemia include:

· Poor growth within the first few weeks of life

· Jaundice

Score:
DBMCI CLT MAINS 2018 755.00 /
Page 17 1200
CLT MAINS - 2018
Exam Title :
(Final)
Email : sasirekha314@gmail.com
Contact : 7708001877

· Bleeding from coagulopathy

· Liver dysfunction and/or hepatomegaly

· Cataracts (sometimes as early as the first few days of life)

· Lethargy

· Hypotonia

· Sepsis ( E coli)

· Seizures

· Brain edema

o In an infant or child with cataracts, galactosemia must be excluded.

o In Hereditary Fructose Intolerance, presentation typically starts at the time of weaning (6


months or more), and it is not associated with cataract.

o In patients of neonatal DM, both the tests would give positive result.

o Essential Fructosuria is asymptomatic.

QUESTION 26. MTQ3MStTQVNJUkVLSEEgUCtzYXNpcmVraGEzMTRAZ21haWwuY29tKzc3MDgwMDE4NzdRV


PTiAyNQ==
The frequency of predominant EEG wave present in cerebral sleep or deep sleep is :-

a) 8-12 Hz
b) 18-30 Hz
c) 4-7 Hz
d) <4 Hz
Correct Answer: D
Your Answer: D
Explanation

Ans. D. <4 Hz

· Awake state - Beta wave dominates. EEG is said to be desynchronised, it displays low voltage,
high frequency activity (18-20 Hz)

· Relaxed individual with eyes closed - Alpha wave predominates, frequency 8-12 Hz & 50-100 V
amplitude. Most marked in parietooccipital area.

o Stage I NREM sleep - Alpha waves interspersed with low frequency (4-7 Hz) theta waves.

o Theta waves are generated in hippocampus.

o Stage II NREM sleep - EEG slows further, but slow wave activity interrupted by sleep spindles
(12-14 Hz) & by large K complexes (large, low potentials)

o Stage III - Delta wave with occasional sleep spindles.

Score:
DBMCI CLT MAINS 2018 755.00 /
Page 18 1200
CLT MAINS - 2018
Exam Title :
(Final)
Email : sasirekha314@gmail.com
Contact : 7708001877

o Stage IV - Deep sleep or cerebral sleep delta wave (<4 Hz)

o REM sleep - EEG again become desynchronized. Low voltage, fast activity of REM sleep
resembles that seen in aroused subject.

o Muscle tone is completely lost; but phasic contractions occur in number of muscles, most
notably eye muscles.

QUESTION 27. MTQ3MStTQVNJUkVLSEEgUCtzYXNpcmVraGEzMTRAZ21haWwuY29tKzc3MDgwMDE4NzdRV


lPTiAyNg==
Anconeus is supplied by:

a) Median nerve
b) Radial nerve
c) Musculocutaneus nerve
d) Posterior interosseous nerve
Correct Answer: B
Your Answer: B
Explanation

Ans. B. R adial nerve

The following are branches of the radial nerve (including the superficial branch of the radial
nerve and the deep branch of the radial nerve/posterior interosseous nerve).

Cutaneous

Cutaneous innervation of the right upper extremity.

Cutaneous innervation by the radial nerve is provided by the following nerve branches:

· Posterior cutaneous nerve of arm (originates in axilla)

· Inferior lateral cutaneous nerve of arm (originates in arm)

· Posterior cutaneous nerve of forearm (originates in arm)

The superficial branch of the radial nerve provides sensory innervation to much of the back of
the hand, including the web of skin between the thumb and index finger.

Motor

Muscles of the posterior forearm. All the labelled muscles (that is, all the visible muscles except
the ones on the dorsal hand and one at top left) are innervated by the radial nerve, and
represent all muscles innervated by the radial nerve except for the Supinator.

Muscular branches of the radial nerve:

· Triceps brachii (lateral and medial heads)

· Anconeus

· Brachioradialis

Score:
DBMCI CLT MAINS 2018 755.00 /
Page 19 1200
CLT MAINS - 2018
Exam Title :
(Final)
Email : sasirekha314@gmail.com
Contact : 7708001877

· Extensor carpi radialis longus

Deep branch of the radial nerve:

· Extensor carpi radialis brevis

· Supinator

Posterior interosseous nerve (a continuation of the deep branch after the Supinator):

· Extensor digitorum

· Extensor digiti minimi

· Extensor carpi ulnaris

· Abductor pollicis longus

· Extensor pollicis brevis

· Extensor pollicis longus

· Extensor indicis

The radial nerve (and its deep branch) provides motor innervation to the muscles in the
posterior compartment of the arm and forearm, which are mostly extensors.

QUESTION 28. MTQ3MStTQVNJUkVLSEEgUCtzYXNpcmVraGEzMTRAZ21haWwuY29tKzc3MDgwMDE4NzdRV


PTiAyNw==
In the given GTT graph, graph B denotes:

a) Uncontrolled Diabetes mellitus


b) Impaired glucose tolerance
c) Renal glycosuria
d) Normal individual

Score:
DBMCI CLT MAINS 2018 755.00 /
Page 20 1200
CLT MAINS - 2018
Exam Title :
(Final)
Email : sasirekha314@gmail.com
Contact : 7708001877
Correct Answer: B
Your Answer: B
Explanation

Ans. B. Impaired glucose tolerance

❖ Normal fasting blood glucose value < 100 mg% (ADA criteria)/ 110 mg% (WHO criteria)

❖ Normal two hour post glucose loading with 75gm < 140 mg%. Physiologically, the level
returns to fasting glucose level or less.

❖ Renal threshold is 180 mg%, which is not crossed physiologically.

❖ Renal glycosuria occurs physiologically in pregnancy & after severe exercise.

QUESTION 29. MTQ3MStTQVNJUkVLSEEgUCtzYXNpcmVraGEzMTRAZ21haWwuY29tKzc3MDgwMDE4NzdRV


lPTiAyOA==
Caisson's disease is due to:

a) Gas embolism
b) Fat embolism
c) Amniotic fluid
d) Tumor embolism
Correct Answer: A
Your Answer: A
Explanation

Score:
DBMCI CLT MAINS 2018 755.00 /
Page 21 1200
CLT MAINS - 2018
Exam Title :
(Final)
Email : sasirekha314@gmail.com
Contact : 7708001877

Ans. A. Gas embolism

· Caisson’s disease (synonyms- Decompression sickness, bends, Compressed air sickness,


Diver’s paralysis, Dysbarism)- symptoms of caisson’s disease are caused by gas bubbles
blocking many blood vessels in different tissues (gas embolism) seen during sudden
decompression in divers.

· If a diver has been beneath the sea long enough that large amounts of nitrogen have dissolved
in his or her body and the diver then suddenly comes back to the surface of the sea, significant
quantities of nitrogen bubbles can develop in the body fluids either intracellularly or
extracellularly and can cause minor or serious damage in almost any area of the body,
depending on the number and sizes of bubbles formed; this is called decompression sickness

· Most common symptom of decompression sickness, affecting 85 to 90% of patients is ‘bends’


or pain in the joints and muscles of legs and arms.

· In 5 to 10% of the patient nervous system symptoms occur ranging from dizziness to paralysis
or collapse and unconsciousness. 2% of patients develop “the chokes” caused by gas embolism
in the capillaries of the lungs; patient complains of dyspnoea followed by severe pulmonary
edema and occasionally, death.

QUESTION 30. MTQ3MStTQVNJUkVLSEEgUCtzYXNpcmVraGEzMTRAZ21haWwuY29tKzc3MDgwMDE4NzdRV


PTiAyOQ==
Which type of collagen is present in cartilage:

a) Type I
b) Type III
c) Type IV
d) Type II
Correct Answer: D
Your Answer: D
Explanation

Ans. D. Type II

Important Collagen types to remember:

Bone: Type I collagen

Cartilage: Type II collagen

Meniscus (knee): Type I collagen

Score:
DBMCI CLT MAINS 2018 755.00 /
Page 22 1200
CLT MAINS - 2018
Exam Title :
(Final)
Email : sasirekha314@gmail.com
Contact : 7708001877

QUESTION 31. MTQ3MStTQVNJUkVLSEEgUCtzYXNpcmVraGEzMTRAZ21haWwuY29tKzc3MDgwMDE4NzdRV


PTiAzMA==
TPP is important for energy production, because it is coenzyme for:

a) Erythrocyte transketolase
b) Fatty acid synthase
c) Phosphofructokinase-1
d) Pyruvate dehydrogenase
Correct Answer: D
Your Answer: D
Explanation

Ans. D. Pyruvate dehydrogenase

TPP, Thiamine Pyrophosphate, is the active form of vitamin B1, Thiamine.

❖ Erythrocyte transketolase is an enzyme of HMP shunt, which is not related to energy


production.

❖ Fatty acid synthase is the enzyme complex involved in fatty acid synthesis, and also not
related to energy production.

❖ PFK-1 is an enzyme of glycolysis, but it does not use TPP, but Mg ++ ion.

❖ Pyruvate dehydrogenase catalyses the bridging reaction between glycolysis and TCA cycle,
and it requires TPP. Other enzymes requiring TPP include α-ketoglutarate dehydrogenase and
branched-chain keto-acid dehydrogenase.

Score:
DBMCI CLT MAINS 2018 755.00 /
Page 23 1200
CLT MAINS - 2018
Exam Title :
(Final)
Email : sasirekha314@gmail.com
Contact : 7708001877

QUESTION 32. MTQ3MStTQVNJUkVLSEEgUCtzYXNpcmVraGEzMTRAZ21haWwuY29tKzc3MDgwMDE4NzdRV


PTiAzMQ==
Pre trematric nerve of 1st pharyngeal arch:

a) Chorda tympani nerve


b) Mandibular nerve
c) Glossopharyngeal nerve
d) Superior laryngeal nerve
Correct Answer: A
Your Answer: B
Explanation

Ans. A. Chorda tympani nerve

Post trematric nerve is the nerve of the corresponding arch

1 st Arch nerve- Mandibular nerve

2 nd Arch – Facial

3 rd Arch – Glossopharyngeal nerve

4 th Arch- Superior laryngeal nerve

6 th Arch- Recurrent laryngeal nerve

Pre trematric nerve persist for 1 st arch only and that is chorda tympani branch of facial nerve

QUESTION 33. MTQ3MStTQVNJUkVLSEEgUCtzYXNpcmVraGEzMTRAZ21haWwuY29tKzc3MDgwMDE4NzdRV


PTiAzMg==
Vomiting centre is situated in the :

a) Hypothalamus
b) Midbrain
c) Pons
d) Medulla
Correct Answer: D
Your Answer: D
Explanation

Ans. D. Medulla

· Vomiting is believed to be controlled by two distinct brain centres-the vomiting centre and the
chemoreceptor trigger zone-both located in the medulla oblongata.

Score:
DBMCI CLT MAINS 2018 755.00 /
Page 24 1200
CLT MAINS - 2018
Exam Title :
(Final)
Email : sasirekha314@gmail.com
Contact : 7708001877

· The vomiting centre initiates and controls the act of emesis, which involves a series of
contractions of the smooth muscles lining the digestive tract.

QUESTION 34. MTQ3MStTQVNJUkVLSEEgUCtzYXNpcmVraGEzMTRAZ21haWwuY29tKzc3MDgwMDE4NzdRV


PTiAzMw==
Which among the following is wrongly marked in the given Image:

a) Corpus callosum
b) Fornix
c) cerebellum
d) hypothalamus
Correct Answer: D
Your Answer: D
Explanation

Ans. D. D- hypothalamus

D is pituitary gland not hypothalamus all other options correct

Score:
DBMCI CLT MAINS 2018 755.00 /
Page 25 1200
CLT MAINS - 2018
Exam Title :
(Final)
Email : sasirekha314@gmail.com
Contact : 7708001877
QUESTION 35. MTQ3MStTQVNJUkVLSEEgUCtzYXNpcmVraGEzMTRAZ21haWwuY29tKzc3MDgwMDE4NzdRV
PTiAzNA==
The following is an example for unusual base :

a) Di hydro uracil
b) Adenine
c) Cytosine
d) Uracil
Correct Answer: A
Your Answer: A
Explanation

Ans. A. Di hydro uracil

Two types of nitrogenous bases are present in all nucleic acids namely : Purines :

o Adenine

o Guanine

Pyrimidines :

· Cytosine

· Thymine

· Uracil

A few other modified pyrimidine bases like dihydrouracil and 5-methyl cytosine are rarely found
in some types of RNA.

QUESTION 36. MTQ3MStTQVNJUkVLSEEgUCtzYXNpcmVraGEzMTRAZ21haWwuY29tKzc3MDgwMDE4NzdRV


PTiAzNQ==
Which of the following is not absorbed in PCT:

a) Na+
b) HCO3
c) PO4
d) H+
Correct Answer: D
Your Answer: D
Explanation

Ans. D. H+

Transport proteins involved in the movement of Na+ and Cl- Across the apical
membranes of Renal Tubular Cells

Score:
DBMCI CLT MAINS 2018 755.00 /
Page 26 1200
CLT MAINS - 2018
Exam Title :
(Final)
Email : sasirekha314@gmail.com
Contact : 7708001877

Apical Transporter Function

Na/glucose Co transporter Na + uptake, glucose uptake

Na + /P 1 Con transporter Na + uptake, Pi uptake

Site
Na + amino acid Con
Na + uptake, amino acid uptake
Proximal tubule transporter

Na/lacate Contransporter Na + uptake, lactate uptake

Na/H exchanger
Na + uptake, H* extrusion
Cl/base exchanger
Cl- uptake

Na-K-2Cl Contransporter Na + uptake, Cl- uptake, K+


uptake
Thick ascending
limb
Na/H exchanger Na + uptake, H* extrusion

K + channels K + extrusion (recycling)


Distal
convoluted
tubule collecting
NaCL con transporter Na + uptake, Cl- uptake
duct
Na + channel (ENaC) Na + uptake

QUESTION 37. MTQ3MStTQVNJUkVLSEEgUCtzYXNpcmVraGEzMTRAZ21haWwuY29tKzc3MDgwMDE4NzdRV


lPTiAzNg==
Gamelli muscle related to which joint:

a) Hip
b) Shoulder
c) Knee
d) Elbow
Correct Answer: A
Your Answer: A
Explanation

Ans. A. Hip

Muscle compartments of hip joint

Score:
DBMCI CLT MAINS 2018 755.00 /
Page 27 1200
CLT MAINS - 2018
Exam Title :
(Final)
Email : sasirekha314@gmail.com
Contact : 7708001877

Adductors Abductors Medial rotators Lateral rotators

Pectineus Gluteus medius Gluteus medius Gluteus maximus

Adductor longus Gluteus minimus Gluteus minimus Piriformis

Superior and inferior


Adductor brevis Tensor fasciae latae Tensor fasciae latae
gamell

Gracllus Sartorius Obturator internus

Adductor magnus Quadratus femoris

Obturator externus Obturator externus

QUESTION 38. MTQ3MStTQVNJUkVLSEEgUCtzYXNpcmVraGEzMTRAZ21haWwuY29tKzc3MDgwMDE4NzdRV


PTiAzNw==
The most important cholorectic is:

a) Bile salt
b) CCK
c) Secretin
d) Gastrin
Correct Answer: A
Your Answer: A
Explanation

Ans. A. Bile salt

· Cholagogue is the one which causes gall bladder contraction, thereby increasing flow (eg CCK
– PZ)

· Cholorectic increases secretion of bile (eg- Bile salt)

QUESTION 39. MTQ3MStTQVNJUkVLSEEgUCtzYXNpcmVraGEzMTRAZ21haWwuY29tKzc3MDgwMDE4NzdRV


lPTiAzOA==
UDP glucose is used for all except:

a) Glycogen synthesis
b) Galactose metabolism
c) Heparin synthesis

Score:
DBMCI CLT MAINS 2018 755.00 /
Page 28 1200
CLT MAINS - 2018
Exam Title :
(Final)
Email : sasirekha314@gmail.com
Contact : 7708001877

d) Sphingolipid synthesis
Correct Answer: D
Your Answer: C
Explanation

Ans. D. Sphingolipid synthesis

UDP Glucose is the precursor and participant in several pathways.

❖ UDP glucose is the glucose transporter/donor in glycogen synthesis.

❖ UDP galactose is converted to UDP glucose in the final step of its metabolism.

❖ UDP glucose is also used in Lactose synthesis.

❖ UDP glucose is transporter/donor of glucose for glycolipids and glycoprotein synthesis.

❖ UDP glucose gets converted to UDP-glucuronic acid, which can then be isomerised to
iduronic acid, both of which are essential precursors to glycosaminoglycan (GAG) synthesis.
UDP-glucuronic acid is also used for conjugation of bilirubin.

QUESTION 40. MTQ3MStTQVNJUkVLSEEgUCtzYXNpcmVraGEzMTRAZ21haWwuY29tKzc3MDgwMDE4NzdRV


PTiAzOQ==
Which structure in Image may be fractured, resulting in loss of the chief flexor of the
thigh?

a) A

Score:
DBMCI CLT MAINS 2018 755.00 /
Page 29 1200
CLT MAINS - 2018
Exam Title :
(Final)
Email : sasirekha314@gmail.com
Contact : 7708001877

b) B
c) C
d) D
Correct Answer: D
Your Answer: C
Explanation

Ans. D. (D)

The iliopsoas muscle is the chief flexor of the thigh and inserts on the lesser trochanter.

QUESTION 41. MTQ3MStTQVNJUkVLSEEgUCtzYXNpcmVraGEzMTRAZ21haWwuY29tKzc3MDgwMDE4NzdRV


PTiA0MA==
Physiological effect that is not produced by stimulation of Kappa opioid receptor is:

a) Sedation
b) Diuresis
c) Miosis
d) Constipation
Correct Answer: D
Your Answer: D
Explanation

Ans. D. Constipation

Because of the stimulation of µ receptor constipation will occur. They can be found in the
intestinal tract.

This will be the cause of constipation i.e. a major side effect of μ agonists, due to inhibition of
peristaltic action.

Effect of stimulation of Kappa opioid receptor includes the following-

• Sedation
• Analgesia
• Miosis
• Dysphoria
• Diuresis

Among the five related receptors the κ-opioid receptor will bind opium-like compounds in the
brain which are responsible for mediating the effects of these compounds.

It effects includes altering the:-

• Mood
• Perception of pain
• Consciousness
• Motor control

Score:
DBMCI CLT MAINS 2018 755.00 /
Page 30 1200
CLT MAINS - 2018
Exam Title :
(Final)
Email : sasirekha314@gmail.com
Contact : 7708001877
QUESTION 42. MTQ3MStTQVNJUkVLSEEgUCtzYXNpcmVraGEzMTRAZ21haWwuY29tKzc3MDgwMDE4NzdRV
PTiA0MQ==
Increased afferent input discharge by the baroreceptor:

a) Increases vagal stimulation


b) Causes peripheral vasoconstriction
c) Increases heart rate
d) Increases sympathetic stimulation
Correct Answer: A
Your Answer: A
Explanation

Ans. A. Increases vagal stimulation.

· Baroreceptor are found in the carotid sinus, as well as the atria, aorta and pulmonary
circulation.

· They are stretch receptors that detect low pressures. With stretch due to an increase in blood
pressure the afferent discharge increases, leading to reduced sympathetic discharge, reduced
vasoconstriction and bradycardia; cardiac output falls, as does peripheral vascular resistance.

· At normal blood pressure the baroreceptor fire slowly, A fall in discharge would increase renin
production.

· Chemoreceptors in the carotid sinus respond to hypoxemia by increasing sympathetic activity.

QUESTION 43. MTQ3MStTQVNJUkVLSEEgUCtzYXNpcmVraGEzMTRAZ21haWwuY29tKzc3MDgwMDE4NzdRV


PTiA0Mg==
A 56 year old homeless man presents to ER with increased fatigability and exertional
dyspnea) Physical examination shows significant lower extremity edema and decreased
sensation over the ankles and feet. Further evaluation reveals cardiac dilation and
increased cardiac output. Which vitamin deficiency is most likely responsible for this
patient’s symptoms?

a) Riboflavin
b) Pyridoxine
c) Niacin
d) Thiamine
Correct Answer: D
Your Answer: D
Explanation

Ans. D. Thiamine

Effects of Thiamine deficiency:

Beriberi:

• Dry Berry Beri:-

– It involves CNS in the form of parasthesia, general weakness, atrophy, foot and wrist drop.

Score:
DBMCI CLT MAINS 2018 755.00 /
Page 31 1200
CLT MAINS - 2018
Exam Title :
(Final)
Email : sasirekha314@gmail.com
Contact : 7708001877

• Wet Berry Beri:-

– It involves CVS in the form of palpitation, tachycardia, AR, edema, high cardiac output failure

• Acute/Infant Berry Beri:-

– Seen in those infants whose mothers milk is deficient in B 1.

Wernicke’s encephalopathy:

• Seen in chronic alcoholics

• Characteristics by global confusion & ophthalmoplegia .

• If psychosis is present, the condition is termed “Wernicke’s Korsakoff’s Psychosis”

QUESTION 44. MTQ3MStTQVNJUkVLSEEgUCtzYXNpcmVraGEzMTRAZ21haWwuY29tKzc3MDgwMDE4NzdRV


PTiA0Mw==
Which of the following statements is true:

a) Curve ‘A’ indicates RPF which increase initially due to decrease plasma oncotic pressure
b) Curve ‘A’ indicates GFR which increase initially due to increase hydrostatic pressure of
glomerulus
c) Curve ‘B’ indicates RPF which decreases due to increase afferent arteriole resistance
d) Curve ‘B’ indicates GFR which decreases due to decrease hydrostatic pressure of
glomerulus
Correct Answer: B
Your Answer: B
Explanation

Score:
DBMCI CLT MAINS 2018 755.00 /
Page 32 1200
CLT MAINS - 2018
Exam Title :
(Final)
Email : sasirekha314@gmail.com
Contact : 7708001877

Ans. B. Curve ‘A’ indicates GFR which increase initially due to increase hydrostatic
pressure of glomerulus

Curve A: GFR, which increases initially due to increased hydrostatic pressure in glomerulus,
later decreases due to increase oncotic pressure of glomerular capillary as well as severe
decrease in hydrostatic pressure of glomerular capillary.
Curve B: RPF, which decreases proportionately with increased efferent arterioles resistance.

QUESTION 45. MTQ3MStTQVNJUkVLSEEgUCtzYXNpcmVraGEzMTRAZ21haWwuY29tKzc3MDgwMDE4NzdRV


PTiA0NA==
Lesch Nyhan syndrome is caused by:

a) HPRT complete deficiency


b) HPRT Partial deficiency
c) Purine nucleoside phosphorylase deficiency
d) PRP synthetase deficiency
Correct Answer: A
Your Answer: A
Explanation

Ans. A. HPRT complete deficiency

· Mutations in the HPRT1 gene cause Lesch-Nyhan syndrome.

· Mutations in the HPRT1 gene cause a severe deficiency of the enzyme hypoxanthine
phosphoribosyltransferase 1.

· This enzyme is responsible for recycling purines, a type of building block of DNA and its
chemical cousin RNA.

· When this enzyme is lacking, the breakdown of purines results in abnormally high levels of
uric acid in the body.

· It is unclear how a shortage of this enzyme causes the neurological and behavioral problems
characteristic of Lesch-Nyhan syndrome.

QUESTION 46. MTQ3MStTQVNJUkVLSEEgUCtzYXNpcmVraGEzMTRAZ21haWwuY29tKzc3MDgwMDE4NzdRV


PTiA0NQ==
Which one of the following statements is true of erythropoiesis and erythropoietin?

a) Hypoxia is the main stimulus to erythropoietin production


b) Erythropoietin increases the maturation time for red cell precursors
c) Erythropoietin levels are found to be low in secondary polycythaemica
d) Bilateral nephrectomy completely abolishes erythropoietin production
Correct Answer: A

Score:
DBMCI CLT MAINS 2018 755.00 /
Page 33 1200
CLT MAINS - 2018
Exam Title :
(Final)
Email : sasirekha314@gmail.com
Contact : 7708001877
Your Answer: A
Explanation

Ans. A. Hypoxia is the main stimulus to erythropoietin production.

· Hypoxia is the chief stimulus to erythropoiesis. Around 85% of erythropoietin (166 amino
acids) is produced by the kidneys and 15% is produced by the liver.

· The rate-controlling enzyme for porphyrin and haem synthesis is ALA-S. Its synthesis is
inhibited by haemoglobin in polycythemia rubra vera.

· Genetically engineered erythropoietin is now available for the treatment of anaemia caused by
renal failure.

QUESTION 47. MTQ3MStTQVNJUkVLSEEgUCtzYXNpcmVraGEzMTRAZ21haWwuY29tKzc3MDgwMDE4NzdRV


PTiA0Ng==
A noncompetitive inhibitor of an enzyme:

a) Increases Km with no or little change in Vmax


b) Decreases Km and decreases Vmax
c) Decreases Vmax
d) Increases Vmax
Correct Answer: C
Your Answer: C
Explanation

Ans. C. Decreases Vmax

· In contrast to competitive inhibitors, noncompetitive inhibitors are not structural analogues of


the substrate.

· Consequently, noncompetitive inhibitors bind to enzymes in locations remote from the active
site.

· For this reason, the degree of inhibition is based solely upon the concentration of inhibitor and
increasing the substrate concentrations do not compete with or change the inhibition.

· Therefore, unlike the increase in Km seen with competitive inhibition, in noncompetitive


inhibition Vmax increases while Km usually remains the same.

· While competitive inhibitors can be overcome at sufficiently high concentration of substrate,


noncompetitive inhibition is irreversible

QUESTION 48. MTQ3MStTQVNJUkVLSEEgUCtzYXNpcmVraGEzMTRAZ21haWwuY29tKzc3MDgwMDE4NzdRV


PTiA0Nw==
Repolarization of a nerve is due to :

a) Hydrogen ions
b) Potassium ions

Score:
DBMCI CLT MAINS 2018 755.00 /
Page 34 1200
CLT MAINS - 2018
Exam Title :
(Final)
Email : sasirekha314@gmail.com
Contact : 7708001877

c) Sodium ions
d) Calcium ions
Correct Answer: B
Your Answer: B
Explanation

Ans. B. Potassium ions

· The Na+ channels rapidly enter the inactivated state and remain in this state for a few
milliseconds before returning to the resting state.

· In addition, the direction of the electrical gradient for Na+ is reversed during the overshoot
because the membrane potential is reversed, and this limits Na+ influx.

· A third factor producing repolarization is the opening of voltage-gated K+ channels.

· The net movement of positive charge out of the cell due to K+ efflux at this time helps
complete the process of repolarization.

· The slow return of the K+ channels to the closed state also explains the after-
hyperpolarization.

QUESTION 49. MTQ3MStTQVNJUkVLSEEgUCtzYXNpcmVraGEzMTRAZ21haWwuY29tKzc3MDgwMDE4NzdRV


PTiA0OA==
Which of the following hormones stimulates gluconeogenesis?

a) Progesterone
b) Glucagon
c) Aldosterone
d) Epinephrine
Correct Answer: B
Your Answer: B
Explanation

Ans. B. Glucagon

· The balance and integration of the metabolism of fats and carbohydrates are mediated by the
hormones insulin, glucagon, epinephrine, and norepinephrine.

· All of these hormones exercise acute effects upon metabolism. Glucagon stimulates
gluconeogenesis and blocks glycolysis.

· When blood sugar levels get low, the α cells of the pancreas release glucagon. The main
targets of glucagon are the liver and adipose tissue.

· In the liver, glucagon stimulates the cyclic AMP–mediated cascade that causes
Phosphorylation of phosphorylase and glycogen synthesis.

· This effectively turns off glycogen synthase and turns on glycogen phosphorylase, thereby
causing a breakdown of glycogen and a production of glucose in liver, which ultimately raises
blood glucose levels.

Score:
DBMCI CLT MAINS 2018 755.00 /
Page 35 1200
CLT MAINS - 2018
Exam Title :
(Final)
Email : sasirekha314@gmail.com
Contact : 7708001877

· Insulin and glucagon are two antagonistic hormones that maintain the balance of sugar and
fatty acids in blood. Insulin is produced by the cells of the pancreas and its release is stimulated
by high levels of glucose in the blood. It has a number of effects, but its major effect is to allow
the entry of glucose into cells.

· Insulin also allows the dephosphorylation of key regulatory enzymes. The consequence of
these actions is to allow glycogen synthesis and storage in both muscle and liver, suppression of
gluconeogenesis, acceleration of glycolysis, promotion of the synthesis of fatty acids, and
promotion of the uptake and synthesis of amino acids into protein.

· All in all, insulin acts to promote anabolism.

QUESTION 50. MTQ3MStTQVNJUkVLSEEgUCtzYXNpcmVraGEzMTRAZ21haWwuY29tKzc3MDgwMDE4NzdRV


PTiA0OQ==
Which of the following is correct regarding the anomaly in the given Image?

a) Persistence of left posterior cardinal vein


b) Persistence of right anterior cardinal vein
c) Persistence of right posterior cardinal vein
d) Persistence of left anterior cardinal vein
Correct Answer: D
Your Answer: D
Explanation

Ans. D. Persistence of left anterior cardinal vein

A double superior vena cava is characterized by the persistence of the left anterior cardinal
vein and failure of the left brachiocephalic vein to form. The persistent left anterior cardinal
vein, the left superior vena cava, drains into the right atrium by way of the coronary sinus.

Score:
DBMCI CLT MAINS 2018 755.00 /
Page 36 1200
CLT MAINS - 2018
Exam Title :
(Final)
Email : sasirekha314@gmail.com
Contact : 7708001877

QUESTION 51. MTQ3MStTQVNJUkVLSEEgUCtzYXNpcmVraGEzMTRAZ21haWwuY29tKzc3MDgwMDE4NzdRV


PTiA1MA==
Part B: 51 to 150

The Image shows the transmission cycle of which of the following arboviruses?

a) JEV
b) WNV
c) YF
d) KFD
Correct Answer: B
Your Answer: A
Explanation

Ans. B. WNV

WEST NILE FEVER VIRUS (WNV)

· Widely distributed in Europe, Africa, Middle East, Asia, Russia and South West Africa.

· Highly prevalent in India (Maharashtra, T.N, A.P, Gujarat, Assam…).

· Closely related to JEV.

· Vector - mainly Culex;

· Reservoir - birds (crows, sparrows, jays); crow and blue jays highly susceptible and die; some
birds remain asymptomatic and become amplifying reservoirs

· Humans/horses are dead end hosts as they have insufficient viremia to infect feeding
mosquitoes.

Score:
DBMCI CLT MAINS 2018 755.00 /
Page 37 1200
CLT MAINS - 2018
Exam Title :
(Final)
Email : sasirekha314@gmail.com
Contact : 7708001877
QUESTION 52. MTQ3MStTQVNJUkVLSEEgUCtzYXNpcmVraGEzMTRAZ21haWwuY29tKzc3MDgwMDE4NzdRV
PTiA1MQ==
The extrinsic incubation period for dengue is-

a) 8-12 days
b) 20-30 days
c) 30-40 days
d) 1-5 days
Correct Answer: A
Your Answer: D
Explanation

Ans. A. 8-12 days

· The EIP is commonly defined as “the interval between the acquisition of an infectious agent by
a vector and the vector's ability to transmit the agent to other susceptible vertebrate hosts”

· For dengue, the E.I.P is 8-12 days.

QUESTION 53. MTQ3MStTQVNJUkVLSEEgUCtzYXNpcmVraGEzMTRAZ21haWwuY29tKzc3MDgwMDE4NzdRV


PTiA1Mg==
Drug having activity against for both HIV and HBV is:

a) Zidovudine
b) Lamivudine
c) Stavudine
d) Didanosine
Correct Answer: B
Your Answer: B
Explanation

Ans. B. Lamivudine

NRTIs:

These compounds inhibit both HIV- 1 and HIV-2, and several have broad- spectrum activity
against other human and animal retroviruses

Drug having activity against for both HIV and HBV are “LET”

· Lamivudine

· Emtricitabine

· Tenofovir

QUESTION 54. MTQ3MStTQVNJUkVLSEEgUCtzYXNpcmVraGEzMTRAZ21haWwuY29tKzc3MDgwMDE4NzdRV


PTiA1Mw==
Abolition of Private practice by government doctors was first proposed by:

Score:
DBMCI CLT MAINS 2018 755.00 /
Page 38 1200
CLT MAINS - 2018
Exam Title :
(Final)
Email : sasirekha314@gmail.com
Contact : 7708001877

a) Kartar Singh Committee


b) Mudaliar Committee
c) Junganwala Committee
d) Shrivastava Committee
Correct Answer: C
Your Answer: B
Explanation

Ans. C. Junganwala Committee

1967 Junganwala- integrate services- equal pay for equal work, special pay for special work,
good service condition, no private practice

QUESTION 55. MTQ3MStTQVNJUkVLSEEgUCtzYXNpcmVraGEzMTRAZ21haWwuY29tKzc3MDgwMDE4NzdRV


PTiA1NA==
Reactivation of cholinesterase enzyme occurs on hydrolysis of the inhibitor by the
same enzyme molecule in case of the following anticholinesterase:

a) Edrophonium
b) Neostigmine
c) Dyflos
d) Tacrine
Correct Answer: B
Your Answer: B
Explanation

Ans. B. Neostigmine

· Edrophonium and tacrine attach only to the anionic site of AChE via weak electrostatic bonds.
Reactivation of enzyme occurs in < 10 min, and does not involve hydrolysis of the inhibitor, but
only its diffusion

· Carbamates like neostigmine attach to both Anionic & esteritic site while OGPs (dyflos) attach
only to esteratic site of the AChE enzyme.

· The carbamylated enzyme undergoes slow hydrolysis (30 minutes to 6 hours) - Considered
Reversible inhibitors

· The phosphorylated enzyme has covalent bonds. It is extremely stable and reacts with water
very slowly (hundreds of hours) – considered Irreversible inhibitors.

QUESTION 56. MTQ3MStTQVNJUkVLSEEgUCtzYXNpcmVraGEzMTRAZ21haWwuY29tKzc3MDgwMDE4NzdRV


PTiA1NQ==
Adenovirus oncogenic in newborn hamsters:

a) Adenovirus type 18
b) Adenovirus type 8
c) Entero virus 72

Score:
DBMCI CLT MAINS 2018 755.00 /
Page 39 1200
CLT MAINS - 2018
Exam Title :
(Final)
Email : sasirekha314@gmail.com
Contact : 7708001877

d) Adenovirus type 7
Correct Answer: A
Your Answer: Unanswered
Explanation

Ans. A. Adenovirus type 18

· There are 57 serotypes of adenoviruses.

· Types 12, 18 and 31 are able to induce tumors when inoculated into new born hamsters.

· However, adenoviruses have not yet directly been implicated in tumors in humans.

QUESTION 57. MTQ3MStTQVNJUkVLSEEgUCtzYXNpcmVraGEzMTRAZ21haWwuY29tKzc3MDgwMDE4NzdRV


PTiA1Ng==
Diagnostic microscopy seen in:

a) Ependymoma
b) Oligodendroglioma
c) Glioblastoma multiforme
d) Medulloblastoma
Correct Answer: A
Your Answer: A
Explanation

Ans. A. Ependymoma

· Perivascular rosette and true rosettes are seen in ependymoma

Score:
DBMCI CLT MAINS 2018 755.00 /
Page 40 1200
CLT MAINS - 2018
Exam Title :
(Final)
Email : sasirekha314@gmail.com
Contact : 7708001877

· Medulloblastoma shows homer Wright rosettes

QUESTION 58. MTQ3MStTQVNJUkVLSEEgUCtzYXNpcmVraGEzMTRAZ21haWwuY29tKzc3MDgwMDE4NzdRV


PTiA1Nw==
A man working as a pest killer comes to OPD with pain abdomen, garlic odour in
breath and transverse lines on nails. The person may be having :

a) Arsenic poisoning
b) Lead poisoning
c) Mercury poisoning
d) Cadmium poisoning
Correct Answer: A
Your Answer: A
Explanation

Ans. A. Arsenic poisoning

Acute Arsenic poisoning is a metallic taste in the mouth and some odor of garlic in the breath.

Common characteristics include:

• Abdominal pain
• Watery diarrhea
• Nausea
• Vomiting

After monthly post an Acute Intoxication, which transverse white striae in the nails (Aldrich-
Mees lines) and it may become apparent.

QUESTION 59. MTQ3MStTQVNJUkVLSEEgUCtzYXNpcmVraGEzMTRAZ21haWwuY29tKzc3MDgwMDE4NzdRV


PTiA1OA==
Which of the following is not useful for “cepacia syndrome”?

a) Meropenem
b) Cotrimoxazole
c) Amikacin
d) Doxycycline
Correct Answer: C
Your Answer: A
Explanation

Ans. C. Amikacin

Cepacia syndrome

· Rapidly fatal syndrome of respiratory distress and septicaemia caused by B. cepacia in Cystic
fibrosis patients

Score:
DBMCI CLT MAINS 2018 755.00 /
Page 41 1200
CLT MAINS - 2018
Exam Title :
(Final)
Email : sasirekha314@gmail.com
Contact : 7708001877

· B. cepacia is intrinsically resistant to many antibiotics

· Cotrimoxazole, meropenem, and doxycycline are the most effective agents in vitro and may be
started as first line agents

Aminoglycosides do not have activity against anaerobes, S. maltophilia, or Burkholderia


cepacia.

QUESTION 60. MTQ3MStTQVNJUkVLSEEgUCtzYXNpcmVraGEzMTRAZ21haWwuY29tKzc3MDgwMDE4NzdRV


PTiA1OQ==
Virus-like particles (VLPs) are present in-

a) Small pox vaccine


b) HPV vaccine
c) Measles vaccine
d) Mumps vaccine
Correct Answer: B
Your Answer: B
Explanation

Ans. B. HPV vaccine

· The vaccines use virus-like particles (VLPs) that consist of the HPV L1 major capsid protein.

· The L1 protein self-assembles into VLPs when expressed in eukaryotic cells (i.e., yeast for
Gardasil vaccine or insect cells for the Cervarix vaccine). These VLPs contain the same epitopes
as the HPV virion. However, they do not contain genetic material and cannot transmit infection.

QUESTION 61. MTQ3MStTQVNJUkVLSEEgUCtzYXNpcmVraGEzMTRAZ21haWwuY29tKzc3MDgwMDE4NzdRV


PTiA2MA==
Identify the vector:

a) Sandfly
b) TseTse fly
c) Mansonia mosquito
d) Simulum fly
Correct Answer: A
Your Answer: A
Explanation

Ans. A. Sandfly

· Phlebotomus

· Habitat- Holes and crevices in walls, tress, dark rooms, sand bed

· Insecticide- DDT (1-2 gm/m2 )

Score:
DBMCI CLT MAINS 2018 755.00 /
Page 42 1200
CLT MAINS - 2018
Exam Title :
(Final)
Email : sasirekha314@gmail.com
Contact : 7708001877

· Sandfly hops DDT sprayed till 4-6 feet height. only female bites

· P. Argentipes- Kala Azar( Visceral Leishmaniasis) P. Papatasi and P.sargeinti- Oriental sore (
cutaneous L.)

QUESTION 62. MTQ3MStTQVNJUkVLSEEgUCtzYXNpcmVraGEzMTRAZ21haWwuY29tKzc3MDgwMDE4NzdRV


PTiA2MQ==
A 48 year old immunocompromised patients attended the emergency block with
complaints of abdominal pain, vomiting and severe diarrhea) Stool examination is
done. Structure shown in the figure are seen on microscopy. Which is the first
infection that should come in the differential diagnosis?

a) Strongyloides
b) Trichuris
c) Wuchereria
d) Ascaris
Correct Answer: A
Your Answer: A
Explanation

Ans. A. Strongyloides.

· This is the rhabditiform larva of Strongyloides in the stool.

· Since the females are ovo-viviparous, eggs are not seen. In immunocompromised,
disseminated infection can occur

· Strongyloides is the smallest nematode and rhabditiform larvae is the most common form seen
in the stool specimen.

QUESTION 63. MTQ3MStTQVNJUkVLSEEgUCtzYXNpcmVraGEzMTRAZ21haWwuY29tKzc3MDgwMDE4NzdRV


PTiA2Mg==
Amyloid deposits stain positively with all the following except:

Score:
DBMCI CLT MAINS 2018 755.00 /
Page 43 1200
CLT MAINS - 2018
Exam Title :
(Final)
Email : sasirekha314@gmail.com
Contact : 7708001877

a) Congo-red
b) Crystal violet
c) Methenamine silver
d) Thioflavin T
Correct Answer: C
Your Answer: B
Explanation

Ans. C. Methenamine silver

Staining Characteristics of Amyloid

Stain used-Lugol’s iodine


Stain on gross Appearance: Imparts purple colour, on addition of dilute
sulphuric acid turns blue.

Stain used-Hematoxylin & Eosin


Hematoxylin &
Appearance- On light microscopy appear as extracellular
Eosin
Homogeneous structure less eosinophilic hyaline material

Metachromatic Stain used- Methyl violet and crystal violet


stains (Rosaline
dyes) Appearance- Rose pink coloration of amyloid deposits

Congo red and Stain used – Congo red


polarized light(All
types of amyloid Appearance – Ordinary light
have affinity for
congo red stain) Polarised –Apple green birefringence light

Stain used Thioflavin- T


Fluorescent stains
Appearance- Yellow under ultraviolet light

Stain used – Various antibody stains against the specific


Immunohistochemi antigenic protein types e.g. Anti
stry
AP anti AA, anti Lambda, anti kappa

Score:
DBMCI CLT MAINS 2018 755.00 /
Page 44 1200
CLT MAINS - 2018
Exam Title :
(Final)
Email : sasirekha314@gmail.com
Contact : 7708001877

Stain used: Toluidine blue

Alcian blue

Non specific stains Periodic acid Schiff

Colour imparted: Orthochromatic blue

Blue green colour

QUESTION 64. MTQ3MStTQVNJUkVLSEEgUCtzYXNpcmVraGEzMTRAZ21haWwuY29tKzc3MDgwMDE4NzdRV


PTiA2Mw==
Under WHO criteria, who is called a blind?

a) <3/60-1/60
b) <1/60-perception
c) No perception of light
d) All of the above
Correct Answer: D
Your Answer: D
Explanation

Ans. D. All of the above

No perception of light is absolute blindness.

WHO defines <3/60 – 1/60 as blindness . Hence, all of the above

QUESTION 65. MTQ3MStTQVNJUkVLSEEgUCtzYXNpcmVraGEzMTRAZ21haWwuY29tKzc3MDgwMDE4NzdRV


PTiA2NA==
Pale infarcts are seen at all of the following sites except:

a) Heart
b) Spleen
c) Kidney
d) Lung
Correct Answer: D
Your Answer: D
Explanation

Ans. D. Lung

Score:
DBMCI CLT MAINS 2018 755.00 /
Page 45 1200
CLT MAINS - 2018
Exam Title :
(Final)
Email : sasirekha314@gmail.com
Contact : 7708001877

1. Venous occlusion (eg ovarian torsion):

2. In loose tissues (such as lungs):

Red infarcts 3. In tissues with dual circulation (e.g. Lung & S. Intestine)
(Haemorrhagic): Occur
with 4. In tissues that were previously congested because of
sluggish venous out flow.

5. When flow is re established to a site of previous arterial


occlusion and necrosis.

1. Arterial occlusions, or

White infarcts: occur 2. In solid organs e.g. Heart, spleen, kidney and brain
with where the solidity of tissue limits the amount of
hemorrhage that can seep into the area of ischemic
necrosis.

QUESTION 66. MTQ3MStTQVNJUkVLSEEgUCtzYXNpcmVraGEzMTRAZ21haWwuY29tKzc3MDgwMDE4NzdRV


PTiA2NQ==
Lidocaine dose should be decreased in a patient with all except:

a) Shock
b) Liver failure
c) Concomitant beta blocker use
d) MI
Correct Answer: D
Your Answer: D
Explanation

Ans. D. MI

Lidocaine dose should be decreased with

· Heart failure: As volume of distribution decreases.

· Liver failure

· Concomitant cispride or beta-blocker therapy

· Prolonged infusion

Lidocaine dose should be increased in

· MI: As lidocaine binds to beta-1 acidic glycoprotein, which is increased in MI. The free drug
decreases and hence for the same clinical effect dose needs to be increased.

Score:
DBMCI CLT MAINS 2018 755.00 /
Page 46 1200
CLT MAINS - 2018
Exam Title :
(Final)
Email : sasirekha314@gmail.com
Contact : 7708001877
QUESTION 67. MTQ3MStTQVNJUkVLSEEgUCtzYXNpcmVraGEzMTRAZ21haWwuY29tKzc3MDgwMDE4NzdRV
PTiA2Ng==
Overlapping of bones in intrauterine death is known as:

a) Spalding Sign
b) Robert’s Sign
c) Wood's Sign.
d) Ewald’s sign
Correct Answer: A
Your Answer: A
Explanation

Ans. A. Spalding Sign

Overlapping of bones in intrauterine death is known as Spalding Sign

QUESTION 68. MTQ3MStTQVNJUkVLSEEgUCtzYXNpcmVraGEzMTRAZ21haWwuY29tKzc3MDgwMDE4NzdRV


PTiA2Nw==
In a town there are 2500 live births within six months. During the same period, 5
women died due to peri-partum infection, 5 died due to electrocution, 2 died due to
obstructed labour, 3 died due to PPH. What is the MMR?

a) 4 per 1000 LB
b) 6 per 1000 LB
c) 40 per 1000 LB
d) 60 per 1000 LB
Correct Answer: A
Your Answer: A
Explanation

Ans. A. 4 per 1000 LB

Live births 2500, No of deaths as per definition= 5 due to infections + 2 due to obstructed
labour+ 3 due to PPH, due to electrocution cannot be included.

So 5+2+3/2500*1000= 4

Death of a women while pregnant, or during delivery or within 42 days ( 6 weeks) of


termination of pregnancy irrespective of site and duration, from any cause related to or
aggravated by the pregnancy or its management but not from accidental or incidental causes.
It’s a ratio, / 1 lakh live births. World 210/ 1 lakh live birth

MCC world- Hemorrhage, India MCC- Hemorrhage .

SDG by 2030 < then 70/lakh. NHP 2017- 100 by 2020

QUESTION 69. MTQ3MStTQVNJUkVLSEEgUCtzYXNpcmVraGEzMTRAZ21haWwuY29tKzc3MDgwMDE4NzdRV


PTiA2OA==

Score:
DBMCI CLT MAINS 2018 755.00 /
Page 47 1200
CLT MAINS - 2018
Exam Title :
(Final)
Email : sasirekha314@gmail.com
Contact : 7708001877

Who is considered to be the father of occupational health:

a) Bernadino Ramazzini
b) John Snow
c) Ignaz Semmelweis
d) Robert Koch
Correct Answer: A
Your Answer: C
Explanation

Ans. A. Bernadino Ramazzini

· Bernadino Ramazzini - father of occupational health

· John Snow – father of epidemiology

· Ignaz Semmelweis pioneer of handwashing

· Robert Koch – Tuberculosis, germ theory

QUESTION 70. MTQ3MStTQVNJUkVLSEEgUCtzYXNpcmVraGEzMTRAZ21haWwuY29tKzc3MDgwMDE4NzdRV


PTiA2OQ==
Recent drug approved for tardive dyskinesia is:

a) Pimavanserin
b) Valbenazine
c) Lofexidine
d) Botox
Correct Answer: B
Your Answer: A
Explanation

Ans. B. Valbenazine

Recent VMAT2 inhibitors approved for treatment of tardive dyskinesia are

· De tetrabenazine

· Valbenazine

QUESTION 71. MTQ3MStTQVNJUkVLSEEgUCtzYXNpcmVraGEzMTRAZ21haWwuY29tKzc3MDgwMDE4NzdRV


PTiA3MA==
Perl’s stain is used to demonstrate:

a) Melanin
b) Hemosiderin
c) Bilirubin
d) Lipofuscin

Score:
DBMCI CLT MAINS 2018 755.00 /
Page 48 1200
CLT MAINS - 2018
Exam Title :
(Final)
Email : sasirekha314@gmail.com
Contact : 7708001877
Correct Answer: B
Your Answer: B
Explanation

Ans. B. Hemosiderin

Perl’s stain (Prussian blue stain) is used for demonstration of hemosiderin in tissues 1471

QUESTION 72. MTQ3MStTQVNJUkVLSEEgUCtzYXNpcmVraGEzMTRAZ21haWwuY29tKzc3MDgwMDE4NzdRV


PTiA3MQ==
True statement regarding gonococcal urethritis-

a) Symptoms are more severe in females.


b) Rectum and prostate are resistant to gonococci
c) Most patients present with dysuria
d) Single dose of ciprofloxacin is effective for treatment.
Correct Answer: C
Your Answer: C
Explanation

Ans. C. Most patients present with dysuria.

· Males commonly are symptomatic (dysuria, urethral discharge) in comparison to females who
are often asymptomatic.

· Prostatitis and epididymitis may occur following ascent of the infection. Proctitis is seen
following anal sex

· Third-generation cephalosporins have remained highly effective as single-dose therapy for


gonorrhea.

· Ciprofloxacin is effective only when given for a week.

QUESTION 73. MTQ3MStTQVNJUkVLSEEgUCtzYXNpcmVraGEzMTRAZ21haWwuY29tKzc3MDgwMDE4NzdRV


PTiA3Mg==
This type of killing is a combination of:

Score:
DBMCI CLT MAINS 2018 755.00 /
Page 49 1200
CLT MAINS - 2018
Exam Title :
(Final)
Email : sasirekha314@gmail.com
Contact : 7708001877

a) Smothering + throttling
b) Smothering + Mugging
c) Smothering + Traumatic asphyxia
d) Smothering + Ligature strangulation
Correct Answer: C
Your Answer: C
Explanation

Ans. C. Smothering & traumatic asphyxia

· Asphyxia by smothering is caused by blocking air entry into the lungs by simultaneous closure
of the nose and mouth.

· They are usually homicidal, rarely suicidal and very rarely accidental. Bruises or abrasions on
the cheeks, around the mouth, lips or lesions within the lips or mouth are the features of
smothering.

QUESTION 74. MTQ3MStTQVNJUkVLSEEgUCtzYXNpcmVraGEzMTRAZ21haWwuY29tKzc3MDgwMDE4NzdRV


PTiA3Mw==
Abhiyan Indradhanush is related to which program:

a) National Pension scheme


b) National Insurance Scheme
c) Employee state Insurance scheme
d) Immunization Scheme
Correct Answer: C
Your Answer: C
Explanation

Ans. C. Employee state Insurance scheme

· ESIC 2.0- 2015 launch- Abhiyan Indradhanush- daily new bed sheet VIBGYOR pattern- Sunday
violet, Saturday red.

· Special OPD for geriatric and disabled, online health records- Project panchdeep

Score:
DBMCI CLT MAINS 2018 755.00 /
Page 50 1200
CLT MAINS - 2018
Exam Title :
(Final)
Email : sasirekha314@gmail.com
Contact : 7708001877

QUESTION 75. MTQ3MStTQVNJUkVLSEEgUCtzYXNpcmVraGEzMTRAZ21haWwuY29tKzc3MDgwMDE4NzdRV


PTiA3NA==
Which of the following beta blockers can cause nightmares?

a) Sotalol
b) Pindolol
c) Nadolol
d) Betaxolol
Correct Answer: B
Your Answer: D
Explanation

Ans. B. Pindolol

Pindolol being a lipid soluble drug can across blood brain barrier and cause central side-effects
like nightmares and seizures.

QUESTION 76. MTQ3MStTQVNJUkVLSEEgUCtzYXNpcmVraGEzMTRAZ21haWwuY29tKzc3MDgwMDE4NzdRV


PTiA3NQ==
Child Pugh classification is related to :

a) Hepatitis
b) Cirrhosis
c) Jaundice
d) Cholangitis
Correct Answer: B
Your Answer: B
Explanation

Ans. B. Cirrhosis

The Child-Pugh classification of cirrhosis distinguishes between class A (well compensated), B


(partially decompensated), and C (decompensated), which correlate with different morphologic
features histologically.

QUESTION 77. MTQ3MStTQVNJUkVLSEEgUCtzYXNpcmVraGEzMTRAZ21haWwuY29tKzc3MDgwMDE4NzdRV


PTiA3Ng==
A 35 year old man who is known to be infected with HIV complains that he has had a
“bad” taste in his mouth and discoloration of his tongue for the past 6 weeks. On
physical examination, there are areas of adherent, yellow-tn, circumscribed plaque on
the lateral aspects of the tongue. This plaque can be scraped off as a pseudomembrane
to show an underlying granular, erythematous base. Which of the following is the most
likely diagnosis?

a) Aphthous ulcer

Score:
DBMCI CLT MAINS 2018 755.00 /
Page 51 1200
CLT MAINS - 2018
Exam Title :
(Final)
Email : sasirekha314@gmail.com
Contact : 7708001877

b) Cheilosis
c) Hairy leukoplakia
d) Oral thrush
Correct Answer: D
Your Answer: D
Explanation

Ans. D. Oral thrush

· The patient has oral thrush, a lesion resulting from oral candidiasis in persons who are
immunocompromised.

· The lesson is typically superficial

QUESTION 78. MTQ3MStTQVNJUkVLSEEgUCtzYXNpcmVraGEzMTRAZ21haWwuY29tKzc3MDgwMDE4NzdRV


PTiA3Nw==
True about V. cholera-

a) One attack gives lifelong immunity


b) Affects children and adults equally in the non-endemic regions
c) Carriers maintain the organism in the inter-epidemic period
d) Pathogenicity of O139 is due to O antigen
Correct Answer: B
Your Answer: D
Explanation

Ans. B. Affects children and adults equally in the non-endemic regions.

· One attack gives short-lasting immunity (6-12 mo).

· During the inter-epidemic periods, the vibrios are maintained in saline waters of coastal seas
and brackish estuaries.

· Pathogenicity of V. cholerae is not d/t O antigen but specific virulence factors like CT, Tc

QUESTION 79. MTQ3MStTQVNJUkVLSEEgUCtzYXNpcmVraGEzMTRAZ21haWwuY29tKzc3MDgwMDE4NzdRV


PTiA3OA==
All are true about type III hypersensitivity except:

a) Serum sickness
b) Arthus reaction
c) Anaphylaxis
d) Lupus nephritis
Correct Answer: C
Your Answer: C
Explanation

Ans. C. Anaphylaxis

Score:
DBMCI CLT MAINS 2018 755.00 /
Page 52 1200
CLT MAINS - 2018
Exam Title :
(Final)
Email : sasirekha314@gmail.com
Contact : 7708001877

· Anaphylaxis is a type I hypersensitivity reaction Immediate, or type I, hypersensitivity is a


rapid immunologic reaction occurring within minutes after the combination of an antigen with
antibody bound to mast cells in individuals previously sensitilized to the antigen .

· These reactions are often called allergy, and the antigens that elicit them are allergens
immediate hypersensitivity may occur as a systemic disorder as a local reaction.

QUESTION 80. MTQ3MStTQVNJUkVLSEEgUCtzYXNpcmVraGEzMTRAZ21haWwuY29tKzc3MDgwMDE4NzdRV


PTiA3OQ==
Motile spermatozoa found in wet mount of vaginal secretions are indicative of
intercourse within the past :

a) 6 hours
b) 12 hours
c) 24 hours
d) 48 hours
Correct Answer: C
Your Answer: B
Explanation

Ans. C. 24 hours

Research had proved that following sexual intercourse motile spermatozoa should be found in
vagina for 24 hrs,that they are likely to be found up to 3 days later, and occasionally they are
found 7 days later.

QUESTION 81. MTQ3MStTQVNJUkVLSEEgUCtzYXNpcmVraGEzMTRAZ21haWwuY29tKzc3MDgwMDE4NzdRV


PTiA4MA==
What is Quetelet’s index:

a) Weight /height in meter 2


b) Height in cm-100
c) Birth wt X 100/ length 3
d) Retained Nitrogen/Absorbed Nitrogen X 100
Correct Answer: A
Your Answer: A
Explanation

Ans. A. Weight /height in meter 2

· BMI or Quetelet’s index is same

· Height in cm-100 – Broca’s index

· Birth wt X 100/ length 3 – Ponderal index

Score:
DBMCI CLT MAINS 2018 755.00 /
Page 53 1200
CLT MAINS - 2018
Exam Title :
(Final)
Email : sasirekha314@gmail.com
Contact : 7708001877

· Retained Nitrogen/Absorbed Nitrogen X 100 – Biological value

QUESTION 82. MTQ3MStTQVNJUkVLSEEgUCtzYXNpcmVraGEzMTRAZ21haWwuY29tKzc3MDgwMDE4NzdRV


PTiA4MQ==
Which of the following causes a dead end infection?

a) C. tetani
b) V. cholerae
c) S. aureus
d) Hemophilus
Correct Answer: A
Your Answer: A
Explanation

Ans. A. C. tetani .

Since, there is no further human to human transmission, tetanus is a dead end infection.

QUESTION 83. MTQ3MStTQVNJUkVLSEEgUCtzYXNpcmVraGEzMTRAZ21haWwuY29tKzc3MDgwMDE4NzdRV


PTiA4Mg==
Shown in the figure is the wet mount of urine sediment. The organism seen has which
of the following character?

Score:
DBMCI CLT MAINS 2018 755.00 /
Page 54 1200
CLT MAINS - 2018
Exam Title :
(Final)
Email : sasirekha314@gmail.com
Contact : 7708001877

a) Has falling leaf motility


b) Has four pairs of flagella
c) Has an undulating membrane
d) Cyst and trophozoite in the life cycle
Correct Answer: C
Your Answer: B
Explanation

Ans. C. Has an undulating membrane.

· Image shows the trophozoite of Trichomonas which has no cyst stage.

· The T. vaginalis trophozoite is oval or pear shaped. It is slightly larger than a white blood cell,
measuring 9 X 7 μm.

· It has four flagellae and a fifth flagellum is folded into an undulating membrane.

· It has jerky or wobbling motility.

· T. vaginalis inhabits the lower genital tract of females and the urethra and prostate of males.
Hence can be seen in the urine sediment.

QUESTION 84. MTQ3MStTQVNJUkVLSEEgUCtzYXNpcmVraGEzMTRAZ21haWwuY29tKzc3MDgwMDE4NzdRV


PTiA4Mw==
The most common gene defect in idiopathic steroid resistant nephritic syndrome:

a) ACE
b) NPHS 2
c) HOX 11
d) PAX
Correct Answer: B
Your Answer: B
Explanation

Ans. B. NPHS 2

Score:
DBMCI CLT MAINS 2018 755.00 /
Page 55 1200
CLT MAINS - 2018
Exam Title :
(Final)
Email : sasirekha314@gmail.com
Contact : 7708001877

· Genetic basis of proteinuria in nephrotic syndrome

· Recently certain gene mutations have been recognized which are associated with certain
glomerulonephritis, producing nephrotic syndrome.

· The gene mutations codes certain proteins and the common feature of these proteins is their
localization to the structures of the glomerular filtration barrier; such as slit diaphragm and
Podocyte cytoskeletal structures such as actin.

· Their specific functions and interaction are incompletely understood, but it is clear that the
integrity of each is necessary to maintain the normal glomerular filtration barrier

Gene Chromosome Protein Disease

NPHS1 19q13 Nephrin Nephrotic syndrome of finish type

NPHS2 1q25-31 Podocin Steroid resistant nephrotic syndrome

QUESTION 85. MTQ3MStTQVNJUkVLSEEgUCtzYXNpcmVraGEzMTRAZ21haWwuY29tKzc3MDgwMDE4NzdRV


PTiA4NA==
Which of the following is slowest acting receptor?

a) Beta 1 receptor
b) M2 receptor
c) Tyrosine kinase receptor
d) Mineralocorticoid receptor
Correct Answer: D
Your Answer: D
Explanation

Ans. D. Mineralocorticoid receptor

· Nuclear receptor are the slowest acting receptors.

· The only nuclear receptor in the options is mineralocorticoid receptors.

QUESTION 86. MTQ3MStTQVNJUkVLSEEgUCtzYXNpcmVraGEzMTRAZ21haWwuY29tKzc3MDgwMDE4NzdRV


PTiA4NQ==
In which of the condition, neostigmine is beneficial symptomatically?

Score:
DBMCI CLT MAINS 2018 755.00 /
Page 56 1200
CLT MAINS - 2018
Exam Title :
(Final)
Email : sasirekha314@gmail.com
Contact : 7708001877

a)

b)

c)

d)

Correct Answer: A
Your Answer: A
Explanation

Ans. A. Cobra

Option A is cobra snake; option B is Mamba snake; Option C is Black widow spider; Option D is
Krait snake.

Score:
DBMCI CLT MAINS 2018 755.00 /
Page 57 1200
CLT MAINS - 2018
Exam Title :
(Final)
Email : sasirekha314@gmail.com
Contact : 7708001877

QUESTION 87. MTQ3MStTQVNJUkVLSEEgUCtzYXNpcmVraGEzMTRAZ21haWwuY29tKzc3MDgwMDE4NzdRV


PTiA4Ng==
As per FSSAI act 2008, the level of iodization of salt at production level of salt should
be:

a) 15 ppm
b) 20 ppm
c) 25 ppm
d) 30 ppm
Correct Answer: D
Your Answer: D
Explanation

Ans. D. 30 ppm

· 30 ppm at production level- 20-40 mg of iodine / kg of salt (iodine is mixed as potassium


iodate)

· 15 ppm at consumer level min ( 15 mg /kg of salt)

· Double fortified salt (DFS)- iron +iodine- 40 microgram of iodine + 1 mg iron per gram of salt

· Iodized oil ( for administration)- oral 2 ml single dose for 2 years and IM 1 ml single dose for 4
years

QUESTION 88. MTQ3MStTQVNJUkVLSEEgUCtzYXNpcmVraGEzMTRAZ21haWwuY29tKzc3MDgwMDE4NzdRV


PTiA4Nw==
Storage conditions which enhance botulinum toxin production in preserved food are
all except:

a) An anaerobic environment
b) pH of >4.6
c) High salt and sugar concentrations
d) Temperatures of >4°c
Correct Answer: C
Your Answer: Unanswered
Explanation

Ans. C. High salt and sugar concentrations

Toxin production in canned food with spores of C. botulinum requires a rare confluence of
product storage conditions: An anaerobic environment, a pH of >4.6, low salt and sugar
concentrations, and temperatures of >4°C.

QUESTION 89. MTQ3MStTQVNJUkVLSEEgUCtzYXNpcmVraGEzMTRAZ21haWwuY29tKzc3MDgwMDE4NzdRV


PTiA4OA==

Score:
DBMCI CLT MAINS 2018 755.00 /
Page 58 1200
CLT MAINS - 2018
Exam Title :
(Final)
Email : sasirekha314@gmail.com
Contact : 7708001877

DALY is:

a) No. of years lost due to ill health, disability or early death


b) Time spent in poor health
c) No. of years of life that would be added by intervention
d) No. of years an individual is expected to live free of disability
Correct Answer: A
Your Answer: A
Explanation

Ans. A. No. of years lost due to ill health, disability or early death

· Time spent in poor health- HALE

· No. of years of life that would be added by intervention - QALY

· No. of years an individual is expected to live free of disability - DFLE

QUESTION 90. MTQ3MStTQVNJUkVLSEEgUCtzYXNpcmVraGEzMTRAZ21haWwuY29tKzc3MDgwMDE4NzdRV


PTiA4OQ==
Rifampicin and ritonavir are:

a) Physical antagonist
b) Chemical antagonist
c) Pharmacokinetic antagonist
d) Competitive antagonist
Correct Answer: C
Your Answer: C
Explanation

Ans. C. Pharmacokinetic antagonist

· Rifampicin is an enzyme inducer of CY3A4 and ritonavir is metabolized by CYP3A4.

· Thus rifampicin can antagonize ritonavir by inducing its metabolism; known as


pharmacokinetic antagonism.

QUESTION 91. MTQ3MStTQVNJUkVLSEEgUCtzYXNpcmVraGEzMTRAZ21haWwuY29tKzc3MDgwMDE4NzdRV


PTiA5MA==
A H & E section of a tissue showing inflammation is given below. Identify the cell:

Score:
DBMCI CLT MAINS 2018 755.00 /
Page 59 1200
CLT MAINS - 2018
Exam Title :
(Final)
Email : sasirekha314@gmail.com
Contact : 7708001877

a) Plasma cell
b) Eosinophil
c) Lymphocyte
d) Macrophage
Correct Answer: A
Your Answer: D
Explanation

Ans. A. Plasma cell

Plasma cells Ovoid cells with abundant deep blue cytoplasm and perinuclear hof, eccentric
nucleus with coarse chromatin and clock face (art wheel) pattern

QUESTION 92. MTQ3MStTQVNJUkVLSEEgUCtzYXNpcmVraGEzMTRAZ21haWwuY29tKzc3MDgwMDE4NzdRV


PTiA5MQ==
Irreversible injury in MI occurs within:

a) 40 min
b) 10 min
c) 60 min

Score:
DBMCI CLT MAINS 2018 755.00 /
Page 60 1200
CLT MAINS - 2018
Exam Title :
(Final)
Email : sasirekha314@gmail.com
Contact : 7708001877

d) 90 min
Correct Answer: A
Your Answer: A
Explanation

Ans. A. 40 min

· Irreversible injury occurs within 20-40 min

· Approximate Time of Onset of key events in ischemic cardiac Myocytes

Feature Time

Onset of ATP depletion


Seconds
Loss of contractility
<2 min
ATP reduced
10min
to 50% of normal
40 min
to 10% normal
20-40 min
Irreversible cell injury
>1 hr
Microvascular injury

QUESTION 93. MTQ3MStTQVNJUkVLSEEgUCtzYXNpcmVraGEzMTRAZ21haWwuY29tKzc3MDgwMDE4NzdRV


PTiA5Mg==
In Primary health care , ABC and VED are related to management of what:

a) National program evaluation at PHC level


b) Drug inventory management at PHC
c) Staff management at PHC
d) Vaccination coverage assessment in PHC area
Correct Answer: B
Your Answer: B
Explanation

Ans. B. Drug inventory management at PHC

· ABC- Always better control A- outstandingly important ( very tight control as costly), B
average important ( less tight control as medium cost), C relatively unimportant ( least control)
for maintaining drug inventory and rationalizing the budget.

· VED, V- Vital- very important 10%, E- essential drugs- ok can come in some time ( 40% of total
item), D- Desirable – can wait for long time ( 50% of drugs), so explain with name of drugs

· FIFO- first in first out, HML- high cost, medium cost, low cost, FSN- fast, slow, non moving,
SDR- scarce, difficult, easy to acquire

Score:
DBMCI CLT MAINS 2018 755.00 /
Page 61 1200
CLT MAINS - 2018
Exam Title :
(Final)
Email : sasirekha314@gmail.com
Contact : 7708001877

QUESTION 94. MTQ3MStTQVNJUkVLSEEgUCtzYXNpcmVraGEzMTRAZ21haWwuY29tKzc3MDgwMDE4NzdRV


PTiA5Mw==
Movement of Listeria monocytogenes inside of host cells is caused by:

a) Inducing host cell actin polymerization


b) The formation of pili (fimbriae) on the listeriae surface
c) Pseudopod formation
d) The motion of listeriae flagella
Correct Answer: A
Your Answer: A
Explanation

Ans. A. Inducing host cell actin polymerization

· Act, a surface protein, induces host cell actin polymerization, which propels them to the cell
membrane. Pushing against the host cell membrane, they cause formation of elongated
protrusions called filopods.

· These filopods are ingested by adjacent epithelial cells, macrophages, and hepatocytes, the
listeriae are released, and the cycle begins again.

· L monocytogenes can move from cell to cell without being exposed to antibodies, complement,
or PMN’s. Shigella flexneri and Rickettsiae also usurp the host cells’ actin and contractile
system to spread their infections.

QUESTION 95. MTQ3MStTQVNJUkVLSEEgUCtzYXNpcmVraGEzMTRAZ21haWwuY29tKzc3MDgwMDE4NzdRV


PTiA5NA==
The most common histological type of thyroid cancer is :

a) Medullary type
b) Follicular type
c) Papillary type
d) Anaplastic type
Correct Answer: C
Your Answer: C
Explanation

Ans. C. Papillary type

Papillary carcinoma of thyroid is the most common thyroid carcinoma, comprising 60-70% of
cases.

QUESTION 96. MTQ3MStTQVNJUkVLSEEgUCtzYXNpcmVraGEzMTRAZ21haWwuY29tKzc3MDgwMDE4NzdRV


PTiA5NQ==
What are the total number of teeth at the age of 11?

Score:
DBMCI CLT MAINS 2018 755.00 /
Page 62 1200
CLT MAINS - 2018
Exam Title :
(Final)
Email : sasirekha314@gmail.com
Contact : 7708001877

a) 2
b) 20
c) 24
d) 30
Correct Answer: C
Your Answer: C
Explanation

Ans. C. 24

Number of teeth are 24 at the age of 11 out of which, 20 are permanent comprising of-

• 8 incisors
• 4 molars
• 8 premolars and 4 deciduous teeth.

Number of Teeth with age:

Age Number of teeth

2-5 20(all deciduous)

6 21-24(eruption of first permanent molars)

7-9 24(12 permanent, 12 deciduous)

10 24(16 permanent, 8 deciduous)

11 24(20 permanent, 4 deciduous)

12 - 14 25-28(eruption of second permanent molars)

14-17 28(all permanent)

17-25 29-32(eruption of 3rd molars)

QUESTION 97. MTQ3MStTQVNJUkVLSEEgUCtzYXNpcmVraGEzMTRAZ21haWwuY29tKzc3MDgwMDE4NzdRV


PTiA5Ng==
What is wrong regarding Pradhan Mantri Surakshit Matritva Abhiyan:

a) The scheme is applicable for only pregnant women


b) The free checkup will take place on 9 th of every month
c) All kinds of medical checkups under this scheme will completely be free
d) Promotion of breast feeding
Correct Answer: D
Your Answer: C
Explanation

Ans. D. Promotion of breast feeding

· PMSMA is the scheme launched for pregnant women under which free checkups will be done
on 9th of every month.

Score:
DBMCI CLT MAINS 2018 755.00 /
Page 63 1200
CLT MAINS - 2018
Exam Title :
(Final)
Email : sasirekha314@gmail.com
Contact : 7708001877

· Breast feeding promotion is not a part of PMSMA rather it’s in another initiative known as
MAA (mothers absolute affection)

QUESTION 98. MTQ3MStTQVNJUkVLSEEgUCtzYXNpcmVraGEzMTRAZ21haWwuY29tKzc3MDgwMDE4NzdRV


PTiA5Nw==
All of the following statements are true about diphtheria except:

a) Toxin production depends on lysogenic conversion by beta phage


b) Non sporing, non motile and non capsulated
c) Toxin detection is done by ELISA test in serum
d) Faucial diphtheria is commonest type
Correct Answer: C
Your Answer: B
Explanation

Ans. C. Toxin detection is done by ELISA test in serum.

Toxin is produced locally and is absorbed from the local site of multiplication causing toxemia.
However, toxin may not always be detectable in the serum.

QUESTION 99. MTQ3MStTQVNJUkVLSEEgUCtzYXNpcmVraGEzMTRAZ21haWwuY29tKzc3MDgwMDE4NzdRV


PTiA5OA==
In calculating dependency ratio, the numerator is:

a) <10 year and >60 years


b) <15 years and >60 years
c) <10 years and >65 years
d) <15 years and >65 years
Correct Answer: D
Your Answer: D
Explanation

Ans. D. <15 years and >65 years

Dependency ratio = (No of people aged 0-14 and those aged 65 and over/number of people
aged 15 to 64 years) X 100

QUESTION 100. MTQ3MStTQVNJUkVLSEEgUCtzYXNpcmVraGEzMTRAZ21haWwuY29tKzc3MDgwMDE4NzdR


PTiA5OQ==
Spindle cell is seen in :

a) Sarcoma
b) Lymphoma
c) Carcinoma

Score:
DBMCI CLT MAINS 2018 755.00 /
Page 64 1200
CLT MAINS - 2018
Exam Title :
(Final)
Email : sasirekha314@gmail.com
Contact : 7708001877

d) None
Correct Answer: A
Your Answer: A
Explanation

Ans. A. Sarcoma

· Two cell types can be seen microscopically in synovial sarcoma. One fibrous type, known as a
spindle or sarcomatous cell, is relatively small and uniform, and found in sheets. The other is
epithelial in appearance.

· Classical synovial sarcoma has a biphasic appearance with both types present. Synovial
sarcoma can also appear to be poorly differentiated or to be monophasic fibrous, consisting only
of sheets of spindle cells. Some authorities state that, extremely rarely, there can be a
monophasic epithelial form which causes difficulty in differential diagnosis.

· Like other soft tissue sarcomas, there is no universal grading system for reporting
histopathology results.

· In Europe, the Trojani or French system is gaining in popularity while the NCI grading system
is more common in the United States.

The Trojani system scores the sample, depending on tumour differentiation, mitotic index, and
tumour necrosis, between 0 and 6 and then converts this into a grade of between 1 and 3, with
1 representing a less aggressive tumour. The NCI system is also a three-grade one, but takes a
number of other factors into account.

QUESTION 101. MTQ3MStTQVNJUkVLSEEgUCtzYXNpcmVraGEzMTRAZ21haWwuY29tKzc3MDgwMDE4NzdR


PTiAxMDA=
A drug following first order kinetics was administered at a dose of 300 mg and the
half-life calculated was 3 hours. If the dose is increased by 600 mg. What is the half
life?

a) 3 hours
b) 6 hours
c) 9 hours
d) 18 hours
Correct Answer: A
Your Answer: A
Explanation

Ans. A. 3 hours

· For a drug following first order kinetics the half-life is constant.

· Hence even after increasing the dose in this case half- life continues to be 3 hours.

QUESTION 102. MTQ3MStTQVNJUkVLSEEgUCtzYXNpcmVraGEzMTRAZ21haWwuY29tKzc3MDgwMDE4NzdR


PTiAxMDE=

Score:
DBMCI CLT MAINS 2018 755.00 /
Page 65 1200
CLT MAINS - 2018
Exam Title :
(Final)
Email : sasirekha314@gmail.com
Contact : 7708001877

A 35-year-old female patient presented with a history of vulval pruritus and vaginal
discharge that started 10 days ago. Pap smear showed numerous acute inflammatory
cells along with planoconvex eggs 50-60 micron in size (Image). What is the likely
diagnosis ?

a) Candida
b) Trichuris
c) Trichomonas
d) Enterobius
Correct Answer: D
Your Answer: D
Explanation

Ans. D. Enterobius

· Seen in the Image are planoconvex, embryonated eggs of Enterobius vermicularis.

· Migrating female worms may enter vagina/urethra and cause vaginitis or urethritis.

· The female may lay eggs there, as in the case in the question above.

QUESTION 103. MTQ3MStTQVNJUkVLSEEgUCtzYXNpcmVraGEzMTRAZ21haWwuY29tKzc3MDgwMDE4NzdR


lPTiAxMDI=
The following is the threshold level of lead in blood which is associated with clinical
manifestations:

a) 50 mcg/dl
b) 70 mcg/dl
c) 90 mcg/dl
d) 110 mcg/dl
Correct Answer: B
Your Answer: D
Explanation

Ans. B. 70 mcg/dl

Score:
DBMCI CLT MAINS 2018 755.00 /
Page 66 1200
CLT MAINS - 2018
Exam Title :
(Final)
Email : sasirekha314@gmail.com
Contact : 7708001877

· Diagnosis- Coproporphyrin in urine (CPU) >150 mcg/L- useful screening test as well indicates
exposure

· Aminolevulinic acid ( ALA) in Urine>5 mg/L

· Quantitative Indicators- Lead in blood- > 70 mcg/dl- appearance of clinical symptoms, lead in
urine- >0.8 mg/l

· Treatment- EDTA- chelation therapy

QUESTION 104. MTQ3MStTQVNJUkVLSEEgUCtzYXNpcmVraGEzMTRAZ21haWwuY29tKzc3MDgwMDE4NzdR


PTiAxMDM=
The tissue of origin of the Kaposi sarcoma is:

a) Lymphoid
b) Neural
c) Vascular
d) Muscular
Correct Answer: C
Your Answer: C
Explanation

Ans. C. Vascular

Kaposi sarcoma (KS) is vascular neoplasm caused by human herpesvirus 8 (HHV8) that is highly
associated with acquired immunodeficiency syndrome (AIDS).

QUESTION 105. MTQ3MStTQVNJUkVLSEEgUCtzYXNpcmVraGEzMTRAZ21haWwuY29tKzc3MDgwMDE4NzdR


PTiAxMDQ=
The spores of which bacterium are used to check efficacy of hot air oven:

a) Bacillus stearothermophilus
b) Bacillus atropheus
c) Bacillus subtilis
d) Bacillus cereus
Correct Answer: C
Your Answer: C
Explanation

Ans. C. Bacillus subtilis

· Spores of Bacillus subtilis are used as biological control in hot air oven sterilization.

· Spores of Bacillus atropheus are used as biological control in ETO sterilization.

· Spores of Bacillus pumilus are used as biological control of gamma sterilization.

Score:
DBMCI CLT MAINS 2018 755.00 /
Page 67 1200
CLT MAINS - 2018
Exam Title :
(Final)
Email : sasirekha314@gmail.com
Contact : 7708001877

· Spores and crystalline insecticidal proteins produced by B. thuringiensis have been used to
control insect pests like mosquitoes.

QUESTION 106. MTQ3MStTQVNJUkVLSEEgUCtzYXNpcmVraGEzMTRAZ21haWwuY29tKzc3MDgwMDE4NzdR


PTiAxMDU=
New Drug is launched in the market at which phase of the clinical drug trial:

a) Phase V
b) Phase III
c) Phase II
d) Phase IV
Correct Answer: B
Your Answer: D
Explanation

Ans. B. Phase III

a. Pre clinical phase/animal experiment/lab/in vitro phase

b. Clinical trial phase-

Phase 0- Healthy human volunteer- 10-12- micro dosing and effect on body

Phase I- Healthy human volunteer- 20-100 – sub therapeutic dose- for dose ranging- safety and
non toxicity- max tolerable dose of the drug

Phase II- patients- 100-300- therapeutic dose- effectiveness of drug in the disease- efficacy and
safety in therapeutic doses- max drugs fail in phase II as efficacy not up to the expectation

Phase III- patients- 1000-2000- therapeutic doses- compare with known existing drug in
therapeutic doses- BABE study- after this u get FDA approval- new drug can be launched in
market

Phase IV- patients- any person taking drug in the general population- post marketing
surveillance- it is the longest phase- long term side effects

Phase V- translational research- data from all reported use- research on ongoing data collected-
no patients or volunteers

QUESTION 107. MTQ3MStTQVNJUkVLSEEgUCtzYXNpcmVraGEzMTRAZ21haWwuY29tKzc3MDgwMDE4NzdR


PTiAxMDY=
Which of the following drugs has no role in maintaining remission in Crohn’s disease?

a) 5 ASA agents
b) Steroids
c) Azathioprine
d) Infliximab
Correct Answer: B

Score:
DBMCI CLT MAINS 2018 755.00 /
Page 68 1200
CLT MAINS - 2018
Exam Title :
(Final)
Email : sasirekha314@gmail.com
Contact : 7708001877
Your Answer: A
Explanation

Ans. B. Steroids

Glucocorticoids play no role in maintenance therapy in either UC or CD

Steroids

· Steroids are used only for treatment of an acute attack of UC (DOC in moderate/severe cases)
and CD (DOC in mild case) but not for maintenance of remission as long-term therapy is
associated with deleterious side-effects.

· Oral budesonide is the initial steroid of choice as it is associated with lesser side-effects.

· Patients with no response to budesonide can be given intravenous (methyl prednisolone or


dexamethasone) or oral (prednisolone) steroids.

QUESTION 108. MTQ3MStTQVNJUkVLSEEgUCtzYXNpcmVraGEzMTRAZ21haWwuY29tKzc3MDgwMDE4NzdR


PTiAxMDc=
Mutation of Which gene is associated with malignant hyperthermia:

a) KCNJ2
b) RYR1
c) CLC1
d) CACN1S
Correct Answer: B
Your Answer: B
Explanation

Ans. B. RYR1

· RYR1—Mutations in the RYR1 gene disrupt the function of the ryanodine receptor, which
regulates calcium release from the sarcoplasmic reticulum.

· RYR1 mutations are linked to a congenital myopathy (central core disease) and to malignant
hyperthermia.

QUESTION 109. MTQ3MStTQVNJUkVLSEEgUCtzYXNpcmVraGEzMTRAZ21haWwuY29tKzc3MDgwMDE4NzdR


PTiAxMDg=
All are true regarding muscular changes after death, except:

Score:
DBMCI CLT MAINS 2018 755.00 /
Page 69 1200
CLT MAINS - 2018
Exam Title :
(Final)
Email : sasirekha314@gmail.com
Contact : 7708001877

a) Rigor mortis occurs 1-2 h after the death


b) Cadaveric spasm involves involuntary muscles
c) Rigor mortis involves voluntary muscles
d) Rigor mortis involves involuntary muscles
Correct Answer: B
Your Answer: B
Explanation

Ans. B. Cadaveric spasm involves involuntary muscles

· The group of voluntary muscles which were at strenuous work just before death goes into a
sudden state of stiffening, instead of passing to primary flaccidity after death.

Salient features of cadaveric spasm:

· Seen immediately after death

· No primary relaxation phase

· Involves only a group of voluntary muscles (which were in contraction before death)

· Exclusively antemortem in nature.

· Cadaveric spasm cannot be produced artificially.

· Great force is required to overcome the stiffness.

· Cadaveric spasm passes into rigor mortis without interruption and disappears when rigor
disappear.

· The attitude/last act of the person at the time of death is preserved. The cause and manner of
death can be determined.

· Mechanism: Unknown, Neurogenic

Conditions: Firearm in the hands of victims in suicidal gunshot injuries, plants and weeds in the
hands of victims in drowning, the weapon in the hands of victims in cut throat injuries
suggesting the manner of death.

Score:
DBMCI CLT MAINS 2018 755.00 /
Page 70 1200
CLT MAINS - 2018
Exam Title :
(Final)
Email : sasirekha314@gmail.com
Contact : 7708001877
QUESTION 110. MTQ3MStTQVNJUkVLSEEgUCtzYXNpcmVraGEzMTRAZ21haWwuY29tKzc3MDgwMDE4NzdR
PTiAxMDk=
MYPA agar is used for?

a) B. anthrax
b) B. cereus
c) Campylobacter
d) S. aureus
Correct Answer: B
Your Answer: B
Explanation

Ans. B. B. cereus

· Mannitol egg yolk phenol red polymyxin agar is the selective medium for Bacillus cereus .

· Polymyxin lysozyme EDTA thallium acetate is the selective medium for Bacillus anthracis .

· Skirrow’s medium and Butzler medium are selective media for Campylobacter .

· Ludlam’s medium and salt agar are selective media for S. aureus .

QUESTION 111. MTQ3MStTQVNJUkVLSEEgUCtzYXNpcmVraGEzMTRAZ21haWwuY29tKzc3MDgwMDE4NzdR


PTiAxMTA=
A 46-year-old AIDS patient presented with cough and intermittent fever. Chest X ray
showed the following Image and direct examination of BAL with fluorescent staining
showed halo around the yeast cell in negative staining. Identify the fungus:

a) Coccidioides immitis
b) Cryptococcus neoformans
c) Geotrichum candidum
d) Candida albicans
Correct Answer: B
Your Answer: B
Explanation

Score:
DBMCI CLT MAINS 2018 755.00 /
Page 71 1200
CLT MAINS - 2018
Exam Title :
(Final)
Email : sasirekha314@gmail.com
Contact : 7708001877

Ans. B. Cryptococcus neoformans

Halo around the yeast cell in negative staining. From this itself you can get the diagnosis - the
only capsulated fungus is Cryptococcus.

QUESTION 112. MTQ3MStTQVNJUkVLSEEgUCtzYXNpcmVraGEzMTRAZ21haWwuY29tKzc3MDgwMDE4NzdR


PTiAxMTE=
New RNTCP software online TB monitoring is?

a) Nikshay
b) Nishkay
c) e-DOTS
d) Nischay
Correct Answer: A
Your Answer: A
Explanation

Ans. A. Nikshay

· Kshayrog in hindi is Tuberculosis

· Nischay- UPT kit supply program from GOI free of cost

· e-DOTS and Nishkay- no such programs exists

· e-Nikshay was tried (app form) but scraped.

QUESTION 113. MTQ3MStTQVNJUkVLSEEgUCtzYXNpcmVraGEzMTRAZ21haWwuY29tKzc3MDgwMDE4NzdR


lPTiAxMTI=
Which of the following is a prodrug is:

a) Phenytoin
b) Carbamazepine
c) Primidone
d) Valproic acid
Correct Answer: C
Your Answer: C
Explanation

Ans. C. Primidone

· Primidone - A deoxy barbiturate converted by liver to phenobarbitone and phenylethyl


malonamide (PEMA).

· Its antiepileptic activity is mainly due to those active metabolites because t1/2 of primidone
(6-14 hr.) is less than that of its active metabolites.

· About 1/3 primidone is excreted unchanged by kidney.

Score:
DBMCI CLT MAINS 2018 755.00 /
Page 72 1200
CLT MAINS - 2018
Exam Title :
(Final)
Email : sasirekha314@gmail.com
Contact : 7708001877

QUESTION 114. MTQ3MStTQVNJUkVLSEEgUCtzYXNpcmVraGEzMTRAZ21haWwuY29tKzc3MDgwMDE4NzdR


PTiAxMTM=
Which of the following changes associated with shock may not revert back to normal?

a) Diffuse alveolar damage


b) Adrenal cortical lipid depletion
c) Acute tubular necrosis
d) Neuronal damage
Correct Answer: D
Your Answer: D
Explanation

Ans. D. Neuronal damage

Three Major Types of Shock

Type of Shock Clinical Example Principal Mechanisms

Myocardial infarction
Failure of myocardial pump
Ventricular rupture
resulting from intrinsic
Arrhythmia
Cardiogenic myocardial damage,
Cardiac tamponade extrinsic pressure, or
pulmonary obstruction to outflow

Fluid loss (e.g. hemorrhage,


Inadequate blood or plasma
Hypovolemic vomiting diarrhea, burns, or
volume
trauma)

Peripheral vasodilation and


Overwhelming microbial pooling of blood; endothelial
infections (bacterial and activation/injury;
fungal)
Septic leukocyte- induced damage,
Superantigens (Eg., toxic disseminated intravascular
shock syndrome) coagulation; activation of
cytokine cascades

QUESTION 115. MTQ3MStTQVNJUkVLSEEgUCtzYXNpcmVraGEzMTRAZ21haWwuY29tKzc3MDgwMDE4NzdR


PTiAxMTQ=
Chhaya (Centrochroman) contains:

a) Levonorgestrel
b) Ormeloxifene

Score:
DBMCI CLT MAINS 2018 755.00 /
Page 73 1200
CLT MAINS - 2018
Exam Title :
(Final)
Email : sasirekha314@gmail.com
Contact : 7708001877

c) Ethinyl estradiol
d) Progesterone
Correct Answer: B
Your Answer: B
Explanation

Ans. B. Ormeloxifene

· 30 mg two times a week for three months, then once a week

QUESTION 116. MTQ3MStTQVNJUkVLSEEgUCtzYXNpcmVraGEzMTRAZ21haWwuY29tKzc3MDgwMDE4NzdR


PTiAxMTU=
The chromosomal anomaly seen in Burkitt’s lymphoma is :

a) t (8 : 14)
b) t (14 : 18)
c) t (2 : 13)
d) t (1 : 13)
Correct Answer: A
Your Answer: A
Explanation

Ans. A. t (8 : 14)

All types of Burkitt's lymphoma are characterized by dysregulation of the c-myc gene by one of
three chromosomal translocations. This gene is found at 8q24.

· The most common variant is t(8;14)(q24;q32), which accounts for approximately 85% of cases.
This involves c-myc and IGH. A variant of this, a three-way translocation, t(8;14;18), has also
been identified.

· A rare variant is at t(2;8)(p12;q24).

· Another rare variant is t(8;22)(q24;q11).

· Combined, the two less-common translocations, t(2;8)(p12;q24) and t(8;22)(q24;q11), account


for the remaining 15% of cases not due to the t(8;14)(q24;q32) translocation

QUESTION 117. MTQ3MStTQVNJUkVLSEEgUCtzYXNpcmVraGEzMTRAZ21haWwuY29tKzc3MDgwMDE4NzdR


PTiAxMTY=
A 63-year old male with a h/o smoking for the past 20 years presents with an acute
onset of left-sided chest pain radiating down the left arm with profuse sweating. An
immediate ECG was performed in the emergency room which revealed a ST-segment
elevation in leads V4-V6. A decision was taken to immediately thrombolyze the patient.
For the purpose of pharmacological thrombolysis, 30 mg of streptokinase was infused
intravenously. The immediate plasma concentration of streptokinase was measured to
be 0.6 mg/dL. What is the volume of distribution of streptokinase in this patient?

a) 50 L

Score:
DBMCI CLT MAINS 2018 755.00 /
Page 74 1200
CLT MAINS - 2018
Exam Title :
(Final)
Email : sasirekha314@gmail.com
Contact : 7708001877

b) 5 L
c) 18 L
d) 180 L
Correct Answer: B
Your Answer: A
Explanation

Ans. B. 5 L

V d = Dose Administered / Plasma Concentration

= 30 mg/0.6 mg/dL

= 50 dL = 5L

QUESTION 118. MTQ3MStTQVNJUkVLSEEgUCtzYXNpcmVraGEzMTRAZ21haWwuY29tKzc3MDgwMDE4NzdR


lPTiAxMTc=
Diagnosis?

a) Systemic sclerosis diffuse type


b) Systemic sclerosis limited type
c) Antibodies to chromatin and histones
d) Antibodies to Sm antigen
Correct Answer: B
Your Answer: D
Explanation

Ans. B. Systemic sclerosis limited type

Score:
DBMCI CLT MAINS 2018 755.00 /
Page 75 1200
CLT MAINS - 2018
Exam Title :
(Final)
Email : sasirekha314@gmail.com
Contact : 7708001877

The microscopy shows anti-centromeric pattern of immunofluorescence seen in Systemic


sclerosis limited type.

QUESTION 119. MTQ3MStTQVNJUkVLSEEgUCtzYXNpcmVraGEzMTRAZ21haWwuY29tKzc3MDgwMDE4NzdR


PTiAxMTg=
Amount payable as enhanced sickness benefit under ESI scheme for persons
undergoing sterilization operation is:

a) 50 % of wages
b) 75% of wages
c) 90 % of wages
d) 100 % of wages
Correct Answer: D
Your Answer: D
Explanation

Ans. D. 100 % of wages

· Sickness benefit- 70% of avg daily wage. Extended sickness benefit- 70% of avg daily wage for
2 yrs (34 diseases)

· Enhanced sickness benefit- family welfare- vasectomy 7 days, tubectomy 14 days at 100%
wage.

· Maternity benefit ( leave with pay)- now till 26 weeks, abortion- 6 weeks, adoption- 12 weeks (
first time), work from home after 26 weeks allowed in special condition.

QUESTION 120. MTQ3MStTQVNJUkVLSEEgUCtzYXNpcmVraGEzMTRAZ21haWwuY29tKzc3MDgwMDE4NzdR


PTiAxMTk=
Which of the following eggs/ ova can be demonstrated in sputum and stool?

a) Pneumocystis
b) Paragonimus
c) Fasciola
d) Clonorchis
Correct Answer: B
Your Answer: B
Explanation

Ans. B. Paragonimus.

· Adults of Paragonimus reside in the cystic cavities in the lungs of the definitive host.

· Eggs are expelled out in sputum and if sputum is swallowed, also in feces.

· The eggs are oval, 90 x 50 μ long unembryonated and have a shouldered operculum.

Score:
DBMCI CLT MAINS 2018 755.00 /
Page 76 1200
CLT MAINS - 2018
Exam Title :
(Final)
Email : sasirekha314@gmail.com
Contact : 7708001877

QUESTION 121. MTQ3MStTQVNJUkVLSEEgUCtzYXNpcmVraGEzMTRAZ21haWwuY29tKzc3MDgwMDE4NzdR


lPTiAxMjA=
All of the following inhibit cell wall synthesis except:

a) Aminoglycosides
b) Cephalosporins
c) Bacitracin
d) Meropenem
Correct Answer: A
Your Answer: A
Explanation

Ans. A. Aminoglycosides

Aminoglycosides (Streptomycin, Gentamicin etc) causes misreading of mRNA code and affect
permeability.

QUESTION 122. MTQ3MStTQVNJUkVLSEEgUCtzYXNpcmVraGEzMTRAZ21haWwuY29tKzc3MDgwMDE4NzdR


lPTiAxMjE=
Major fibril protein in Primary Amyloidosis is:

a) AL
b) AA
c) Transthyretin
d) Procalcitonin
Correct Answer: A
Your Answer: Unanswered
Explanation

Ans. A. AL

Important Systemic amyloidosis

Score:
DBMCI CLT MAINS 2018 755.00 /
Page 77 1200
CLT MAINS - 2018
Exam Title :
(Final)
Email : sasirekha314@gmail.com
Contact : 7708001877
QUESTION 123. MTQ3MStTQVNJUkVLSEEgUCtzYXNpcmVraGEzMTRAZ21haWwuY29tKzc3MDgwMDE4NzdR
ElPTiAxMjI=
A patient is admitted with multiple bacterial infections and is found to have a
complete absence of C3. Which complement-mediated function would remain intact in
such a patient?

a) Lysis of bacteria
b) Opsonization of bacteria
c) Generation of anaphylatoxins
d) None of the above.
Correct Answer: D
Your Answer: A
Explanation

Ans. D. None of the above

All these functions are mediated by complement components that come after C3 and in its
absence cannot be activated.

QUESTION 124. MTQ3MStTQVNJUkVLSEEgUCtzYXNpcmVraGEzMTRAZ21haWwuY29tKzc3MDgwMDE4NzdR


PTiAxMjM=
Which of the following oncogene is associated with a high incidence of Medullary
carcinoma of thyroid?

a) P 53
b) Her 2 neu
c) RET protooncogene
d) Rb gene
Correct Answer: C
Your Answer: C
Explanation

Ans. C. RET protooncogene

· RET protooncogene is a growth factor receptor (receptor tyrosine kinase)

· The RET protein is a receptor for the glial cell lined derived neurotrophic factor and
structurally releated proteins that promote cell survival during neural development.

· RET is normally expressed in the following cells.

· Parafollicular C ells of the thyroid

· Adrenal medulla

· Parathyroid cell precursors.

QUESTION 125. MTQ3MStTQVNJUkVLSEEgUCtzYXNpcmVraGEzMTRAZ21haWwuY29tKzc3MDgwMDE4NzdR


lPTiAxMjQ=

Score:
DBMCI CLT MAINS 2018 755.00 /
Page 78 1200
CLT MAINS - 2018
Exam Title :
(Final)
Email : sasirekha314@gmail.com
Contact : 7708001877

Which of the following acts as a predominant arteriolar dilator?

a) Nitro-glycerine
b) Clevidipine
c) Enalaprilat
d) Nitroprusside
Correct Answer: B
Your Answer: B
Explanation

Ans. B. Clevidipine

· Predominant venodilators: Nitrates.

· Predominant arteriolar dilators: CCBs, KCOs and hydralazine.

· Rest are mixed vasodilators

QUESTION 126. MTQ3MStTQVNJUkVLSEEgUCtzYXNpcmVraGEzMTRAZ21haWwuY29tKzc3MDgwMDE4NzdR


lPTiAxMjU=
Identify the National health Program:

a) NVBDCP
b) NLEP
c) NHM
d) IDSP
Correct Answer: B
Your Answer: B
Explanation

Ans. B. NLEP (National Leprosy Eradication Program)

Score:
DBMCI CLT MAINS 2018 755.00 /
Page 79 1200
CLT MAINS - 2018
Exam Title :
(Final)
Email : sasirekha314@gmail.com
Contact : 7708001877

The lotus here represents beauty and purity. Affected thumb tip and normal fingers, along with
the shape of the house it makes symbolizes that affected person can become a useful member of
the society. The rising sun is for hope and optimism.

QUESTION 127. MTQ3MStTQVNJUkVLSEEgUCtzYXNpcmVraGEzMTRAZ21haWwuY29tKzc3MDgwMDE4NzdR


lPTiAxMjY=
Biochemical Oxygen demand is:

a) Amount of O2 absorbed by 1 Lit of sewage at 20 degree Celsius for 5 days


b) Amount of O2 absorbed by 1 Lit of sludge at 20 degree Celsius for 5 days
c) Amount of O2 absorbed by 1 Lit of sewage at 0 degree Celsius for 7 days
d) Amount of O2 absorbed by 1 Lit of sewage at 40 degree Celsius for 7 days
Correct Answer: A
Your Answer: D
Explanation

Ans. A. Amount of O2 absorbed by 1 Lit of sewage at 20 degree Celsius for 5 days

A low BOD is an indicator of good quality of water and a high BOD indicates polluted water
because the dissolved oxygen is consumed by bacteria when large amount of organic matter is
present in sewage.

QUESTION 128. MTQ3MStTQVNJUkVLSEEgUCtzYXNpcmVraGEzMTRAZ21haWwuY29tKzc3MDgwMDE4NzdR


ElPTiAxMjc=
With idiopathic Parkinson disease, Lewy bodies are typically found in the:

a) Substantia nigra
b) Nucleus basalis of Meynert
c) Dorsal raphe
d) All of the above
Correct Answer: D
Your Answer: A
Explanation

Ans. D. All of the above.

· The clinical presentation of Lewy body disease varies according to the site of Lewy body
formation and associated neuronal loss. In Parkinson disease, the Lewy bodies are found in the
substantia nigra of the midbrain, coupled with the loss of pigmented neurons. In persons with
the dementia of diffuse Lewy body disease, there are Lewy bodies in the neocortex. Some
persons have the Lewy bodies in both locations.

· The basal ganglia and diencephalon may also be involved in some cases.

· Lewy bodies are spherical, intraneuronal, cytoplasmic, eosinophilic inclusions comprising


abnormally truncated and phosphorylated intermediate neurofilament proteins, alpha-synuclein,
ubiquitin, and associated enzymes.

Score:
DBMCI CLT MAINS 2018 755.00 /
Page 80 1200
CLT MAINS - 2018
Exam Title :
(Final)
Email : sasirekha314@gmail.com
Contact : 7708001877

· With idiopathic Parkinson disease, Lewy bodies are typically found in the substantia nigra,
nucleus basalis of Meynert, dorsal raphe, locus ceruleus, dorsal motor nucleus of the vagus
nerve, and hypothalamus. In cases Lewy body dementia, cortical Lewy bodies are prominent,
but there are typically findings of Alzheimer disease as well.

QUESTION 129. MTQ3MStTQVNJUkVLSEEgUCtzYXNpcmVraGEzMTRAZ21haWwuY29tKzc3MDgwMDE4NzdR


lPTiAxMjg=
Which of the following antirheumatic drugs can be administered intravenously?

a) Adalimumab
b) Etanercept
c) Infliximab
d) Anakinra
Correct Answer: C
Your Answer: C
Explanation

Ans. C. Infliximab

Drug Dose

1 Adalimumab 40 mg s.c. every 2 weeks

2 Etanercept 25 – 50 mg s.c. once or twice weekly

3 Infliximab 3-5 mg/kg IV every 4-8 weeks

4 Anakinra 100 mg s.c. daily

QUESTION 130. MTQ3MStTQVNJUkVLSEEgUCtzYXNpcmVraGEzMTRAZ21haWwuY29tKzc3MDgwMDE4NzdR


lPTiAxMjk=
Study the given Image. Given Gun is designed for use up to:

Score:
DBMCI CLT MAINS 2018 755.00 /
Page 81 1200
CLT MAINS - 2018
Exam Title :
(Final)
Email : sasirekha314@gmail.com
Contact : 7708001877

a) 30 – 50 m
b) 50 – 80 m
c) 80 – 110 m
d) 110 – 130 m
Correct Answer: A
Your Answer: Unanswered
Explanation

Ans. A. 30 – 50 m

SHOTGUNS

· The smooth-bore weapon consists of a metal barrel which has parallel sides “double barreled”.
- They usually fire a large number of small spherical lead shot.

· A shot gun is designed for use up to 30 – 50 m. - Common sizes 19mm, 11mm. –

· Ammunition consists of cartridge (metal base containing a central firing cap supporting plastic
or cardboard tube).

· Inside the cartridge there is a wad of paper & cardboard, with a mass of lead pellets.

QUESTION 131. MTQ3MStTQVNJUkVLSEEgUCtzYXNpcmVraGEzMTRAZ21haWwuY29tKzc3MDgwMDE4NzdR


lPTiAxMzA=
All are the criteria for food fortification except:

a) Deficiency of nutrient
b) Vehicle food must be consumed by the target group
c) Affordable
d) Taste should be good so that target group consumes
Correct Answer: D
Your Answer: A
Explanation

Ans. D. Taste should be good so that target group consumes

Taste is not a criteria, but the nutrient should not alter the taste of the vehicle food

QUESTION 132. MTQ3MStTQVNJUkVLSEEgUCtzYXNpcmVraGEzMTRAZ21haWwuY29tKzc3MDgwMDE4NzdR


lPTiAxMzE=
5-Lipo-oxygenase synthesis inhibitor is:

a) Zileuton
b) Zafirlukast
c) Montelukast
d) Omalizumab
Correct Answer: A
Your Answer: A

Score:
DBMCI CLT MAINS 2018 755.00 /
Page 82 1200
CLT MAINS - 2018
Exam Title :
(Final)
Email : sasirekha314@gmail.com
Contact : 7708001877
Explanation

Ans. A. Zileuton

· Zileuton- is a 5 LOX inhibitor blocks LTC4/D4 as well as LT B4 synthesis, clinical efficacy in


asthma similar to montelukast.

· The duration of action is short with hepatotoxic potentials. These limitations have restricted to
use.

QUESTION 133. MTQ3MStTQVNJUkVLSEEgUCtzYXNpcmVraGEzMTRAZ21haWwuY29tKzc3MDgwMDE4NzdR


lPTiAxMzI=
Biopsy specimen from a diabetic patient showed the following feature: Identify:

a) Aspergillus
b) Mucor
c) Rhizomucor
d) Penicillium
Correct Answer: B
Your Answer: B
Explanation

Ans. B. Mucor

Score:
DBMCI CLT MAINS 2018 755.00 /
Page 83 1200
CLT MAINS - 2018
Exam Title :
(Final)
Email : sasirekha314@gmail.com
Contact : 7708001877

In the given image you are not seeing any rhizoids that rules out rhizo mucor; Aseptate hyphae
rules out Penicillium and Aspergillus. So the answer is MUCOR.

QUESTION 134. MTQ3MStTQVNJUkVLSEEgUCtzYXNpcmVraGEzMTRAZ21haWwuY29tKzc3MDgwMDE4NzdR


PTiAxMzM=
Which of the following is example of instability of chromosome:

a) Ataxia telangiectasia
b) Downs syndrome
c) Kliffel feil syndrome
d) None of above
Correct Answer: A
Your Answer: A
Explanation

Ans. A. Ataxia telangiectasia

· Ataxia-telangiectasia (AT) is an autosomal recessive genetic disorder characterized by


cerebellar ataxia, oculocutaneous telangiectasia, and immunodeficiency.

· The mutant ATM gene has sequence similarity to the phosphatidyl-inositol-3 kinases that are
involved in signal transduction.

· The ATM gene belongs to a conserved family of genes that monitor DNA repair and coordinate
DNA synthesis with cell division.

· The deleterious effects of the ATM gene are widespread. Truncal ataxia may become evident
when walking begins and is progressive.

· Telangiectasia, primarily represented by dilated blood vessels in the ocular sclera, in a


butterfly y area of the face, and on the ears, is an early diagnostic feature. Immunodeficiency
may be clinically manifest by recurrent and all patients have overt immunodeficiency.

· Ovarian agenesis is a frequent occurrence.

· Persistence of very high serum levels of oncofetal proteins, including fetoprotein and
carcinoembryonic antigen, may be of diagnostic value.

· Frequent causes of death are chronic pulmonary disease and malignancy.

· Lymphomas are most common, although carcinomas also occur.

· The immunologic abnormalities seem to be related to maldevelopment of the thymus.

· The markedly hypoplastic thymus is similar in appearance to an embryonic thymus.

· The peripheral T cell pool is reduced in size, especially in lymphoid tissue compartments.
Cutaneous anergy and delayed rejection of skin allografts are common.

· B lymphocyte development is normal, most patients are deficient in serum IgE and IgA, and a
smaller number have reduced serum levels of IgG, particularly of the IgG2, IgG4 subclasses.

Score:
DBMCI CLT MAINS 2018 755.00 /
Page 84 1200
CLT MAINS - 2018
Exam Title :
(Final)
Email : sasirekha314@gmail.com
Contact : 7708001877

· The defect in DNA repair mechanisms in AT patients renders their cells highly susceptible to
radiation-induced chromosomal damage and resultant tumor development.

QUESTION 135. MTQ3MStTQVNJUkVLSEEgUCtzYXNpcmVraGEzMTRAZ21haWwuY29tKzc3MDgwMDE4NzdR


PTiAxMzQ=
All of the following are systems of PEM classification except:

a) Welcome’s classification
b) Gomez classification
c) IAP classification
d) Quetelet’s index
Correct Answer: D
Your Answer: D
Explanation

Ans. D. Quetelet’s index

This is BMI all others are systems of PEM classification.

QUESTION 136. MTQ3MStTQVNJUkVLSEEgUCtzYXNpcmVraGEzMTRAZ21haWwuY29tKzc3MDgwMDE4NzdR


PTiAxMzU=
Legally “Abortion” is a termination of pregnancy:

a) Before 6 weeks
b) Before 16 weeks
c) Before 26 weeks
d) Before full term
Correct Answer: D
Your Answer: Unanswered
Explanation

Ans. D. Before full term

Legally, abortion (miscarriage) is the premature expulsion of the fetus from the mother’s womb
at any time of pregnancy before the term of pregnancy is completed.

QUESTION 137. MTQ3MStTQVNJUkVLSEEgUCtzYXNpcmVraGEzMTRAZ21haWwuY29tKzc3MDgwMDE4NzdR


lPTiAxMzY=
Identify the logo:

Score:
DBMCI CLT MAINS 2018 755.00 /
Page 85 1200
CLT MAINS - 2018
Exam Title :
(Final)
Email : sasirekha314@gmail.com
Contact : 7708001877

a) National malaria eradication program


b) National vector borne diseases control program
c) Integrated vector control
d) Zika surveillance project
Correct Answer: B
Your Answer: B
Explanation

Ans. B. National vector borne diseases control program

· The National Vector Borne Disease Control Programme (NVBDCP) is a comprehensive


programme for prevention and control of vector borne diseases namely Malaria, Filaria, Kala-
azar, Japanese Encephalitis (JE), Dengue and Chikungunya which is covered under the overall
umbrella of NRHM.

· Directorate of National Vector Borne Disease Control Programme (NVBDCP) is the central
nodal agency for the prevention and control of vector borne diseases.

QUESTION 138. MTQ3MStTQVNJUkVLSEEgUCtzYXNpcmVraGEzMTRAZ21haWwuY29tKzc3MDgwMDE4NzdR


lPTiAxMzc=
Mechanism of action of rifampicin is by :-

a) Inhibition of RNA dependent DNA polymerase


b) Inhibition of DNA dependent RNA polymerases
c) Interferes with translocation.
d) Inhibitits synthesis of mycolic acid, that form part of mycobacterial cell wall.
Correct Answer: B
Your Answer: B
Explanation

Ans. B. Inhibition of DNA dependent RNA polymerases

· Rifampin inhibits DNA dependent RNA polymerase of mycobacteria & other microorganisms
by forming a stable drug-enzyme complex leading to suppression of initiation of chain formation
(but not chain elongation) in RNA synthesis.

· Inhibition of synthesis of mycolic acid is mechanism of isoniazid.

Score:
DBMCI CLT MAINS 2018 755.00 /
Page 86 1200
CLT MAINS - 2018
Exam Title :
(Final)
Email : sasirekha314@gmail.com
Contact : 7708001877

QUESTION 139. MTQ3MStTQVNJUkVLSEEgUCtzYXNpcmVraGEzMTRAZ21haWwuY29tKzc3MDgwMDE4NzdR


lPTiAxMzg=
The Electron Microscopy is virtually diagnostic in renal biopsy study of:

a) Goodpasture’s syndrome
b) Churg-Strauss Syndrome
c) Alport syndrome
d) Wegner’s granulomatosis
Correct Answer: C
Your Answer: C
Explanation

Ans. C. Alport syndrome

· Alport’s syndrome presents with characteristic histological features on electron microscopy.

· Such changes may present in other diseases, but are most pronounced and widespread in
Alport’s syndrome.

Features of Alport’s syndrome on electron microscopy :

· Alport’s patients early in their disease typically have diffuse thinning of the basement
membrane which thickens over time into multilamellous surrounding lucent areas that often
contain granules of varying density.

· The characteristic electron electron microscopic finding of fully developed disease is that
glomerular basement membrane shows irregular foci of thickening alternating with attenuation
(thinning) with pronounced splitting and lamination of lamina densa often with distinctive
basket weave appearance.

QUESTION 140. MTQ3MStTQVNJUkVLSEEgUCtzYXNpcmVraGEzMTRAZ21haWwuY29tKzc3MDgwMDE4NzdR


lPTiAxMzk=
What is the highest permissible level of Fluoride in water according to WHO criteria:

a) 1.5 mg/Lit
b) 1 mg/Lit
c) 0.5 mg/Lit
d) 2 mg/Lit
Correct Answer: A
Your Answer: B
Explanation

Ans. A. 1.5 mg/Lit

1 mg/Lit in Indian standards, 0.5 mg/Lit under review.

Score:
DBMCI CLT MAINS 2018 755.00 /
Page 87 1200
CLT MAINS - 2018
Exam Title :
(Final)
Email : sasirekha314@gmail.com
Contact : 7708001877
QUESTION 141. MTQ3MStTQVNJUkVLSEEgUCtzYXNpcmVraGEzMTRAZ21haWwuY29tKzc3MDgwMDE4NzdR
PTiAxNDA=
Which of the following anti-microbials is associated with prolongation of the QT
interval?

a) Co-amoxiclav
b) Gentamicin
c) Cefuroxime
d) Erythromycin
Correct Answer: D
Your Answer: D
Explanation

Ans. D. Erythromycin

· The macrolides are associated with a prolongation of the QT interval.

· Other antimicrobials associated with prolonged QT include quinine, levofloxacin.

QUESTION 142. MTQ3MStTQVNJUkVLSEEgUCtzYXNpcmVraGEzMTRAZ21haWwuY29tKzc3MDgwMDE4NzdR


PTiAxNDE=
The below Image of contraceptive is a type of:

a) Permanent Male sterilization


b) II generation Copper T
c) III generation Copper T
d) Permanent female sterilization
Correct Answer: D
Your Answer: D
Explanation

Ans. D. Permanent female sterilization

Essure – Permanent Birth Control

· Permanent sterilization method- micro inserts are placed in Fallopian tubes- once activated
tissue growth and scarring- in 3 months occlusion- no anesthesia, no incision, no admission
required. 99.8 % effective, 36 min procedure time

· Disadvantage- no protection from STI, ectopic and minor risk of perforation

Score:
DBMCI CLT MAINS 2018 755.00 /
Page 88 1200
CLT MAINS - 2018
Exam Title :
(Final)
Email : sasirekha314@gmail.com
Contact : 7708001877

QUESTION 143. MTQ3MStTQVNJUkVLSEEgUCtzYXNpcmVraGEzMTRAZ21haWwuY29tKzc3MDgwMDE4NzdR


lPTiAxNDI=
Wermer's syndrome is considered a?

a) MEN type I
b) MEN type II
c) MEN type III
d) MEN type IV
Correct Answer: A
Your Answer: A
Explanation

Ans. A. MEN type I

· Multiple endocrine neoplasia type 1 (MEN-1 syndrome) or Wermer syndrome is part of a group
of disorders that affect the endocrine system.

· These disorders greatly increase the risk of developing multiple cancerous and noncancerous
tumors in glands such as the parathyroid, pituitary, and pancreas. Multiple endocrine neoplasia
occurs when tumors are found in at least two endocrine glands.

· Tumors can also develop in organs and tissues other than endocrine glands. If the tumors
become cancerous, some cases can be life threatening. The disorder affects 1 in 30,000 people.

QUESTION 144. MTQ3MStTQVNJUkVLSEEgUCtzYXNpcmVraGEzMTRAZ21haWwuY29tKzc3MDgwMDE4NzdR


PTiAxNDM=
Which of the following is a stimulant laxative?

a) Lactulose
b) Calcium polycarbophil
c) Bisacodyl
d) Milk of Magnesia
Correct Answer: C
Your Answer: D
Explanation

Ans. C. Bisacodyl

Drugs for constipation

Score:
DBMCI CLT MAINS 2018 755.00 /
Page 89 1200
CLT MAINS - 2018
Exam Title :
(Final)
Email : sasirekha314@gmail.com
Contact : 7708001877

· Bran

· Psyllium

1 Bulk forming laxatives · Methylcellulose

· Calcium polycarbophil

· Isopghol

· Docusate

2 Surfactant laxatives · Poloxamer

· Lactulose

· Phenolphthalein

· Bisacodyl
3 Stimulant laxatives
· Sodium picosulfate

· Oxyphenisatin

· Magnesium sulfate

· Magnesium citrate

· Magnesium hydroxide

· Milk of magnesia
4 Osmotic laxatives
· Sodium sulfate

· Sodium phosphate

· Sodium potassium tartarated

· Castor oil

· Senna
5 Anthraquinone laxatives
· Cascara sagrada

QUESTION 145. MTQ3MStTQVNJUkVLSEEgUCtzYXNpcmVraGEzMTRAZ21haWwuY29tKzc3MDgwMDE4NzdR


PTiAxNDQ=
A 33 year old HIV+ homosexual man from Thailand presented with a four month
history of a generalized papular rash, associated with diarrhea and weight loss. A

Score:
DBMCI CLT MAINS 2018 755.00 /
Page 90 1200
CLT MAINS - 2018
Exam Title :
(Final)
Email : sasirekha314@gmail.com
Contact : 7708001877
biopsy showed numerous small yeast cells and the culture shown below was isolated.
What is the causative organism?

a) Cryptococcus neoformans
b) Penicillium marneffi
c) Histoplasma capsulatum
d) Trichophyton rubrum
Correct Answer: B
Your Answer: C
Explanation

Ans. B. Penicillium marneffi

Penicillium marneffi (New nomenclature: Talaromyces marneffi) is unique dimorphic fungus


which is an emerging opportunistic fungal disease endemic in southeast Asia.

· Since it is a dimorphic fungi – u can see yeast cells

· The clinical feature includes umbilicated papular, molluscum contagiosum-like skin lesions, in
face, upper trunk, pinnae and arms and non specific febrile illness in patient at risk. (M/c in
HIV)

· The fungus produces brick red coloured pigment in SDA.

Score:
DBMCI CLT MAINS 2018 755.00 /
Page 91 1200
CLT MAINS - 2018
Exam Title :
(Final)
Email : sasirekha314@gmail.com
Contact : 7708001877

QUESTION 146. MTQ3MStTQVNJUkVLSEEgUCtzYXNpcmVraGEzMTRAZ21haWwuY29tKzc3MDgwMDE4NzdR


PTiAxNDU=
Which check point is most important in cell cycle that causes dysregulated growth and
mutant phenotype:

a) G1/S
b) G2/M
c) G1/ G2
d) S/ G2
Correct Answer: A
Your Answer: A
Explanation

Ans. A. G1/S

Defects in the G1/S checkpoint are more important in cancer, in that these lead to dysregulated
growth as well as a mutator phenotype.

QUESTION 147. MTQ3MStTQVNJUkVLSEEgUCtzYXNpcmVraGEzMTRAZ21haWwuY29tKzc3MDgwMDE4NzdR


PTiAxNDY=
Zolpidem has a half-life of:

a) 2–3 mins
b) 2–3 hours
c) 2–3 days
d) 12–18 hours
Correct Answer: B
Your Answer: Unanswered
Explanation

Ans. B. 2–3 hours

· Zolpidem is a prescription medication used for the short-term treatment of insomnia, as well
as some brain disorders.

· It is a short-acting non benzodiazepine hypnotic that potentiates gamma-aminobutyric acid


(GABA), an inhibitory neurotransmitter, by binding to GABAA receptors at the same location as
benzodiazepines.

· It works quickly (usually within 15 minutes) and has a short half-life (2–3 hours).

QUESTION 148. MTQ3MStTQVNJUkVLSEEgUCtzYXNpcmVraGEzMTRAZ21haWwuY29tKzc3MDgwMDE4NzdR


lPTiAxNDc=
The toxicity associated with ingestion of magnets is due to:

Score:
DBMCI CLT MAINS 2018 755.00 /
Page 92 1200
CLT MAINS - 2018
Exam Title :
(Final)
Email : sasirekha314@gmail.com
Contact : 7708001877

a) Cobalt
b) Cadmium
c) Barium
d) Antimony
Correct Answer: A
Your Answer: B
Explanation

Ans. A. Cobalt

Ingestion of magnets can cause acute cobalt toxicity.

QUESTION 149. MTQ3MStTQVNJUkVLSEEgUCtzYXNpcmVraGEzMTRAZ21haWwuY29tKzc3MDgwMDE4NzdR


lPTiAxNDg=
Installing Condom vending machines in high prevalence area is an example of:

a) Appropriate technology
b) Social Marketing
c) Community participation
d) Intersectoral involvement
Correct Answer: B
Your Answer: B
Explanation

Ans. B. Social Marketing

· Social marketing is the use of commercial marketing principles and techniques to improve the
welfare of people and the physical, social and economic environment in which they live.

Score:
DBMCI CLT MAINS 2018 755.00 /
Page 93 1200
CLT MAINS - 2018
Exam Title :
(Final)
Email : sasirekha314@gmail.com
Contact : 7708001877

· It involves planning, and long term approach to change the human behavior.

QUESTION 150. MTQ3MStTQVNJUkVLSEEgUCtzYXNpcmVraGEzMTRAZ21haWwuY29tKzc3MDgwMDE4NzdR


lPTiAxNDk=
Diagnosis?

a) Follicular carcinoma
b) Papillary carcinoma
c) Anaplastic carcinoma
d) Medullary carcinoma
Correct Answer: C
Your Answer: A
Explanation

Ans. C. Anaplastic carcinoma

Microscopically the parent tissue is not visible but it shows marked nuclear polymorphism
suggestive of anaplastic carcinoma.

QUESTION 151. MTQ3MStTQVNJUkVLSEEgUCtzYXNpcmVraGEzMTRAZ21haWwuY29tKzc3MDgwMDE4NzdR


lPTiAxNTA=
Part C: Questions 151 - 300

Name the angle drawn in the Image:

Score:
DBMCI CLT MAINS 2018 755.00 /
Page 94 1200
CLT MAINS - 2018
Exam Title :
(Final)
Email : sasirekha314@gmail.com
Contact : 7708001877

a) Gissane’s angle
b) Neutral angle
c) Bohler’s angle
d) Kite’s angle
Correct Answer: C
Your Answer: C
Explanation

Ans. C. Bohler’s angle

· Bohler’s angle is drawn to confirm the presence of a calcaneal fracture.

· It is formed by lines joining top of posterior facet to top of calcaneal tuberosity and another
line joining top of anterior facet to top of superior facet in the lateral view of calcaneum.

· An angle less than 25 degree signifies depression of the articular surface.

QUESTION 152. MTQ3MStTQVNJUkVLSEEgUCtzYXNpcmVraGEzMTRAZ21haWwuY29tKzc3MDgwMDE4NzdR


lPTiAxNTE=
Videostroboscopy is used for all of the following except:

a) Papilloma
b) Nodule
c) Ulcer
d) Neurological disorder
Correct Answer: A
Your Answer: Unanswered
Explanation

Ans. A. Papilloma

Stroboscopy is a special method used to visualize vocal fold vibration. It uses a synchronized ,
flashing light passed through a flexible or rigid telescope.

Score:
DBMCI CLT MAINS 2018 755.00 /
Page 95 1200
CLT MAINS - 2018
Exam Title :
(Final)
Email : sasirekha314@gmail.com
Contact : 7708001877

The flashes of light from the stroboscope are synchronized to the vocal fold vibration at a
slightly slower speed, allowing the examiner to observe vocal fold vibration during sound
production in what appears to be slow motion. This slow motion picture is an illusion, as the
speed of actual vocal fold vibration is not changed by stroboscopy. Because vocal fold vibration
is so fast, the slow motion view is actually derived from many successive vibration cycles. This
special viewing helps to evaluate each vocal fold’s vibration properties during the different
phases of the vocal fold’s vibration cycle.

Video laryngoscopy with stroboscopy is the essential diagnostic procedure for the evaluation of
laryngeal mucosa, vocal fold motion biomechanics, and mucosal vibration. Nodule and ulcer
both will affect the vibration of vocal fold and can be diagnosed by videostroboscopy. It is also
used in certain neurological disorders that affect vocal fold vibration.

However it can also be used in papilloma disease but mainly in between the surgeries to look
for post operative scarring (remember papilloma requires multiple surgeries). However
papilloma doesn’t usually require videostroboscopy for its diagnosis

QUESTION 153. MTQ3MStTQVNJUkVLSEEgUCtzYXNpcmVraGEzMTRAZ21haWwuY29tKzc3MDgwMDE4NzdR


lPTiAxNTI=
Which type of cells predominantly seen in Rheumatoid Arthritis?

a) T Cells
b) B Cells
c) Dendritic cells
d) Giant cells
Correct Answer: A
Your Answer: A
Explanation

Ans. A. T Cells

Rheumatoid arthritis ( RA ) is one of the most common auto immune chronic inflammatory
syndromes. Evidence supports a role for T-helper (Th) cells , Th17 cells , and impaired CD4
+CD25(hi) regulatory T cell (Regulatory) function in the pathogenesis of RA.

QUESTION 154. MTQ3MStTQVNJUkVLSEEgUCtzYXNpcmVraGEzMTRAZ21haWwuY29tKzc3MDgwMDE4NzdR


PTiAxNTM=
A 33-year-old woman presents for psychiatric evaluation after making suicidal threats.
She reports a long-standing pattern of self-mutilation, impulsiveness, and no
relationship lasting longer than 1 month. Which of the following is the most likely
diagnosis?

a) Histrionic personality disorder


b) Antisocial personality disorder
c) Narcissistic personality disorder
d) Borderline personality disorder
Correct Answer: D

Score:
DBMCI CLT MAINS 2018 755.00 /
Page 96 1200
CLT MAINS - 2018
Exam Title :
(Final)
Email : sasirekha314@gmail.com
Contact : 7708001877
Your Answer: D
Explanation

Ans. D. Borderline personality disorder

Borderline personality disorder is characterized by a long-standing pattern of impulsiveness,


recklessness, unstable relationships, and suicidal and self-mutilating behaviors. It may also be
associated with kleptomania.

QUESTION 155. MTQ3MStTQVNJUkVLSEEgUCtzYXNpcmVraGEzMTRAZ21haWwuY29tKzc3MDgwMDE4NzdR


PTiAxNTQ=
A female aged 20 presented with vaginal discharge which is non irritating and
malodorous. Saline wet preparation reveals clue cells. Which type of vaginitis Clue
cells are used to detect?

a) Bacterial vaginosis
b) Candidal vaginitis
c) Primary syphilis
d) Secondary syphilis
Correct Answer: A
Your Answer: A
Explanation

Ans. A. Bacterial vaginosis

Clue cells are considered to be the most reliable indicators of BV.

• The positive predictive value is of around 95% of this test for the presence of BV.
• These vaginal epithelial cells contain many attached bacteria and hence create a poorly
defined border of stippled cells.

QUESTION 156. MTQ3MStTQVNJUkVLSEEgUCtzYXNpcmVraGEzMTRAZ21haWwuY29tKzc3MDgwMDE4NzdR


PTiAxNTU=
Which of the following statement is characteristic of Cri-du-Chat Syndrome:

a) Hypotonia
b) Tall stature
c) Hypotelorism
d) Encephalocele
Correct Answer: A
Your Answer: Unanswered
Explanation

Ans. A. Hypotonia

Cri-du-Chat Syndrome is due to a 5p- deletion. The main features are:

• Hypotonia

Score:
DBMCI CLT MAINS 2018 755.00 /
Page 97 1200
CLT MAINS - 2018
Exam Title :
(Final)
Email : sasirekha314@gmail.com
Contact : 7708001877

• Short stature

• Characteristic cry because of laryngeal abnormalities

• Microcephaly with protruding metopic suture

• Moon-like face

• Hypertelorism

• Bilateral epicanthic folds

• High-arched palate

• Wide and flat nasal bridge

• Mental retardation.

QUESTION 157. MTQ3MStTQVNJUkVLSEEgUCtzYXNpcmVraGEzMTRAZ21haWwuY29tKzc3MDgwMDE4NzdR


lPTiAxNTY=
An 80 years old man with past history of AMI is found to have LBBB on ECG. He is
asymptomatic; on cardiac auscultation the most likely finding is?

a) S3
b) S4
c) Opening snap
d) Reversed split S2
Correct Answer: D
Your Answer: D
Explanation

Ans. D. Reversed split S2

Second heart sound (S2) is composed of aortic closure followed by pulmonic closure. Because,
inspiration increases blood return to the right side of heart, pulmonic closure is delayed which
results in normal splitting of S2 during inspiration. Paradoxical or reverse splitting of S2 refers
to narrowed splitting of S2 during inspiration as compared with expiration because pulmonic
closure precedes aortic closure. It can occur in the following - LBBB, Aortic stenosis, HOCM,
severe hypertension, and RV pacemaker.

QUESTION 158. MTQ3MStTQVNJUkVLSEEgUCtzYXNpcmVraGEzMTRAZ21haWwuY29tKzc3MDgwMDE4NzdR


lPTiAxNTc=
Identify this syndrome:

Score:
DBMCI CLT MAINS 2018 755.00 /
Page 98 1200
CLT MAINS - 2018
Exam Title :
(Final)
Email : sasirekha314@gmail.com
Contact : 7708001877

a) Goldenhar syndrome
b) Crouzon syndrome
c) Russell Silver syndrome
d) Treacher Collins syndrome
Correct Answer: D
Your Answer: D
Explanation

Ans. D. Treacher Collins syndrome

Prognosis of Treacher Collins syndrome

• The child usually will show normal intelligence

• Hearing loss is treated to ensure better performance in school.

• Plastic surgery may be required for facial defects

• Children usually grow to become functioning adults of normal intelligence

QUESTION 159. MTQ3MStTQVNJUkVLSEEgUCtzYXNpcmVraGEzMTRAZ21haWwuY29tKzc3MDgwMDE4NzdR


lPTiAxNTg=
Flame-shaped haemorrhages occurs at the level of:

a) Outer nuclear layer


b) Outer plexiform layer

Score:
DBMCI CLT MAINS 2018 755.00 /
Page 99 1200
CLT MAINS - 2018
Exam Title :
(Final)
Email : sasirekha314@gmail.com
Contact : 7708001877

c) Inner plexiform layer


d) Nerve fiber layer
Correct Answer: D
Your Answer: D
Explanation

Ans. D. Nerve fiber layer

Flame-shaped haemorrhages are superficial; while the dot and blot haemorrhages are present
in the deeper layers of retina.

QUESTION 160. MTQ3MStTQVNJUkVLSEEgUCtzYXNpcmVraGEzMTRAZ21haWwuY29tKzc3MDgwMDE4NzdR


lPTiAxNTk=
Uhthoff’s sign is seen in:

a) Diabetic retinopathy
b) Demyelinating optic neuritis
c) Craniopharyngioma
d) Diabetes insipidus
Correct Answer: B
Your Answer: B
Explanation

Ans. B. Demyelinating optic neuritis

Uhthoff’s phenomenon is seen in demyelinating disease in which there is an impairment of


vision with increased body temperature.

QUESTION 161. MTQ3MStTQVNJUkVLSEEgUCtzYXNpcmVraGEzMTRAZ21haWwuY29tKzc3MDgwMDE4NzdR


ElPTiAxNjA=
Miller Fischer test is used for :-

a) Evaluating cognition in NPH.


b) Severity of paralysis in GBS.
c) Grading muscle involvement in ALS.
d) Measuring disability in Multiple sclerosis.
Correct Answer: A
Your Answer: C
Explanation

Ans. A. Evaluating cognition in NPH.

Evaluating cognition in NPH. The CSF tap test, sometimes lumbar tap test or Miller Fisher Test,
is a medical test that is used to decide whether shunting of cerebrospinal fluid (CSF) would be
helpful in a patient with suspected normal pressure hydrocephalus (NPH).

Score:
DBMCI CLT MAINS 2018 755.00 /
Page 100 1200
CLT MAINS - 2018
Exam Title :
(Final)
Email : sasirekha314@gmail.com
Contact : 7708001877

The test involves removing 30 mL of cerebrospinal fluid (CSF) through a lumbar puncture, after
which cognitive function is clinically reassessed. Clinical improvement showed a high predictive
value for subsequent success with shunting.

QUESTION 162. MTQ3MStTQVNJUkVLSEEgUCtzYXNpcmVraGEzMTRAZ21haWwuY29tKzc3MDgwMDE4NzdR


ElPTiAxNjE=
Which one of the following imaging modalities is most sensitive for localization of
extra adrenal pheochromocytoma?

a) USG
b) DOPA -PET
c) MRI
d) MIBG scan
Correct Answer: B
Your Answer: B
Explanation

Ans. B. DOPA - PET

Pheochromocytoma

· MRI has been the primary method of diagnosis for adrenal pheochromocytoma which shows
light bulb appearance on a T2 weighted MRI. MIBG scanning can play a complementary role in
evaluating intra adrenal pheochromocytoma.

· It is 80-90% sensitive for adrenal one. MIBG scintigraphy is extremely valuable in imaging of
extra adrenal and recurrent pheochromocytoma. MIBG is a precursor of catecholamines and
therefore, is actively taken up in catecholamines producing tissues. recently introduction of F-
DOPA PET has markedly increased the diagnostic accuracy of imaging investigation for
localization of both intra as well as extra adrenal pheochromocytoma.

QUESTION 163. MTQ3MStTQVNJUkVLSEEgUCtzYXNpcmVraGEzMTRAZ21haWwuY29tKzc3MDgwMDE4NzdR


ElPTiAxNjI=
What could be the clinical manifestation of this patient ?

Score:
DBMCI CLT MAINS 2018 755.00 /
Page 101 1200
CLT MAINS - 2018
Exam Title :
(Final)
Email : sasirekha314@gmail.com
Contact : 7708001877

a) Syncope
b) Seizures
c) Mental retardation
d) Ataxia
Correct Answer: B
Your Answer: B
Explanation

Ans. B. Seizures

This is a non contrast axial CT scan of the head taken at high parietal level. Here we can
identify multiple subcentimetric calcified lesions scattered in the neuro parenchyma typically at
the gray white matter interface. Some of the lesions on left side in posterior aspect have
surrounding hypodensity/edema. These findings suggest multiple infective granulomas, mostly
Neurocysticercosis.

The commonest clinical manifestation of NCC is seizures and headache

QUESTION 164. MTQ3MStTQVNJUkVLSEEgUCtzYXNpcmVraGEzMTRAZ21haWwuY29tKzc3MDgwMDE4NzdR


lPTiAxNjM=
The shortest life expectancy is associated with which clinical sign in amyotrophic
lateral sclerosis?

Score:
DBMCI CLT MAINS 2018 755.00 /
Page 102 1200
CLT MAINS - 2018
Exam Title :
(Final)
Email : sasirekha314@gmail.com
Contact : 7708001877

a) Atrophy of the gastrocnemius


b) Fasciculations in the lumbrical muscles
c) Atrophy of the pectoralis muscles
d) Fasciculations in the tongue
Correct Answer: D
Your Answer: D
Explanation

Ans. D. Fasciculations in the tongue

In ALS, early involvement of musculature supplied by the cranial nerves has a much graver
prognosis than early limb involvement. This may be a consequence of disturbed swallowing,
with recurrent aspiration as a result, or disturbed ventilatory activity. Fasciculations of the
tongue develop with deterioration of hypoglossal nuclei.

QUESTION 165. MTQ3MStTQVNJUkVLSEEgUCtzYXNpcmVraGEzMTRAZ21haWwuY29tKzc3MDgwMDE4NzdR


lPTiAxNjQ=
Maximum analgesic action is seen with:

a) Catecholamine
b) Propofol
c) Ketamine
d) Thiopentone
Correct Answer: C
Your Answer: B
Explanation

Ans. C. Ketamine

· Ketamine produces profound analgesia. It produces a state where one feels “dissociated” from
his own body.

· It does not produce unconsciousness. Propofol and thiopentone are the induction agents
causing unconsciousness quickly.

· They are poor analgesics. Catecholamines e.g. Adrenaline, Noradrenaline do not produce
analgesia.

QUESTION 166. MTQ3MStTQVNJUkVLSEEgUCtzYXNpcmVraGEzMTRAZ21haWwuY29tKzc3MDgwMDE4NzdR


lPTiAxNjU=
Anesthetic agent of choice in renal failure is:

a) Methoxyflurane
b) Isoflurane
c) Enflurane
d) None of the above
Correct Answer: B

Score:
DBMCI CLT MAINS 2018 755.00 /
Page 103 1200
CLT MAINS - 2018
Exam Title :
(Final)
Email : sasirekha314@gmail.com
Contact : 7708001877
Your Answer: D
Explanation

Ans. B. Isoflurane

· Methoxyflurane and enflurane are metabolized in the body producing inorganic fluoride which
is quite toxic to the renal tubules and can cause high-output renal failure; hence contra-
indicated in renal failure.

· Because of this problem, they are practically out of use nowadays.

Since isoflurane is sufficiently lipid insoluble; it is exhaled back through respiration before any
significant metabolism occurs.

QUESTION 167. MTQ3MStTQVNJUkVLSEEgUCtzYXNpcmVraGEzMTRAZ21haWwuY29tKzc3MDgwMDE4NzdR


ElPTiAxNjY=
A 6 yr old boy presents with small hypopigmented scaly macules on cheek. Some of his
friends also have similar lesions. Most probable diagnosis is?

a) Pityriasis rosea
b) Pityriasis versicolor
c) Indeterminate leprosy
d) Pityriasis alba
Correct Answer: D
Your Answer: D
Explanation

Ans. D. Pityriasis alba

· Occurs predominantly in children between the ages of 3 and 16 years. the sexes are equally
susceptible.

Score:
DBMCI CLT MAINS 2018 755.00 /
Page 104 1200
CLT MAINS - 2018
Exam Title :
(Final)
Email : sasirekha314@gmail.com
Contact : 7708001877

· The individual lesion is a rounded, oval or irregular plaque, which is red, pink or skin-coloured
and has fine lamellar or branny scaling. Initially, the erythema may be conspicuous and there
may even be minimal serous crusting. Later, the erythema subsides completely, and at the stage
at which the lesions are commonly seen by a physician they show only persistent fine scaling
and hypopigmentation.

· There are usually several patches ranging from 0.5 to 2 cm in diameter, but they may be
larger, especially on the trunk. In children, the lesions are often confined to the face, and are
most common around the mouth, chin and cheeks. In 20% of affected children the neck, arms
and there are scattered lesions on the trunk and limbs.

· The course is extremely variable. The average duration of the common facial form in childhood
is a year or more.

P. Versicolor.

· Scaly macule.

· Perifollicular

· Seen on upper trunk and shoulders commonly

· Rare in children and common in young adult

Indeterminated leprosy:

· Non scaly

· Epidermal atrophy

· Anesthesia

P. Rosea.

· Herald patch

· Collarette scaling

· Firtree appearance

· Common in females

QUESTION 168. MTQ3MStTQVNJUkVLSEEgUCtzYXNpcmVraGEzMTRAZ21haWwuY29tKzc3MDgwMDE4NzdR


ElPTiAxNjc=
A 48-year-old patient presents with dysphagia to both liquids and solids and
regurgitation for 3 months. The dysphagia was non progressive. What is the most
likely diagnosis?

a) Carcinoma esophagus
b) Lower esophageal mucosal ring
c) Achalasia cardia
d) Reflux esophagitis with esophageal stricture.

Score:
DBMCI CLT MAINS 2018 755.00 /
Page 105 1200
CLT MAINS - 2018
Exam Title :
(Final)
Email : sasirekha314@gmail.com
Contact : 7708001877
Correct Answer: C
Your Answer: B
Explanation

Ans. C. Achalasia cardia

Dysphagia of achalasia is progressive initially then reaches a plateau phase in most cases

· Dysphagia of carcinoma and stricture is clearly progressive.

· Regurgitation is also pointing towards achalasia

QUESTION 169. MTQ3MStTQVNJUkVLSEEgUCtzYXNpcmVraGEzMTRAZ21haWwuY29tKzc3MDgwMDE4NzdR


ElPTiAxNjg=
Which of the following is MOST SUGGESTIVE of Neonatal Small Bowel Obstruction?

a) Bilious vomiting
b) Refusal of feeds
c) Generalized abdominal distension
d) Failure to pass meconium in the first 24 hours
Correct Answer: A
Your Answer: A
Explanation

Ans. A. Bilious vomiting

Bilious vomiting in a neonate is not considered normal and has to be considered as


suggestive of Intestinal Obstruction until proven otherwise.

QUESTION 170. MTQ3MStTQVNJUkVLSEEgUCtzYXNpcmVraGEzMTRAZ21haWwuY29tKzc3MDgwMDE4NzdR


ElPTiAxNjk=
What is the probable diagnosis?

Score:
DBMCI CLT MAINS 2018 755.00 /
Page 106 1200
CLT MAINS - 2018
Exam Title :
(Final)
Email : sasirekha314@gmail.com
Contact : 7708001877

a) Ankylosing spondylitis
b) Diffuse idiopathic skeletal hyperostosis (DISH)
c) OA spine
d) Osteopetrosis
Correct Answer: A
Your Answer: A
Explanation

Ans. A. Ankylosing spondylitis

· Involvement of the lumbar spine and the classical Bamboo spine appearance points towards
the diagnosis of Ankylosing spondylitis.

· A close differential diagnosis remains to be DISH. See the table below to differentiate the
two conditions.

QUESTION 171. MTQ3MStTQVNJUkVLSEEgUCtzYXNpcmVraGEzMTRAZ21haWwuY29tKzc3MDgwMDE4NzdR


lPTiAxNzA=
Which of the following is NOT a likely diagnosis ?

Score:
DBMCI CLT MAINS 2018 755.00 /
Page 107 1200
CLT MAINS - 2018
Exam Title :
(Final)
Email : sasirekha314@gmail.com
Contact : 7708001877

a) Abscess
b) Glioblastoma
c) Metastasis
d) Meningioma
Correct Answer: D
Your Answer: D
Explanation

Ans. D. Meningioma

· Thick ring enhancing lesion with perilesional edema is seen in left temporal lobe. D/d of such
thick ring enhancing lesion include — abscess, high grade astrocytoma (glioblastoma), solitary
metastasis.

· Meningioma is NOT likely, for simple reason that meningioma is not a parenchymal (intra-
axial) lesion. It arises from arachnoid cap cells and is a extra-axial benign tumor
(extraparenchymal).

· Moreover meningiomas are highly vascular, and typically show homogenous vivid
enhancement. Dural tail sign may be seen.

QUESTION 172. MTQ3MStTQVNJUkVLSEEgUCtzYXNpcmVraGEzMTRAZ21haWwuY29tKzc3MDgwMDE4NzdR


lPTiAxNzE=
The arrow in this TVS image suggests the diagnosis of?

Score:
DBMCI CLT MAINS 2018 755.00 /
Page 108 1200
CLT MAINS - 2018
Exam Title :
(Final)
Email : sasirekha314@gmail.com
Contact : 7708001877

a) Endometrial polyp
b) Adenomyotic nodule
c) Calcified fibroid
d) AVM
Correct Answer: C
Your Answer: C
Explanation

Ans. C. Calcified fibroid

· The arrow points to a echogenic lesion with acoustic shadowing, suggesting calcification. This
calcification is in the myometrium- a calcified fibroid

· Thin, echogenic, endometrial line is normally seen.

· AVMs are better picked up on Doppler

· Adenomyosis is usually diffuse causing thinking of posterior wall with ‘globular ‘uterus and of
focal, the nodule is hypoechoic (black).

QUESTION 173. MTQ3MStTQVNJUkVLSEEgUCtzYXNpcmVraGEzMTRAZ21haWwuY29tKzc3MDgwMDE4NzdR


ElPTiAxNzI=
A patient came with history of supra condylar humerus fracture. After injury he hasn’t
been able to flex the thumb. The nerve most likely injured is:

a) Superficial branch of ulnar nerve

Score:
DBMCI CLT MAINS 2018 755.00 /
Page 109 1200
CLT MAINS - 2018
Exam Title :
(Final)
Email : sasirekha314@gmail.com
Contact : 7708001877

b) Deep branch of ulnar nerve


c) PIN
d) AIN
Correct Answer: D
Your Answer: D
Explanation

Ans. D. AIN

The most common nerve involved in Supracondylar humerus fractures is Anterior interosseous
nerve (AIN), a branch of median nerve. FPL is supplied by same.

QUESTION 174. MTQ3MStTQVNJUkVLSEEgUCtzYXNpcmVraGEzMTRAZ21haWwuY29tKzc3MDgwMDE4NzdR


PTiAxNzM=
What is the diagnosis of otoscopic Image?

a) Bullous myringitis
b) Vestibular schwannoma

Score:
DBMCI CLT MAINS 2018 755.00 /
Page 110 1200
CLT MAINS - 2018
Exam Title :
(Final)
Email : sasirekha314@gmail.com
Contact : 7708001877

c) Trauma of tympanic membrane


d) Glomus tumour
Correct Answer: D
Your Answer: D
Explanation

Ans. D. Glomus tumour

Vascular mass behind an intact tympanic membrane (which proved to be a paraganglioma).


There is also an air-fluid level present over the inferior aspect of the glomus.

QUESTION 175. MTQ3MStTQVNJUkVLSEEgUCtzYXNpcmVraGEzMTRAZ21haWwuY29tKzc3MDgwMDE4NzdR


lPTiAxNzQ=
Most potential route for transmission of meningitis from CNS to inner ear is:

a) Cochlear aqueduct
b) Endolymphatic sac
c) Vestibular aqueduct
d) Hyrtle fissure
Correct Answer: A
Your Answer: A
Explanation

Ans. A. Cochlear aqueduct

Cochlear aqueduct connects scala tympani with subarachnoid space. so infection can be
transmitted

QUESTION 176. MTQ3MStTQVNJUkVLSEEgUCtzYXNpcmVraGEzMTRAZ21haWwuY29tKzc3MDgwMDE4NzdR


lPTiAxNzU=
This instrument can be applied to all presentations except?

a) Face
b) Vertex
c) After coming head in breech

Score:
DBMCI CLT MAINS 2018 755.00 /
Page 111 1200
CLT MAINS - 2018
Exam Title :
(Final)
Email : sasirekha314@gmail.com
Contact : 7708001877

d) Brow
Correct Answer: D
Your Answer: D
Explanation

Ans. D. Brow

a. Simpson forceps are the most commonly used among the types of forceps and has an
elongated cephalic curve. These are used when there is substantial molding , that is, temporary
elongation of the fetal head as it moves through the birth canal

b. There is the ample pelvic curve in the single blade above and the cephalic curve evident in
the articulated blades below. The fenestrated blade and the wide shank in front of the English-
style lock characterize the Simpson forceps.

QUESTION 177. MTQ3MStTQVNJUkVLSEEgUCtzYXNpcmVraGEzMTRAZ21haWwuY29tKzc3MDgwMDE4NzdR


lPTiAxNzY=
Image below shows the treatment of a 3-month-child who had congenital dislocation of
hip (CDH). What is the apparatus in use called as?

a) Von Rosen splint


b) Pavlik harness
c) Petrie’s orthosis
d) Boston brace
Correct Answer: B
Your Answer: A
Explanation

Ans. B. Pavlik harness

Pavlik harness is an abduction orthosis used to treat cases of CDH in age group less than 6
months. By maintaining the joint in abduction it ensures containment of the head within the
acetabulum so that development remains normal and the joint remodels.

Score:
DBMCI CLT MAINS 2018 755.00 /
Page 112 1200
CLT MAINS - 2018
Exam Title :
(Final)
Email : sasirekha314@gmail.com
Contact : 7708001877
QUESTION 178. MTQ3MStTQVNJUkVLSEEgUCtzYXNpcmVraGEzMTRAZ21haWwuY29tKzc3MDgwMDE4NzdR
ElPTiAxNzc=
The Vipoma syndrome also is called the WDHA syndrome because of the presence of ;

a) Watery diarrhea, hypokalemia, and achlorhydria


b) Watery diarrhea, hyperchloremia and achlorhydria
c) Watery diarrhea, hyperkalemia, and achlorhydria
d) Watery diarrhea, hypochloremia, and achlorhydria
Correct Answer: A
Your Answer: A
Explanation

Ans. A. Watery diarrhea, hypokalemia, and achlorhydria

Vipoma

· Syndrome associated with a pancreatic neoplasm secreting vasoactive intestinal polypeptide


(VIP).

· The Vipoma syndrome also is called the WDHA syndrome because of the presence of watery
diarrhea, hypokalemia, and achlorhydria.

· Serum VIP levels must be measured on multiple occasions because the excess secretion of VIP
is episodic, and single measurements might be normal and misleading.

· A CT scan localizes most Vipoma, although as with all islet cell tumors, EUS is the most
sensitive imaging method.

· Vipoma are more commonly located in the distal pancreas and most have spread outside the
pancreas.

· Palliative debulking operations can sometimes improve symptoms for a period along with
Somatostatin analogy.

· Hepatic artery embolization also has been reported as a potentially beneficial treatment

QUESTION 179. MTQ3MStTQVNJUkVLSEEgUCtzYXNpcmVraGEzMTRAZ21haWwuY29tKzc3MDgwMDE4NzdR


lPTiAxNzg=
Most common complication of extracapsular cataract surgery is -

a) Retinal detachment
b) Opacification of posterior capsule
c) Vitreous haemorrhage
d) None
Correct Answer: B
Your Answer: B
Explanation

Ans. B. Opacification of posterior capsule

Score:
DBMCI CLT MAINS 2018 755.00 /
Page 113 1200
CLT MAINS - 2018
Exam Title :
(Final)
Email : sasirekha314@gmail.com
Contact : 7708001877

"Opacification of the posterior capsule caused by proliferation of cells in capsular bag remains
the most frequent complication of ECCE and IOL surgery".

QUESTION 180. MTQ3MStTQVNJUkVLSEEgUCtzYXNpcmVraGEzMTRAZ21haWwuY29tKzc3MDgwMDE4NzdR


lPTiAxNzk=
Nerve supply to the lacrimal sac is:

a) Supratrochlear nerve
b) Nasociliary nerve
c) Infraorbital nerve
d) Supraorbital nerve
Correct Answer: B
Your Answer: A
Explanation

Ans. B. Nasociliary nerve

· The lacrimal sac is the upper dilated end of the nasolacrimal duct, and is lodged in a deep
groove formed by the lacrimal bone and frontal process of the maxilla. It connects the lacrimal
canaliculi, which drain tears from the eye's surface, and the nasolacrimal duct, which conveys
this fluid into the nasal cavity.

· Infratrochlear nerve is the terminal branch of the nasociliary nerve.

· It supplies the lacrimal sac; medial conjunctiva and canaliculi.

QUESTION 181. MTQ3MStTQVNJUkVLSEEgUCtzYXNpcmVraGEzMTRAZ21haWwuY29tKzc3MDgwMDE4NzdR


PTiAxODA=
Which of the following is a treatment option for Volkmann’s ischemia?

a) Fasciotomy
b) Cock up splint
c) Bone shortening
d) Turn buckle splint
Correct Answer: A
Your Answer: A
Explanation

Ans. A. Fasciotomy

Volkmann’s ischemia is synonymous with Compartment syndrome. If the same is missed, there
occurs necrosis followed by fibrosis of fore arm flexors leading to a flexion contracture in hand
also called as Volkmann’s ischemic contracture.

Score:
DBMCI CLT MAINS 2018 755.00 /
Page 114 1200
CLT MAINS - 2018
Exam Title :
(Final)
Email : sasirekha314@gmail.com
Contact : 7708001877
QUESTION 182. MTQ3MStTQVNJUkVLSEEgUCtzYXNpcmVraGEzMTRAZ21haWwuY29tKzc3MDgwMDE4NzdR
PTiAxODE=
Identify the projective personality test?

a) CAT
b) TAT
c) MMPI
d) Rorschach Test
Correct Answer: A
Your Answer: Unanswered
Explanation

Ans. A. CAT

· It is Children Apperception Test [CAT]- modified for Indian children by Uma Chaudhry i.e.
Indian Modification of Children's Apperception Test (CAT).

· It is projective personality test used in children, to assess their cause of psychological stress.

QUESTION 183. MTQ3MStTQVNJUkVLSEEgUCtzYXNpcmVraGEzMTRAZ21haWwuY29tKzc3MDgwMDE4NzdR


lPTiAxODI=
Which among the following is named as New Psychoactive Substance [NPS]?

a) Cocaine
b) Bath Salt
c) Cannabis
d) LSD
Correct Answer: B

Score:
DBMCI CLT MAINS 2018 755.00 /
Page 115 1200
CLT MAINS - 2018
Exam Title :
(Final)
Email : sasirekha314@gmail.com
Contact : 7708001877
Your Answer: D
Explanation

Ans. B. Bath Salt

· Bath Salt - is synthetic cathinones also known as- mephedrone/ meaw-meaw it is discovered as
new psychoactive substance/ substance of abuse.

· They are new means of getting high/ euphoria.

QUESTION 184. MTQ3MStTQVNJUkVLSEEgUCtzYXNpcmVraGEzMTRAZ21haWwuY29tKzc3MDgwMDE4NzdR


PTiAxODM=
A patient presenting with painful Myositis Ossificans around Elbow. What should be
the preferred treatment option in this case?

a) Active mobilization
b) Passive mobilization
c) Infra Red Therapy
d) Immobilization
Correct Answer: D
Your Answer: D
Explanation

Ans. D. Immobilization

· Treatment of myositis ossificans (MO):when MO is active, immobilisation is the best treatment


& any attempt at removal of the mass is hazardous.

· When silent and ALP/ESR levels are normal, it can safely be removed (generally after 1.5
years) if causing block in movements.

· Indomethacin / low dose radiation / bisphosphonates can be given prophylactically in high risk
patients to prevent MO formation.

QUESTION 185. MTQ3MStTQVNJUkVLSEEgUCtzYXNpcmVraGEzMTRAZ21haWwuY29tKzc3MDgwMDE4NzdR


PTiAxODQ=
Which Part of the fallopian tube which is most common site for female tubal
sterilization?

a) Cornua
b) Ampulla
c) Isthmus
d) Infundibulum
Correct Answer: C
Your Answer: C
Explanation

Ans. C. Isthmus

Score:
DBMCI CLT MAINS 2018 755.00 /
Page 116 1200
CLT MAINS - 2018
Exam Title :
(Final)
Email : sasirekha314@gmail.com
Contact : 7708001877

· For female tubal sterilization, Isthmus is said to be the most common site. It is the part of the
fallopian tube which is sealed or cut in a sterilization procedure or tubal ligation.

· The mid isthmic portion of the tube is grasped in the bipolar electrocoagulation technique.
Additionally, radiofrequency electric current is applied to three adjacent areas and coagulating
3 cm of tube. Hulka clips are also placed across the mid isthmus.

QUESTION 186. MTQ3MStTQVNJUkVLSEEgUCtzYXNpcmVraGEzMTRAZ21haWwuY29tKzc3MDgwMDE4NzdR


PTiAxODU=
The following reflex will be brisk in all of the following conditions except:

a) Pseudobulbar palsy
b) MND
c) Multiple sclerosis
d) Syringomyelia
Correct Answer: D
Your Answer: D
Explanation

Ans. D. Syringomyelia

· Jaw jerk-Afferent impulse of this reflex are carried through sensory portion of the trigeminal
nerve the efferent impulse through trigeminal nerve with its reflex centre in PONS.

· So this jerk like any other jerk will become brisk in a UMN lesion/supranuclear lesion OR a
lesion above pons, never in a lesion with the spinal cord. All are lesions supranuclear/UMN
except Syringomyelia

Score:
DBMCI CLT MAINS 2018 755.00 /
Page 117 1200
CLT MAINS - 2018
Exam Title :
(Final)
Email : sasirekha314@gmail.com
Contact : 7708001877
QUESTION 187. MTQ3MStTQVNJUkVLSEEgUCtzYXNpcmVraGEzMTRAZ21haWwuY29tKzc3MDgwMDE4NzdR
lPTiAxODY=
Which of the following is true about obstructive azoospermia ?

a) ↑FSH and ↑LH


b) Normal FSH and Normal LH
c) ↑LH Normal FSH
d) ↑FSH , Normal LH
Correct Answer: B
Your Answer: B
Explanation

Ans. B. Normal FSH and Normal LH

Obstructive Azoospermia is associated with normal levels of FSH and LH.

Azoospermia

Hypogonadotrophic Hypergonadotropic
Obstructive Azoospermia
Azoospermia Azoospermia

Azoospermia due to Azoospermia due to


obstructive lesion invas or hypothalamic or pituitary
epididymis(Bilateral) failure Azoospermia due to testicular
(end organ) failure
o Normal FSH o Low FSH
o Persistently elevated FSH
o Normal LH o Low LH
o Testicular volume is usually
o Normal Testosterone o Low Testosterone
reduced
o Testicular volume is usually Testicular volume is usually
normal reduced

QUESTION 188. MTQ3MStTQVNJUkVLSEEgUCtzYXNpcmVraGEzMTRAZ21haWwuY29tKzc3MDgwMDE4NzdR


lPTiAxODc=
A female aged 25 presents with a 12 month history of palpitations, intermittent
diarrhea, anxiety, and a 1-month history of "bulging of both eyes."Most likely cause of
her symptoms is which among he following ?

a) Graves' disease
b) Hashimoto's thyroiditis
c) Multinodular toxic goiter
d) Papillary carcinoma
Correct Answer: A
Your Answer: A
Explanation

Ans. A. Graves' disease

Score:
DBMCI CLT MAINS 2018 755.00 /
Page 118 1200
CLT MAINS - 2018
Exam Title :
(Final)
Email : sasirekha314@gmail.com
Contact : 7708001877

· The most common cause of hyperthyroidism in a young women is referred to as Graves'


disease and this disease is the only one that leads to exophthalmos (i.e. bulging of both eyes).

· This is an autoimmune disorder where a thyroid-stimulating IgG immunoglobulin (TSI) binds


to the TSH receptors resulting in elevated thyroid hormone release.

· Lymphocytic infiltration of the extraocular muscles leads to exophthalmos.

· Hashimoto's thyroiditis causes hypothyroidism and is related with antimicrosomal antibodies


and diffusely enlarged thyroid gland and against the thyroid parenchyma.

· Multinodular toxic goiter also results in hyperthyroidism, but does not lead to exophthalmos.
Papillary carcinoma will only very rarely present as a hypersecreting nodule.

· Most cases will be cold, nonsecreting nodules. There is no exophthalmos.

· This is considered to be one of the most common thyroid cancer. Out of all the thyroid cancers
this type has the best prognosis.

QUESTION 189. MTQ3MStTQVNJUkVLSEEgUCtzYXNpcmVraGEzMTRAZ21haWwuY29tKzc3MDgwMDE4NzdR


lPTiAxODg=
Which of the following agent is most commonly associated with carcinoma cervix :

a) HPV 16
b) HPV 18
c) HPV 33
d) HPV 35
Correct Answer: A
Your Answer: A
Explanation

Ans. A. HPV 16

Overall the most common agent associated with carcinoma cervix is HPV 16 (followed
by HPV 18)

HPV 16 is the dominant cancer related HPV accounting for 40-70 percent of invasive s
quamous cell cervical worldwide. This serotype is also the most common HPV found among low
grade lesion and in women without neoplasia. The prevalence of HPV 18 is much lower than
that of HPV 16 in an even higher proportion or cervical adenocarcinomas and adenosquamous
carcinomas.

HPV and Cervical Cancer

· HPV is the primary etiological infection agent in cervical cancer

· HPV DNA is found in 99.7% of cervical cancers

· More than 100 HPV types have now been identified and have been classified as High Risk(HR)
or low risk (LR) subtypes based on their cervical cancer oncogenicity

Overall the most common single agent is HPV 16

Score:
DBMCI CLT MAINS 2018 755.00 /
Page 119 1200
CLT MAINS - 2018
Exam Title :
(Final)
Email : sasirekha314@gmail.com
Contact : 7708001877

The most prevalent HPV subtype in squamous all carcinoma is HPV 16

The most prevalent HPV subtype in adenocarcinoma cervix is HPV 18

Low risk HPV subtypes High risk HPV subtypes

· Example include HPV types


16,18,31,33,35,39,45,51,52,56,58,59,68,73
and 82
· Example include HPV type 6,11,42,43
· These agents are associated with high
and 44
grade lesions (CIN II, CIN III) and invasive
· These agents are associated with cancers in addition to condylomata and
condylomata and low grade lesions eg CIN low grade lesions
I
The most prevalent High Risk HPV
subtypes

Are 16,18,45 and 31

QUESTION 190. MTQ3MStTQVNJUkVLSEEgUCtzYXNpcmVraGEzMTRAZ21haWwuY29tKzc3MDgwMDE4NzdR


lPTiAxODk=
Clinical & histopathological Images shown. Diagnosis?

Score:
DBMCI CLT MAINS 2018 755.00 /
Page 120 1200
CLT MAINS - 2018
Exam Title :
(Final)
Email : sasirekha314@gmail.com
Contact : 7708001877

a) Atopic dermatitis
b) Syringoma
c) Sarcoidosis
d) Lichen nitidus
Correct Answer: B
Your Answer: B
Explanation

Ans. B. Syringoma

Syringoma: being tumour of sweat glands , in which there are small cords and strands of
epithelial cells, some in a “tadpole” configuration. There are scattered lumens, often lined with
clear cells, with a bluish substance within them.

QUESTION 191. MTQ3MStTQVNJUkVLSEEgUCtzYXNpcmVraGEzMTRAZ21haWwuY29tKzc3MDgwMDE4NzdR


lPTiAxOTA=
Blood collected from acid citrate dextrose can be stored for:

a) 28 days
b) 21 days
c) 14 days
d) 7 days
Correct Answer: B
Your Answer: Unanswered
Explanation

Ans. B. 21 days

· With CPDA1 (citrate phosphate dextrose adenine) blood can be stored for 35 days.

Score:
DBMCI CLT MAINS 2018 755.00 /
Page 121 1200
CLT MAINS - 2018
Exam Title :
(Final)
Email : sasirekha314@gmail.com
Contact : 7708001877

· With ACD (acid citrate dextrose) it can be stored for 21 days.

QUESTION 192. MTQ3MStTQVNJUkVLSEEgUCtzYXNpcmVraGEzMTRAZ21haWwuY29tKzc3MDgwMDE4NzdR


lPTiAxOTE=
The local anesthetic known to cause methemoglobinemia as adverse effect is:

a) Lidocaine
b) Bupivacaine
c) Tetracaine
d) Prilocaine
Correct Answer: D
Your Answer: D
Explanation

Ans. D. Prilocaine

· The systemic toxicity of prilocaine is comparatively low, however its metabolite, o-toluidine, is
known to cause methemoglobinemia.

· As methemoglobinemia reduces the amount of hemoglobin that is available for oxygen


transport, this side effect is potentially life threatening.

· Therefore dose limits for prilocaine should be strictly observed. Prilocaine is not recommended
for use in labor pains or infants.

QUESTION 193. MTQ3MStTQVNJUkVLSEEgUCtzYXNpcmVraGEzMTRAZ21haWwuY29tKzc3MDgwMDE4NzdR


ElPTiAxOTI=
Identify the instrument shown in the Image below?

Score:
DBMCI CLT MAINS 2018 755.00 /
Page 122 1200
CLT MAINS - 2018
Exam Title :
(Final)
Email : sasirekha314@gmail.com
Contact : 7708001877

a) Applanation tonometer
b) Indentation Tonometer
c) Pascal Dynamic Contour Tonometer
d) Rebound Tonometer
Correct Answer: A
Your Answer: Unanswered
Explanation

Ans. A. Applanation tonometer

TONOMETRY

§ It is the procedure eye care professionals perform to determine the intraocular pressure
(IOP), the fluid pressure inside the eye.

Methods:

APPLANATION TONOMETRY

§ Applanation tonometry is based on the Imbert-Fick principle, which states that the pressure
inside an ideal dry, thin-walled sphere equals the force necessary to flatten its surface divided
by the area of flattening (P = F/A, where P = pressure, F = force and A = area). In applanation
tonometry, the cornea is flattened and the IOP is determined by varying the applanating force
or the area flattened.

1. The Goldmann applanation tonometer measures the force necessary to flatten an area of the
cornea of 3.06mm diameter.

2. The Perkins tonometer is a type of portable applanation tonometer, which may be useful in
children, anesthetised patients who need to lie flat, or patients unable to co-operate with a
sitting slit lamp examination.

3. Non-Contact Tonometry

§ In air puff tonometry, the applanating force is a column of air which is emitted with gradually
increasing intensity.

1. Ocular Response Analyzer

INDENTATION TONOMETRY

§ The principle of indentation tonometry is that a force or a weight will indent or sink into a soft
eye further than into a hard eye.

1. Schiotz Tonometer

2. It consists of a curved footplate which is placed on the cornea of a supine subject. A weighted
plunger attached to the footplate sinks into the cornea in an amount that is indirectly
proportional to the pressure in the eye.

3. Pneumo tonometer

· It is an applanation tonometer with some aspects of indentation tonometry. It consists of a


5mm diameter, slightly convex, silicone tip on the end of a piston that rides on a stream of air.
The cornea is indented by the silicone tip. When the cornea and the tip are flat, the pressure
pushing forward on the tip is equal to the IOP.

Score:
DBMCI CLT MAINS 2018 755.00 /
Page 123 1200
CLT MAINS - 2018
Exam Title :
(Final)
Email : sasirekha314@gmail.com
Contact : 7708001877

1. Tono-Pen

· The Tono-Pen involves both applanation and indentation processes

REBOUND TONOMETRY

· It determines intraocular pressure by bouncing a small plastic tipped metal probe against the
cornea. The device uses an induction coil to magnetise the probe and fire it against the cornea.
As the probe bounces against the cornea and back into the device, it creates an induction
current from which the intraocular pressure is calculated.

PASCAL DYNAMIC CONTOUR TONOMETER

· The Pascal Dynamic Tonometer utilizes a piezoelectric sensor embedded in the tip of the
tonometer to measure the dynamic pulsatile fluctuations in IOP.

QUESTION 194. MTQ3MStTQVNJUkVLSEEgUCtzYXNpcmVraGEzMTRAZ21haWwuY29tKzc3MDgwMDE4NzdR


PTiAxOTM=
Which of the following vitamins or minerals needs to be supplemented in infants that
are exclusively breast-fed?

a) Iron
b) Vitamin C
c) Vitamin D
d) Calcium
Correct Answer: C
Your Answer: C
Explanation

Ans. C. Vitamin D

· Vitamin D is found in smaller amounts in breast milk when compared with infant formulas.

· Infants that are exclusively breast-fed need to be supplemented with vitamin D.

· Iron is also found in smaller amounts in breast milk; however, it has increased bioavailability,
and supplementation is not required.

· Vitamin C, calcium, and folic acid are found in the appropriate quantities in breast milk.

QUESTION 195. MTQ3MStTQVNJUkVLSEEgUCtzYXNpcmVraGEzMTRAZ21haWwuY29tKzc3MDgwMDE4NzdR


lPTiAxOTQ=
Which clinical sign is demonstrated in the Image:

Score:
DBMCI CLT MAINS 2018 755.00 /
Page 124 1200
CLT MAINS - 2018
Exam Title :
(Final)
Email : sasirekha314@gmail.com
Contact : 7708001877

a) Hyperreflexia
b) Myotonia
c) Babinski.
d) Myerson sign.
Correct Answer: D
Your Answer: D
Explanation

Ans. D. Myerson sign.

· Myerson’s sign or glabellar tap sign is a medical condition where a patient is unable to resist
blinking when tapped on the glabella, the area above the nose and between the eyebrows.

· It is often referred to as the glabellar reflex. It is often an early symptom of Parkinson's


disease, but can also be seen in early dementia as well as other progressive neurologic illness.

· It is named for Abraham Myerson, an American neurologist.

QUESTION 196. MTQ3MStTQVNJUkVLSEEgUCtzYXNpcmVraGEzMTRAZ21haWwuY29tKzc3MDgwMDE4NzdR


PTiAxOTU=
A 45 years old female presents with pruritus and jaundice. She has raised ALP and
GGT. Anti-mitochondrial antibody was positive. What is the next line of investigation?

a) ERCP
b) MRCP
c) PET
d) Liver Biopsy
Correct Answer: B
Your Answer: D
Explanation

Score:
DBMCI CLT MAINS 2018 755.00 /
Page 125 1200
CLT MAINS - 2018
Exam Title :
(Final)
Email : sasirekha314@gmail.com
Contact : 7708001877

Ans. B. MRCP

· MRCP is an excellent imaging technique for gall bladder and biliary system.

· It can demonstrate ductal obstruction, strictures or other intra ductal abnormalities.

· Contrast is not required.

· The patient in question is most probably a case of primary biliary cirrhosis.

QUESTION 197. MTQ3MStTQVNJUkVLSEEgUCtzYXNpcmVraGEzMTRAZ21haWwuY29tKzc3MDgwMDE4NzdR


lPTiAxOTY=
According to WHO cut off level for hemoglobin in child for anemia is?

a) 10 gm
b) 11 gm
c) 12 gm
d) 13 gm
Correct Answer: B
Your Answer: C
Explanation

Ans. B. 11 gm

Classification of mental retardation:

· The definition of anemia in infancy and childhood is different from adults.

QUESTION 198. MTQ3MStTQVNJUkVLSEEgUCtzYXNpcmVraGEzMTRAZ21haWwuY29tKzc3MDgwMDE4NzdR


lPTiAxOTc=
Treatment for a women suffering from Antiphospholipid antibody syndrome (APL) with
past history of stillbirths and abortion is:

a) Aspirin only
b) Low dose Aspirin + LMW heparin
c) Start Low dose Aspirin + LMW heparin + Prednisolone
d) No treatment
Correct Answer: B
Your Answer: C
Explanation

Score:
DBMCI CLT MAINS 2018 755.00 /
Page 126 1200
CLT MAINS - 2018
Exam Title :
(Final)
Email : sasirekha314@gmail.com
Contact : 7708001877

Ans. B. Low dose Aspirin + LMW heparin

· Combined unfractionated heparin and low-dose aspirin regimens are thought to reduce the
risk of spontaneous pregnancy loss by 54%, resulting in a live-birth rate of 70-80%.

· Evidence from small, controlled trials (25-50 patients) has suggested that patients with
recurrent pregnancy loss associated with antiphospholipid antibodies and without prior
thromboembolism benefit from treatment with low-dose unfractionated heparin at prophylactic
doses of 5,000-10,000 IU twice daily, in addition to low-dose aspirin at 70-81 mg daily.

QUESTION 199. MTQ3MStTQVNJUkVLSEEgUCtzYXNpcmVraGEzMTRAZ21haWwuY29tKzc3MDgwMDE4NzdR


lPTiAxOTg=
Identify the fetal lie:

a) Complete breech
b) Incomplete breech
c) Frank breech
d) Vertex
Correct Answer: B
Your Answer: B
Explanation

Ans. B. Incomplete breech

Score:
DBMCI CLT MAINS 2018 755.00 /
Page 127 1200
CLT MAINS - 2018
Exam Title :
(Final)
Email : sasirekha314@gmail.com
Contact : 7708001877

QUESTION 200. MTQ3MStTQVNJUkVLSEEgUCtzYXNpcmVraGEzMTRAZ21haWwuY29tKzc3MDgwMDE4NzdR


lPTiAxOTk=
The single most important test that serves as an indicator of the severity of acute viral
hepatitis is:

a) ALT
b) AST
c) HBsAg
d) Prothrombin time
Correct Answer: D
Your Answer: D
Explanation

Ans. D. Prothrombin time

The prothrombin test gives an indication of the degree of hepatic injury. The other tests listed
do not. A prolonged prothrombin time can be an ominous sign.

QUESTION 201. MTQ3MStTQVNJUkVLSEEgUCtzYXNpcmVraGEzMTRAZ21haWwuY29tKzc3MDgwMDE4NzdR


PTiAyMDA=
A female patient is on 5mg haloperidol since 7months, recently for last 4 days of
duration he has developed abnormal, involuntary irregular movements of neck which
is increased by stress and decrease during sleep. Diagnosis is?

a) Akathisia

Score:
DBMCI CLT MAINS 2018 755.00 /
Page 128 1200
CLT MAINS - 2018
Exam Title :
(Final)
Email : sasirekha314@gmail.com
Contact : 7708001877

b) Tardive dyskinesia
c) Rabbit syndrome
d) Acute dystonia
Correct Answer: B
Your Answer: B
Explanation

Ans. B. Tardive dyskinesia

· As the patient in question is taking anti psychotic since long more than 6 months so she will
definitely experiencing some delayed side effect i.e. Tardive [late onset] dyskinesia [ she has
developed involuntary, abnormal irregular movements]

· Treatment of Tardive dyskinesia is - Clozapine

QUESTION 202. MTQ3MStTQVNJUkVLSEEgUCtzYXNpcmVraGEzMTRAZ21haWwuY29tKzc3MDgwMDE4NzdR


PTiAyMDE=
Evidence based use of this biological therapy is in which psychiatric disorder?

a) Schizophrenia
b) OCD
c) Depression
d) None of the above
Correct Answer: C
Your Answer: C
Explanation

Ans. C. Depression

· This is rTMS - its evidence based use is only in depression when used for mental disorders

· It is opd based procedure

QUESTION 203. MTQ3MStTQVNJUkVLSEEgUCtzYXNpcmVraGEzMTRAZ21haWwuY29tKzc3MDgwMDE4NzdR


lPTiAyMDI=
A 63-year-old female presented to the emergency room with an acute aneurysmal
subarachnoid hemorrhage (SAH) and underwent a clipping procedure on admission.
She has since been recovering in the intensive care unit. On the tenth postoperative
day, she was noted to have slurred speech, worsening right facial droop, and right arm
weakness. She was more difficult to arouse. Non-contrast head computed tomography
(CT) and four-vessel angiogram are shown shows ischemia in MCA territory. Which of
the following is the best treatment for her condition?

a) I/V rtPA
b) Intraarterial Calcium channel blockers.
c) VP Shunt.
d) I/V Fosphenytoin.
Correct Answer: B

Score:
DBMCI CLT MAINS 2018 755.00 /
Page 129 1200
CLT MAINS - 2018
Exam Title :
(Final)
Email : sasirekha314@gmail.com
Contact : 7708001877
Your Answer: B
Explanation

Ans. B. Intraarterial Calcium channel blockers.

· The patient is suffering from post subarachnoid hemorrhage vasospasm. This is a common
complication that can occur in the first 14 days after SAH, and can lead to further neurological
deficits or even death if not quickly recognized and treated. The main emergent therapy
remains intraarterial administration of calcium channel blockers like nimodipine and
angioplasty.

· IV tPA would not be indicated, even if this patient had an ischemic stroke, because of her
recent hemorrhage and cranial surgery.

· Although seizures are a possible cause of reduced level of alertness, new focal findings in the
setting of an SAH should prompt an evaluation for vasospasm.

QUESTION 204. MTQ3MStTQVNJUkVLSEEgUCtzYXNpcmVraGEzMTRAZ21haWwuY29tKzc3MDgwMDE4NzdR


PTiAyMDM=
A 57-year-old woman presents with difficulty swallowing and a complaint of a lump in
the throat. Describe the abnormality seen on her barium swallow:

a) Zenker’s diverticulum
b) Ca hypopharynx
c) Ca oesophagus
d) All of the above
Correct Answer: A
Your Answer: A
Explanation

Ans. A. Zenker’s diverticulum

It shows a smooth-surfaced filling defect in the barium column which arises from the posterior
pharyngeal wall at approximately the level of the 5th cervical vertebra. This is the typical
appearance of a prominent cricopharyngeus muscle. In this case, there is no associated
Zenker’s diverticulum.

QUESTION 205. MTQ3MStTQVNJUkVLSEEgUCtzYXNpcmVraGEzMTRAZ21haWwuY29tKzc3MDgwMDE4NzdR


PTiAyMDQ=
A teacher presented with tiredness of voice and hoarseness. On laryngoscopic
examination, keyhole appearance of glottis present. Which of the following is true
regarding management:

a) Treated with voice rest and speech therapy


b) Needs surgery of excision of muscle
c) Case of failure of abduction of muscles
d) Intra cordal injection of botulinum injection
Correct Answer: A

Score:
DBMCI CLT MAINS 2018 755.00 /
Page 130 1200
CLT MAINS - 2018
Exam Title :
(Final)
Email : sasirekha314@gmail.com
Contact : 7708001877
Your Answer: D
Explanation

Ans. A. Treated with voice rest and speech therapy

It is a case of phonasthenia. Treatment is voice rest and speech therapy.

QUESTION 206. MTQ3MStTQVNJUkVLSEEgUCtzYXNpcmVraGEzMTRAZ21haWwuY29tKzc3MDgwMDE4NzdR


PTiAyMDU=
Most common Cause of pulses paradoxus are:

a) Restrictive cardiomyopathy
b) Constrictive pericarditis
c) Cardiac tamponade
d) All
Correct Answer: C
Your Answer: C
Explanation

Ans. C. Cardiac Tamponade

QUESTION 207. MTQ3MStTQVNJUkVLSEEgUCtzYXNpcmVraGEzMTRAZ21haWwuY29tKzc3MDgwMDE4NzdR


PTiAyMDY=
The cells seen after 72 hours in the infracted area in MI are:

a) Neutrophils
b) Lymphocytes
c) Macrophages
d) Monocytes
Correct Answer: C
Your Answer: C
Explanation

Ans. C. Macrophages

· Acute inflammatory reaction with a predominance of neutrophils is seen up to 2-3 days.

Score:
DBMCI CLT MAINS 2018 755.00 /
Page 131 1200
CLT MAINS - 2018
Exam Title :
(Final)
Email : sasirekha314@gmail.com
Contact : 7708001877

· After 3 days (i.e. after 72 hours).

· Macrophages become the predominant cell type and become involved in removal of necrotic
debris.

QUESTION 208. MTQ3MStTQVNJUkVLSEEgUCtzYXNpcmVraGEzMTRAZ21haWwuY29tKzc3MDgwMDE4NzdR


PTiAyMDc=
This a delayed (1 hour) phase image of an IVU study. The apparent finding in this
study is ?

a) Right hydronephrosis
b) Left hydronephrosis and proximal hydroureter
c) UB rupture
d) Right ureterocele
Correct Answer: B
Your Answer: B
Explanation

Ans B. Left hydronephrosis and proximal hydroureter

This is a delayed phase image. The contrast material in the renal pelvicalyceal system is
appearing white as it is Iodinated contrast material – a positive contrast material. Most of the
contrast on right side has washed off. Hence the right ureter is not well seen.

It is apparent that the left pelvicalyceal system is dilated along with dilatation of the proximal
left ureter as well. This could be due to a left ureteric calculus, but it is not possible to see the
calculus here as the contrast will overlap the opacity of the calculus.

Score:
DBMCI CLT MAINS 2018 755.00 /
Page 132 1200
CLT MAINS - 2018
Exam Title :
(Final)
Email : sasirekha314@gmail.com
Contact : 7708001877
QUESTION 209. MTQ3MStTQVNJUkVLSEEgUCtzYXNpcmVraGEzMTRAZ21haWwuY29tKzc3MDgwMDE4NzdR
PTiAyMDg=
The drug of choice to treat Chlamydia infection in pregnancy is :

a) Tetracycline
b) Doxycycline
c) Erythromycin
d) Azithromycin
Correct Answer: D
Your Answer: C
Explanation

Ans. D. Azithromycin

· The drug of choice for treatment of chlamydial infection in pregnancy is Azithromycin.

· Azithromycin has now replaced erythromycin as the drug of first choice for the treatment of
Chlamydia infection in pregnancy.

· Azithromycin has the distinct advantage of single dose treatment (Ig as a single dose ) and
hence a 100% compliance

· The recommended regimen during pregnancy (recent) is Azithromycin or Amoxycillin with

Erythromycin as third choice.

Treatment of Chlamydia in pregnancy:

Most recent recommendation

Regimen Drug

First choice Azithromycin or Amoxycillin

Erythromycin base

Alternative Erythromycin

Ethylsuccinate

QUESTION 210. MTQ3MStTQVNJUkVLSEEgUCtzYXNpcmVraGEzMTRAZ21haWwuY29tKzc3MDgwMDE4NzdR


PTiAyMDk=
Fetal blood loss in abnormal cord insertion is seen in:

a) Vasa previa
b) Decidua basalis
c) Battle dore placenta

Score:
DBMCI CLT MAINS 2018 755.00 /
Page 133 1200
CLT MAINS - 2018
Exam Title :
(Final)
Email : sasirekha314@gmail.com
Contact : 7708001877

d) Succenturiate placenta
Correct Answer: A
Your Answer: A
Explanation

Ans. A. Vasa previa

· Vasa previa is an uncommon condition in which fetal blood vessels traverse the lower uterine
segment in advance of the presenting part.

· Neither the umbilical cord nor the placenta supports the vessels.

· Vasa previa presents with painless vaginal bleeding at the time of spontaneous rupture of
membranes or amniotomy (AROM).

· Fetal shock or demise can occur rapidly.

· Fetal mortality for cases not recognized before the onset of labor is reported to range between
22.5% and 100

QUESTION 211. MTQ3MStTQVNJUkVLSEEgUCtzYXNpcmVraGEzMTRAZ21haWwuY29tKzc3MDgwMDE4NzdR


PTiAyMTA=
Commonest cause of first trimester abortion is:

a) Monosomy
b) Trisomy
c) Triploidy
d) Aneuploidy
Correct Answer: D
Your Answer: B

Score:
DBMCI CLT MAINS 2018 755.00 /
Page 134 1200
CLT MAINS - 2018
Exam Title :
(Final)
Email : sasirekha314@gmail.com
Contact : 7708001877
Explanation

Ans. D. Aneuploidy

· The most common cause of first trimester abortion is aneuploidy and among it the most
common cause is trisomy as a group. The most common individual cause is monosomy X

· The most common cause of recurrent abortions is idiopathic followed by APLA followed by
uterine structural anomalies

QUESTION 212. MTQ3MStTQVNJUkVLSEEgUCtzYXNpcmVraGEzMTRAZ21haWwuY29tKzc3MDgwMDE4NzdR


PTiAyMTE=
One month after receiving a 14-day course of omeprazole, clarithromycin, and
amoxicillin for Helicobacter pylori–associated gastric ulcer disease, a 44-year-old
woman still has mild dyspepsia and pain after meals. What is the appropriate next step
in management?

a) Empirical long-term proton pump inhibitor therapy


b) Endoscopy with biopsy to rule out gastric adenocarcinoma
c) H. pylori serology testing
d) Urea breath test
Correct Answer: D
Your Answer: D
Explanation

Ans. D. Urea breath test

· It is impossible to know whether the patient’s continued dyspepsia is due to persistent H.


pylori as a result of treatment failure or to some other cause.

· A quick non-invasive test to look for the presence of H. pylori is a urea breath test. This test
can be done as an outpatient and gives a rapid, accurate response. Patients should not have
received any proton pump inhibitors or antimicrobials in the meantime. Stool antigen test is
another good option if urea breath testing is not available.

· If the urea breath test is positive >1 month after completion of first-line therapy, second-line
therapy with a proton pump inhibitor, bismuth subsalicylate, tetracycline, and metronidazole
may be indicated.

· If the urea breath test is negative, the remaining symptoms are unlikely due to persistent H.
pylori infection. Serology is useful only for diagnosing infection initially, but it can remain
positive and therefore misleading in those who have cleared H. pylori.

· Endoscopy is a consideration to rule out ulcer or upper gastrointestinal malignancy but is


generally preferred after two failed attempts to eradicate H. pylori.

Score:
DBMCI CLT MAINS 2018 755.00 /
Page 135 1200
CLT MAINS - 2018
Exam Title :
(Final)
Email : sasirekha314@gmail.com
Contact : 7708001877

QUESTION 213. MTQ3MStTQVNJUkVLSEEgUCtzYXNpcmVraGEzMTRAZ21haWwuY29tKzc3MDgwMDE4NzdR


lPTiAyMTI=
Three people in a family with colorectal carcinoma are observed to have Lynch
syndrome. They also have which other tumors?

a) Colon, Breast, Endometrium


b) Breast, Ovary, Endometrium
c) Colon, Breast, Ovary
d) Colon, Endometrium, Ovary
Correct Answer: D
Your Answer: B
Explanation

Ans. D. Colon, Endometrium, Ovary

· Hereditary Nonpolyposis Colon Cancer (abbreviated as HNPCC, Lynch Syndrome): HNPCC


is an autosomal dominant inherited syndrome. Errors in the mismatch repair are responsible for
the origination of genetic defects associated with HNPCC.

Score:
DBMCI CLT MAINS 2018 755.00 /
Page 136 1200
CLT MAINS - 2018
Exam Title :
(Final)
Email : sasirekha314@gmail.com
Contact : 7708001877

· Development of Colorectal Carcinoma at an early age is considered as the


characteristic of HNPCC.

HNPCC may also result in extracolonic malignancies including:

• Endometrial, which is most common


• Urinary Tract carcinomas
• Ovarian
• Biliary
• Pancreas
• Stomach
• Small bowel

· Patients who are at risk of developing this, it is indicative to obtain screening colonoscopy
commencing at either age 20 to 25 years or 10 years younger than the youngest age at
diagnosis in the family, whichever comes first.

· Thus, the patients mentioned in the question above may also have Colon, Endometrium and
Ovary tumors.

QUESTION 214. MTQ3MStTQVNJUkVLSEEgUCtzYXNpcmVraGEzMTRAZ21haWwuY29tKzc3MDgwMDE4NzdR


PTiAyMTM=
Commonest type of mesenteric cyst is:

a) Enterogenous
b) Chylolymphatic
c) Dermoid
d) Urogenital remnant
Correct Answer: B
Your Answer: A
Explanation

Ans. B. Chylolymphatic

Mesenteric cysts are classified as

· Chylolymphatic,

· Simple (mesothelial)

· Enterogenous

· Urogenital remnant

· Dermoid (teratomatous cyst)

· Chylolymphatc cyst, the most common mesenteric cyst, arises in congenitally misplaced
lymphatic tissue that has no efferent communication with he lymphatic system; it arises most
frequently in the mesentery of the ileum usually solitary has a blood supply that is independent
from the adjacent intestine so that enucleation is possible without resection of intestine.

Score:
DBMCI CLT MAINS 2018 755.00 /
Page 137 1200
CLT MAINS - 2018
Exam Title :
(Final)
Email : sasirekha314@gmail.com
Contact : 7708001877

QUESTION 215. MTQ3MStTQVNJUkVLSEEgUCtzYXNpcmVraGEzMTRAZ21haWwuY29tKzc3MDgwMDE4NzdR


PTiAyMTQ=
Not a feature of pseudocyesis:

a) Weight gain
b) Morning sickness
c) Enlargement of uterus
d) Amenorrhea
Correct Answer: C
Your Answer: A
Explanation

Ans. C. Enlargement of uterus

Pseudocyesis or phantom pregnancy or pseudopregnancy is a condition in which a patient has


nearly all of the usual signs and symptoms of pregnancy (such as enlargement of the abdomen,
weight gain, cessation of menses, and morning sickness) but is not pregnant. This condition is
usually seen in women who either are very desirous of having children or wishes to avoid
pregnancy. Treatment usually is done by psychiatric means.

QUESTION 216. MTQ3MStTQVNJUkVLSEEgUCtzYXNpcmVraGEzMTRAZ21haWwuY29tKzc3MDgwMDE4NzdR


PTiAyMTU=
“Giant’s roll over” test for Pregnancy Induced Hypertension is done at:

a) 22-24 wks
b) 28-32 wks
c) 24-26 wks
d) 32-34 wks
Correct Answer: B
Your Answer: A
Explanation

Ans. B. 28-32 wks

Roll over test:

This screening test for PIH is done between 28-32 wks. BP is measured with the patient on her
side first and then the patient is asked to roll on her back to check the BP one again. An
increase of 20mHg in diastolic pressure from side to back position indicates a positive “roll over
test”. A negative test is of value.

QUESTION 217. MTQ3MStTQVNJUkVLSEEgUCtzYXNpcmVraGEzMTRAZ21haWwuY29tKzc3MDgwMDE4NzdR


PTiAyMTY=
A 8 month child with diarrhoea, should not be immediately given:

Score:
DBMCI CLT MAINS 2018 755.00 /
Page 138 1200
CLT MAINS - 2018
Exam Title :
(Final)
Email : sasirekha314@gmail.com
Contact : 7708001877

a) ORS
b) Vit. A
c) Calories 175Kcal/kg/d
d) Potassium supplementation for 4 weeks
Correct Answer: C
Your Answer: C
Explanation

Ans. C. Calories 175Kcal/kg/d

This is a child with SAM and so much high calories should not be given in initial phase of
treatment

QUESTION 218. MTQ3MStTQVNJUkVLSEEgUCtzYXNpcmVraGEzMTRAZ21haWwuY29tKzc3MDgwMDE4NzdR


lPTiAyMTc=
Unconjugated hyperbilirubinemia is typical of:

a) Dubin-Johnson syndrome.
b) Hereditary spherocytosis.
c) Rotor's syndrome.
d) biliary atresia
Correct Answer: B
Your Answer: B
Explanation

Ans. B. Hereditary spherocytosis

· Dubin Johnson syndrome is associated with a conjugated hyperbilirubinemia as well as mental


retardation.

Score:
DBMCI CLT MAINS 2018 755.00 /
Page 139 1200
CLT MAINS - 2018
Exam Title :
(Final)
Email : sasirekha314@gmail.com
Contact : 7708001877

· Rotor's syndrome is a rare autosomal recessive condition associated with the inability to
excrete organic anions into bile and is associated with a conjugated hyper bilirubinaemia.

· Biliary atresia is also associated with a conjugated jaundice.

· On the other hand, haemolysis is associated with unconjugated hyper bilirubinaemia as is


Gilbert's disease.

Rotor's syndrome

· Rotor's syndrome is an autosomal recessive condition characterized by a chronic,


predominantly conjugated hyper bilirubinaemia. The mechanism is similar to that seen in the
Dubin-Johnson syndrome, with defective excretion of organic anions into bile.

· The liver is a normal colour on biopsy, unlike Dubin-Johnson syndrome.

· The prognosis is excellent.

QUESTION 219. MTQ3MStTQVNJUkVLSEEgUCtzYXNpcmVraGEzMTRAZ21haWwuY29tKzc3MDgwMDE4NzdR


PTiAyMTg=
Which one of the following drugs should be avoided in wolff-parkinson – white (WPW)
syndrome?

a) Digoxin
b) Adenosine
c) Procainamide
d) Amiodarone
Correct Answer: A
Your Answer: A
Explanation

Ans. A. Digoxin

Pre – Excitation (WPW) Syndrome

Direct connections between atria and ventricle through


Pathophysiology of
kent bundies AB bypass tract can be associated with
wolff-Parkinson- white
certain congenital abnormalities, the most important is
syndrome
Ebstein’s abnormally.

Shortening of the PR interval and a slurring of QRS


ECG
complex called delta (D) wave.

Score:
DBMCI CLT MAINS 2018 755.00 /
Page 140 1200
CLT MAINS - 2018
Exam Title :
(Final)
Email : sasirekha314@gmail.com
Contact : 7708001877

Flecainide, disopyramide or amiodarone

Digoxin and verapamil increase conduction in bypass tract


Treatment and should be avoided

Transvenous catheter radiofrequency ablation of bypass


tract is treatment of choice.

QUESTION 220. MTQ3MStTQVNJUkVLSEEgUCtzYXNpcmVraGEzMTRAZ21haWwuY29tKzc3MDgwMDE4NzdR


PTiAyMTk=
Concerning CNS involvement in AIDS:

a) Toxoplasma may give rise to a focal lesion with neurological weakness.


b) The HIV virus can be isolated from the brain of an encephalopathic patient.
c) A diagnostic elevation in the CSF IgM occurs in toxoplasmic infection.
d) Occular involvement may cause blindness.
Correct Answer: C
Your Answer: Unanswered
Explanation

Ans. C. A diagnostic elevation in the CSF IgM occurs in toxoplasmic infection

· Cerebral toxoplasmosis presents very variably, from an acute encephalopathy to subtle


neurological syndromes. It should be considered in all undiagnosed neurological disease in the
under ones, especially if there are retinal lesions.

· Characteristic are hydrocephalus, seizures with focal defects, spinal or bulbar palsies,
microcephaly, and decreased IQ. Investigations such as skull x-ray or CT scan show calcification
of the periventricular area, tachyzoite in the CSF and positive blood titres.

· Pyrimethamine and Sulphadiazine have a synergistic effect in treating it, and folinic acid may
be necessary to prevent seizures.

QUESTION 221. MTQ3MStTQVNJUkVLSEEgUCtzYXNpcmVraGEzMTRAZ21haWwuY29tKzc3MDgwMDE4NzdR


lPTiAyMjA=
A 40 year old female is on long term treatment for Rheumatoid arthritis. Fundus
examination was done on follow up visit. See image below. What is the most probable
drug responsible for this ?

Score:
DBMCI CLT MAINS 2018 755.00 /
Page 141 1200
CLT MAINS - 2018
Exam Title :
(Final)
Email : sasirekha314@gmail.com
Contact : 7708001877

a) Leflunomide
b) Chloroquine
c) Ibuprofen
d) Methotrexate
Correct Answer: B
Your Answer: B
Explanation

Ans. B. Chloroquine

When high doses of Chloroquine are used for prolonged duration, it can result in Bull’s eye
maculopathy .

EXTRA EDGE

Factors predisposing to chloroquine toxicity:

a) Duration of use more than 5 years

b) Cumulative dose more than 100 grams

c) Daily dose more than 250mg or more than 3mg/kg

QUESTION 222. MTQ3MStTQVNJUkVLSEEgUCtzYXNpcmVraGEzMTRAZ21haWwuY29tKzc3MDgwMDE4NzdR


lPTiAyMjE=
An adult patient presents with leg pain. Investigation reveals a gangrene of his toe. W
hat is the ankle to brachial arterial pressure ratio in this patient ?

a) 1
b) 0.3
c) 0.5
d) 0.8
Correct Answer: B
Your Answer: B
Explanation

Ans. B. 0.3

Score:
DBMCI CLT MAINS 2018 755.00 /
Page 142 1200
CLT MAINS - 2018
Exam Title :
(Final)
Email : sasirekha314@gmail.com
Contact : 7708001877

· As the patient mentioned in the question above has developed gangrene of the toe ankle to
brachial pressure ratio must be less than 0.3.

· In order to ascertain the presence and magnitude of occlusive disease, ratio of ankle to
brachial systolic pressure (ankle brachial index) is obtained.

· The ankle brachial index of 0.9 - 1.0 indicates minimal arterial occlusive disease or normalcy.

· ABI of 0-5 -0.9 is suggestive towards the level of claudication.

· Presence of ischemic rest pain or severe occlusive arterial disease is indicative in case the ABI
is less than 0.5.

· ABI less than 0.3 is compatible with trophic changes over the lower extremity.

QUESTION 223. MTQ3MStTQVNJUkVLSEEgUCtzYXNpcmVraGEzMTRAZ21haWwuY29tKzc3MDgwMDE4NzdR


ElPTiAyMjI=
Staphylococcus aureus endocarditis in intravenous drug abusers:

a) Is less common than Staphylococcus epidermitis endocarditis


b) Preferentially involves the tricuspid valve
c) Should not be treated with valve replacement surgery
d) Is commonly associated with murmurs and peripheral stigmata
Correct Answer: B
Your Answer: B
Explanation

Ans. B. Preferentially involves the tricuspid valve

· Over half the cases of endocarditis in intravenous drug abusers are caused by Staphylococcus
aureus.

· The tricuspid valve is preferentially involved (50% of cases), followed by the aortic (25%) and
mitral (20%) valves.

· Cardiac murmurs and peripheral stigmata are rare with endocarditis in intravenous drug
abusers, especially when the tricuspid valve is involved. Septic and aseptic pulmonary emboli
are common.

· If positive blood cultures persist on antibiotic therapy for the treatment of endocarditis,
surgical valve replacement is necessary in all cases.

QUESTION 224. MTQ3MStTQVNJUkVLSEEgUCtzYXNpcmVraGEzMTRAZ21haWwuY29tKzc3MDgwMDE4NzdR


PTiAyMjM=
Magenta red tongue with cheilosis is caused by deficiency of?

a) Thiamine
b) Riboflavin

Score:
DBMCI CLT MAINS 2018 755.00 /
Page 143 1200
CLT MAINS - 2018
Exam Title :
(Final)
Email : sasirekha314@gmail.com
Contact : 7708001877

c) Cyanocobalamine
d) Ascorobic acid
Correct Answer: B
Your Answer: B
Explanation

Ans. B. Riboflavin

Dietary level per


Contributing
day associated with
Nutrient Clinical finding factors to
overt deficiency in
Deficiency
adults

Magenta tongue,
Riboflavin angular stomatitis, <0.6 mg -
seborrhoea, cheilosis

QUESTION 225. MTQ3MStTQVNJUkVLSEEgUCtzYXNpcmVraGEzMTRAZ21haWwuY29tKzc3MDgwMDE4NzdR


lPTiAyMjQ=
Hypokalemic periodic paralysis is inherited as an autosomal dominant disorder with
incomplete Penetrance; it is resulted because of mutation in :

a) Sodium channel
b) Potassium channel
c) Chrolide channel
d) Calcium channel
Correct Answer: D
Your Answer: B
Explanation

Ans. D. Calcium channel

Hypokalemic Periodic Paralysis

· Onset occurs at adolescence.

· Men are more often affected because of decreased Penetrance in women.

· Hypokalemic periodic paralysis is inherited as an autosomal dominant disorder with


incomplete Penetrance.

· Mutations in the voltage-sensitive, skeletal muscle calcium channel cause the disease.

QUESTION 226. MTQ3MStTQVNJUkVLSEEgUCtzYXNpcmVraGEzMTRAZ21haWwuY29tKzc3MDgwMDE4NzdR


lPTiAyMjU=
Regarding the following type of skin lesion. Which of the following statement is
incorrect?

Score:
DBMCI CLT MAINS 2018 755.00 /
Page 144 1200
CLT MAINS - 2018
Exam Title :
(Final)
Email : sasirekha314@gmail.com
Contact : 7708001877

a) It is related to Seborrhea
b) May be seen in patients with Parkinson’s disease, history of Cerebrovascular accidents
c) Also known as Dandruff if it occurs in the scalp.
d) It may be associated with a yeast-like fungus, Pityrosporum orbiculare.
Correct Answer: A
Your Answer: D
Explanation

Ans. A. It is related to Seborrhea

The patient in question is suffering from Seborrheic Dermatitis.

SEBORRHEIC DERMATITIS

The term covers at least three common patterns of eczema, mainly affecting hairy areas, and
often showing characteristic greasy yellowish scales. These patterns may merge together:

· A red scaly or exudative eruption of the scalp, ears ,face and eyebrows. May be associated
with chronic blepharitis and otitis externa.

· Dry scaly ‘petaloid’ lesions of the presternal and interscapular areas. There may also be
extensive follicular papules or pustules on the trunk (seborrheic folliculitis or Pityrosporum
folliculitis).

· Intertriginous lesions of the armpits, umbilicus or groins, or under spectacles or hearing


aids.

In the first week of life, it typically occurs in the scalp (“cradle cap”), face, or groin.

In adults, it is seen in patients with Parkinson’s disease, CVA and HIV infection.

Most common site is the scalp : recognized as dandruff.

Cause:

Score:
DBMCI CLT MAINS 2018 755.00 /
Page 145 1200
CLT MAINS - 2018
Exam Title :
(Final)
Email : sasirekha314@gmail.com
Contact : 7708001877

· This condition is not obviously related to seborrhoea.

· The success of treatments directed against yeasts has suggested that overgrowth of the
Pityrosporum yeast skin commensal plays an important part in the development of seborrheic
eczema.

Treatment

· Therapy is suppressive rather than curative.

· Topical antifungal, topical steroids, Two per cent sulphur and 2% salicylic acid .

· Medicated shampoo, which may contain ketoconazole, tar, salicylic acid, sulphur, zinc or
selenium sulphide.

· For intertriginous lesions a weak steroid – antiseptic or steroid – antifungal combination is


often effective.

· For severe and unresponsive cases a short course of oral itraconazole may be helpful.

· Leser-Trelat sign: Sudden eruption of multiple seborrheic keratosis lesions in associated


internal malignancy

QUESTION 227. MTQ3MStTQVNJUkVLSEEgUCtzYXNpcmVraGEzMTRAZ21haWwuY29tKzc3MDgwMDE4NzdR


lPTiAyMjY=
Etomidate is characterized by all of the following except:

a) Rapid induction
b) Minimal change in cardiac function
c) Minimal change in respiratory rate
d) Analgesic
Correct Answer: D
Your Answer: A
Explanation

Ans. D. Analgesic

Etomidate

· Rapid induction

· Minimal change in cardiac function

· Minimal change in respiratory rate

· Not analgesic

· Cause pain and myoclonus on injection and nausea postoperatively

· Prolonged administration may cause adrenal suppression.

Score:
DBMCI CLT MAINS 2018 755.00 /
Page 146 1200
CLT MAINS - 2018
Exam Title :
(Final)
Email : sasirekha314@gmail.com
Contact : 7708001877

QUESTION 228. MTQ3MStTQVNJUkVLSEEgUCtzYXNpcmVraGEzMTRAZ21haWwuY29tKzc3MDgwMDE4NzdR


ElPTiAyMjc=
Identify the Instrument?

a) Boyles
b) O2 cylinder
c) Quinkes needle
d) LMA
Correct Answer: C
Your Answer: C
Explanation

Ans. C. Quinkes needle

The Quincke spinal needles have a Luer lock connection on the hub, a medium length cutting
bevel, and a sharp tip. The fitted arrangement of stylet and cannula hub provides proper needle
bevel orientation.

QUESTION 229. MTQ3MStTQVNJUkVLSEEgUCtzYXNpcmVraGEzMTRAZ21haWwuY29tKzc3MDgwMDE4NzdR


lPTiAyMjg=
Darier’s sign is seen in:

a) Xeroderma pigmentosa
b) Urticaria pigmentosa
c) Herpes zoster

Score:
DBMCI CLT MAINS 2018 755.00 /
Page 147 1200
CLT MAINS - 2018
Exam Title :
(Final)
Email : sasirekha314@gmail.com
Contact : 7708001877

d) Glucagonoma
Correct Answer: B
Your Answer: B
Explanation

Ans. B. Urticaria pigmentosa

Urticaria pigmentosa

· It is the most common form of mastocytosis.

· Onset is before 2 yrs.

· Can appear in adults also.

· Generalized and symmetrical eruptions (usually maculopapular) with predilection for trunk.

· Palms and soles tend to be spared and mucous membranes rarely affected.

· Localized or generalized pruritus after exposure to physical stimuli, aggravated by scratching


or rubbing is major complaint.

· Gentle rubbing or stroking of a lesion causes local itching, erythema, and wheal formation
after 2-5 minutes, which may persist for 30 min to several hrs. This is called Darier’s sign

· Darier’s sign is due to local release of histamine from mast cell granules.

· Darier’s sign may also be positive in clinically normal skin (dermographism).

· Rx.

o Antihistaminic

o Aspirin

o Photochemotherapy

o Disodium cromoglycate

o Ketalifen

o Interferons

QUESTION 230. MTQ3MStTQVNJUkVLSEEgUCtzYXNpcmVraGEzMTRAZ21haWwuY29tKzc3MDgwMDE4NzdR


ElPTiAyMjk=
A59-year-old woman presents for the first time with untreated congestive heart failure
(CHF).Urinalysis and urine biochemistry is most likely to show which of the following?

a) Decreased urinary sodium content


b) Low urine specific gravity
c) Increased urinary chloride content
d) Proteinuria
Correct Answer: A

Score:
DBMCI CLT MAINS 2018 755.00 /
Page 148 1200
CLT MAINS - 2018
Exam Title :
(Final)
Email : sasirekha314@gmail.com
Contact : 7708001877
Your Answer: A
Explanation

Ans. A. Decreased urinary sodium content

· High urinary specific gravity, nocturia, and daytime oliguria occur in addition to low urinary
sodium content in untreated CHF.

· These changes are the result of the activation of the renin-angiotensin-aldosterone system.

QUESTION 231. MTQ3MStTQVNJUkVLSEEgUCtzYXNpcmVraGEzMTRAZ21haWwuY29tKzc3MDgwMDE4NzdR


lPTiAyMzA=
Intravenous resuscitation is needed in a patient with 30% burns. Which among the
following denotes the BEST CHOICE of fluid in the first 24 hours?

a) Normal saline
b) Dextrose 5%
c) Ringer lactate
d) Whole blood
Correct Answer: C
Your Answer: C
Explanation

Ans. C. Ringer lactate

· The most commonly used Fluid for Resuscitation in burns is represented by Ringer lactate.

· Some centres use Human Albumin Solution or Fresh-Frozen Plasma while some centres use
Hypertonic saline.

· Intravenous Resuscitation is indicated for children with a burn more than 10% TBSA and
adults with 15% TBSA.

· The most extensively used formula for computing Fluid Replacement in the first 24 hours is
Parkland formula.

Parkland Formula:

· Volume in ml = Total percentage body surface area x weight in kilograms x 4.

· In the initial 8 hours, Half of this volume is given while the second half is given in the
subsequent 16 hours

· Maintenance fluid that is chiefly Dextrose Saline is also given in children.

QUESTION 232. MTQ3MStTQVNJUkVLSEEgUCtzYXNpcmVraGEzMTRAZ21haWwuY29tKzc3MDgwMDE4NzdR


lPTiAyMzE=
Williams ‘syndrome is an autosomal dominant disorder characterized by all of the
following features, except :

Score:
DBMCI CLT MAINS 2018 755.00 /
Page 149 1200
CLT MAINS - 2018
Exam Title :
(Final)
Email : sasirekha314@gmail.com
Contact : 7708001877

a) Hypercalcemia
b) Mental retardation
c) Subvalvular aortic stenosis
d) Elfin facies
Correct Answer: C
Your Answer: C
Explanation

Ans. C. Subvalvular aortic stenosis

William's syndrome or Idiopathic hypercalcemia of infancy is an autosomal dominant disorder


characterized by multiple congenital developmental defects, including supravalvular aortic
stenosis, mental retardation, and an elfin facies, in association with hypercalcemia due to
abnormal sensitivity to vitamin D.

QUESTION 233. MTQ3MStTQVNJUkVLSEEgUCtzYXNpcmVraGEzMTRAZ21haWwuY29tKzc3MDgwMDE4NzdR


ElPTiAyMzI=
A child presents with this hyperpigmented patch over the back since birth. The level of
melanin in this patch on biopsy is likely to be-

a) Epidermal
b) Dermoepidermal junction
c) Superficial dermis
d) Deep dermis
Correct Answer: D
Your Answer: B

Score:
DBMCI CLT MAINS 2018 755.00 /
Page 150 1200
CLT MAINS - 2018
Exam Title :
(Final)
Email : sasirekha314@gmail.com
Contact : 7708001877
Explanation

Ans. D. Deep dermis

Deep dermis. Notice the bluish color which is s/o deep dermal melanin. The lesion is actually a
Mongolian spot.

QUESTION 234. MTQ3MStTQVNJUkVLSEEgUCtzYXNpcmVraGEzMTRAZ21haWwuY29tKzc3MDgwMDE4NzdR


PTiAyMzM=
What is the most common benign tumour of the stomach -

a) Adenoma
b) Lipoma
c) Hamartoma
d) Leiomyoma
Correct Answer: A
Your Answer: D
Explanation

Ans. A. Adenoma

· Epithelial polyps are most common benign tumor of stomach. They are of five types.

· Hyperplastic (most common) type.

· Adenomatous (2nd most common).

· Hamartomatous.

· Inflammatory.

· Heterotopic.

QUESTION 235. MTQ3MStTQVNJUkVLSEEgUCtzYXNpcmVraGEzMTRAZ21haWwuY29tKzc3MDgwMDE4NzdR


PTiAyMzQ=
Which is not the part of active management of 3rd stage of labour?

a) Administration of oxytocin at delivery of anterior shoulder


b) Early cord clamping
c) Controlled cord traction to deliver placenta
d) Uterine massage
Correct Answer: B
Your Answer: B
Explanation

Ans. B. Early cord clamping

· According to WHO 2007 active management of labour include

Score:
DBMCI CLT MAINS 2018 755.00 /
Page 151 1200
CLT MAINS - 2018
Exam Title :
(Final)
Email : sasirekha314@gmail.com
Contact : 7708001877

a) administration of a uterotonic soon after the birth of the baby preferably oxytocin

b) clamping of the cord following observation of a uterine contraction ( at around 3 minutes ie.
Delayed

c) delivery of placenta by controlled cord traction followed by uterine massage.

QUESTION 236. MTQ3MStTQVNJUkVLSEEgUCtzYXNpcmVraGEzMTRAZ21haWwuY29tKzc3MDgwMDE4NzdR


PTiAyMzU=
In cervical incompetence, encerclage operation done are:

a) Mc Donald operation
b) Purandare’s operation
c) Khanna’s sling operation
d) Abdominal sling operation
Correct Answer: A
Your Answer: A
Explanation

Ans. A. Mc Donald operation

Mc Donald cerclage procedure and Shirodkar cerclage procedure are done for managing
incompetent os.

QUESTION 237. MTQ3MStTQVNJUkVLSEEgUCtzYXNpcmVraGEzMTRAZ21haWwuY29tKzc3MDgwMDE4NzdR


lPTiAyMzY=
The principle supports of the uterus are:

a) The iliosacral ligaments


b) The pyriformis muscle
c) The transverse cervical ligaments
d) The infundibular ligaments
Correct Answer: C
Your Answer: C
Explanation

Ans. C. The transverse cervical ligaments

· The principle supports of the uterus are the transverse cervical ligaments (cardinal ligaments),
uterosacral ligaments and the round ligament.

· The infundibular ligaments attach the ovaries to the posteo-lateral wall of the uterus.

· The pyriformis muscle lines the lateral wall of the pelvis overlying the iliosacral ligament.

Score:
DBMCI CLT MAINS 2018 755.00 /
Page 152 1200
CLT MAINS - 2018
Exam Title :
(Final)
Email : sasirekha314@gmail.com
Contact : 7708001877
QUESTION 238. MTQ3MStTQVNJUkVLSEEgUCtzYXNpcmVraGEzMTRAZ21haWwuY29tKzc3MDgwMDE4NzdR
lPTiAyMzc=
A Patient presented with Fever and cough. Most likely diagnosis is ?

a) Right upper lobe consolidation


b) Right middle lobe consolidation
c) Left upper lobe consolidation
d) Lingular consolidation
Correct Answer: B
Your Answer: B
Explanation

Ans. B. Right middle lobe consolidation

If you watch carefully you will see a ill defined haziness in the immediate right paracardiac
region.

This haziness is obscuring the right heart border and hence is suggestive of right middle lobe
consolidation. This question is a direct application of the Silhouette sign – Most important sign
in chest radiology.

QUESTION 239. MTQ3MStTQVNJUkVLSEEgUCtzYXNpcmVraGEzMTRAZ21haWwuY29tKzc3MDgwMDE4NzdR


lPTiAyMzg=
Which of the following is investigation of choice for Small intestinal tumor:

a) Barium meal follow through


b) USG
c) Barium Enteroclysis
d) CT Enterography
Correct Answer: D
Your Answer: D
Explanation

Score:
DBMCI CLT MAINS 2018 755.00 /
Page 153 1200
CLT MAINS - 2018
Exam Title :
(Final)
Email : sasirekha314@gmail.com
Contact : 7708001877

Ans. D. CT Enterography

· Enteroclysis is not more widely used because of perceived patient discomfort potentially high
radiation dose and extraluminal pathology is not assessable.

· USG has a limited role in the diagnosis and management of disorders of the small intestine.

· CT (IV + Oral contrast) show the extent of spread of the mass and evaluate sites of neoplastic
involvement like abdominal lymph nodes and solid organs, allowing accurate staging at a single
examination. Bowel distension with oral mannitol (CT enterography) has further enhanced the
evaluation of small bowel pathology.

QUESTION 240. MTQ3MStTQVNJUkVLSEEgUCtzYXNpcmVraGEzMTRAZ21haWwuY29tKzc3MDgwMDE4NzdR


lPTiAyMzk=
Regarding torsades de pointes, false statement is:

a) More Common in males


b) Hypocalcemia can be a causative false
c) IV.mg is initial treatment of choice for acquired cause.
d) Congenital cause is treated with beta blocker
Correct Answer: A
Your Answer: D
Explanation

Ans. A. More Common in males

CAUSES OF TORSADES DE POINTES:

I. Congenital

II Acquired : Bradycardia

Hypokalemia

Drugs- Quinidine , procainamide, sotalol, amiodarone

Disopyramide, phenothiazine, CA

Score:
DBMCI CLT MAINS 2018 755.00 /
Page 154 1200
CLT MAINS - 2018
Exam Title :
(Final)
Email : sasirekha314@gmail.com
Contact : 7708001877

Torsades de pointes is more common in females ( because of ↑QT)

Mechanism – Initiation by early after depolarisation

- Perpetuation by transmural entery

Treatment-

I. Congenital: Betablocker, pacing, IC

II Acquired: I.V Magnesium, pacing

QUESTION 241. MTQ3MStTQVNJUkVLSEEgUCtzYXNpcmVraGEzMTRAZ21haWwuY29tKzc3MDgwMDE4NzdR


PTiAyNDA=
Which of the following conditions disappear spontaneously in the first year of life :

a) Port wine stain


b) Naevus flammeus
c) Salmon ‘s patch
d) Strawberry haemangioma
Correct Answer: C
Your Answer: D
Explanation

Ans. C. Salmon ‘s patch

SALMON PATCH:

· Also k/a macular stain or stork bite

· Present at birth

· Seen over forehead in the midline and over the occiput. Disappear by the age of 1 year

QUESTION 242. MTQ3MStTQVNJUkVLSEEgUCtzYXNpcmVraGEzMTRAZ21haWwuY29tKzc3MDgwMDE4NzdR


PTiAyNDE=
A patient is complaining of nasal blockage on examination bilateral nasal polyps are
seen. C.T SCAN of PNS shows following Image. Diagnosis is:

Score:
DBMCI CLT MAINS 2018 755.00 /
Page 155 1200
CLT MAINS - 2018
Exam Title :
(Final)
Email : sasirekha314@gmail.com
Contact : 7708001877

a) Inverted papilloma
b) Tumour of maxillary sinus
c) Allergic fungal sinusitis
d) Nasal spur
Correct Answer: C
Your Answer: C
Explanation

Ans. C. Allergic fungal sinusitis

CT images demonstrate severe, expansile opacification of multiple sinuses bilaterally as well as


involvement of the nasal cavity. There is high density (bright) mucus within the sinuses. These
features suggest allergic fungal Rhinosinusitis.

QUESTION 243. MTQ3MStTQVNJUkVLSEEgUCtzYXNpcmVraGEzMTRAZ21haWwuY29tKzc3MDgwMDE4NzdR


lPTiAyNDI=
True About Bera Finding In Acoustic Neuroma:

a) Latency of wave 1 is increased in effected ear


b) Latency of wave 5 is increased in effected wave
c) No wave formation in BERA
d) No change in BERA
Correct Answer: B
Your Answer: C
Explanation

Score:
DBMCI CLT MAINS 2018 755.00 /
Page 156 1200
CLT MAINS - 2018
Exam Title :
(Final)
Email : sasirekha314@gmail.com
Contact : 7708001877

Ans. B. Latency of wave 5 is increased in effected wave

· In acoustic neuroma: Latency of wave 5 is increased in effected wave

· Increased interaural latency of wave 5 of >0.2 msec,: difference of latency of wave 5 in the
two ears is >0.2 msec suggests retrocochlear pathology. this is most diagnostic finding in
acoustic neuroma

QUESTION 244. MTQ3MStTQVNJUkVLSEEgUCtzYXNpcmVraGEzMTRAZ21haWwuY29tKzc3MDgwMDE4NzdR


PTiAyNDM=
Which of the following serves as the MOST DIAGNOSTIC sign of Pregnancy?

a) Amenorrhoea
b) Quickening
c) Foetal heart sounds
d) Distention of abdomen
Correct Answer: C
Your Answer: C
Explanation

Ans. C. Foetal heart sounds

Presence of fetal heart sounds is considered to be one of the most diagnostic sign of pregnancy.

Other absolute signs of pregnancy are mentioned as follows:

· Perception of active fetal movements and Palpation of fetal parts.

· Ultrasonographic evidence of gestational sac and embryo in early pregnancy and even later.

Probable signs:

· Osiander’s sign (increased pulsation felt through lateral fornix by 8 weeks), Chadwick’s sign
(Blue hue of vagina), and Goodell sign (softening of cervix)

· Enlargement of the abdomen.

· Change in shape, size as well as consistency of uterus

· Softening of cervix

· Internal as well as external ballottement

· Contraction of Braxton Hicks

QUESTION 245. MTQ3MStTQVNJUkVLSEEgUCtzYXNpcmVraGEzMTRAZ21haWwuY29tKzc3MDgwMDE4NzdR


PTiAyNDQ=
A woman comes to Gyne OPD with 6 week gestation for check up. She gives history a
molar pregnancy evacuation 1 year ago, the records for which are not available. She is
counciled all except?

Score:
DBMCI CLT MAINS 2018 755.00 /
Page 157 1200
CLT MAINS - 2018
Exam Title :
(Final)
Email : sasirekha314@gmail.com
Contact : 7708001877

a) Risk of developing trophoblastic disease in this pregnancy is 10%


b) Early sonographic evaluation is recommended
c) Placenta or products of conception are sent for pathological evaluation
d) bHCG level is measured 6 wk postpartum
Correct Answer: A
Your Answer: D
Explanation

Ans. A. Risk of developing trophoblastic disease in this pregnancy is 10%

Women with prior gestational trophoblastic disease or successfully treated neoplasia usually do
not have impaired fertility and their pregnancy outcomes are usually normal. The primary
concern in these women is their 2-percent risk for developing trophoblastic disease in a
subsequent pregnancy. Sonographic evaluation is recommended in early pregnancy, and
subsequently if indicated. At delivery, the placenta or products of conception are sent for
pathological evaluation, and a serum hCG level is measured 6 weeks post- partum.

QUESTION 246. MTQ3MStTQVNJUkVLSEEgUCtzYXNpcmVraGEzMTRAZ21haWwuY29tKzc3MDgwMDE4NzdR


PTiAyNDU=
Which is the deformity shown in the image?

a) Prayer sign
b) Sclerodactyly
c) Boutonniere deformity
d) Arachnodactyly
Correct Answer: C
Your Answer: C
Explanation

Ans. C. Boutonniere deformity

The Image shows flexion at PIP and extension at DIP, which is suggestive of process of
boutonniere deformity. It is seen in RA.

Score:
DBMCI CLT MAINS 2018 755.00 /
Page 158 1200
CLT MAINS - 2018
Exam Title :
(Final)
Email : sasirekha314@gmail.com
Contact : 7708001877
QUESTION 247. MTQ3MStTQVNJUkVLSEEgUCtzYXNpcmVraGEzMTRAZ21haWwuY29tKzc3MDgwMDE4NzdR
lPTiAyNDY=
In a case of ovarian tumor revealed unilateral ovarian tumor with ascites positive for
malignant cells. Laparotomy is performed. All other structures were free of disease.
Stage of her disease is:

a) Stage I c
b) Stage III a
c) Stage III b
d) Stage III c
Correct Answer: A
Your Answer: A
Explanation

Ans. A. Stage I c

The patient falls under FIGO staging, stage Ic because it has ovarian tumor with ascites positive
for malignant cells.

QUESTION 248. MTQ3MStTQVNJUkVLSEEgUCtzYXNpcmVraGEzMTRAZ21haWwuY29tKzc3MDgwMDE4NzdR


lPTiAyNDc=
MC Heart disease seen in the state shown in image:

a) Mitral Stenosis

Score:
DBMCI CLT MAINS 2018 755.00 /
Page 159 1200
CLT MAINS - 2018
Exam Title :
(Final)
Email : sasirekha314@gmail.com
Contact : 7708001877

b) Mitral regurgitation
c) Aortic stenosis
d) Aortic regurgitation
Correct Answer: A
Your Answer: A
Explanation

Ans. A. Mitral Stenosis

· It is the commonest heart lesion met during pregnancy.

· Normal mitral valve area ranges between 4 and 6 cm2.

· Symptoms usually appear when stenosis narrows this to less than 2.5 cm2.

· Women with mitral valve area < 1cm2 have the high rate of pulmonary edema (55%) and
arrhythmia (33%).

· In asymptomatic cases, the mortality is < 1% but once it is significantly symptomatic,


mortality ranges between 5% and 15%.

QUESTION 249. MTQ3MStTQVNJUkVLSEEgUCtzYXNpcmVraGEzMTRAZ21haWwuY29tKzc3MDgwMDE4NzdR


lPTiAyNDg=
True about lunate dislocation?

a) Lunate dislocates anteriorly, while other carpal bones remains in place.


b) Lunate stays in place while other carpal bones dislocate posteriorly.
c) Most common to be involved is ulnar nerve
d) None
Correct Answer: A
Your Answer: A
Explanation

Ans. A. Lunate dislocates anteriorly, while other carpal bones remains in place.

Wrist dislocations

Types

· Lunate dislocation – lunate dislocates anteriorly, while other carpal bones remains in place.

· Perilunate dislocation – lunate stays in place while other carpal bones dislocate posteriorly.

Perilunate dislocation is the more common type, & when it is associated with fracture scaphoid,
it is known as trans scaphoid perilunate fracture dislocation. Median nerve is the M/C nerve
injury in lunate/ perilunate dislocation.

QUESTION 250. MTQ3MStTQVNJUkVLSEEgUCtzYXNpcmVraGEzMTRAZ21haWwuY29tKzc3MDgwMDE4NzdR


lPTiAyNDk=

Score:
DBMCI CLT MAINS 2018 755.00 /
Page 160 1200
CLT MAINS - 2018
Exam Title :
(Final)
Email : sasirekha314@gmail.com
Contact : 7708001877

Amyloidosis deposition most commonly occurs in :

a) Renal vessels
b) Knee joints
c) Skin
d) Cornea
Correct Answer: A
Your Answer: A
Explanation

Ans. A. Renal vessels

· Amyloid is abnormal pathological protein resulting from improper folding of proteins and
deposited extracellularly as fibrils and disrupt their normal function.

· The most common site of deposition is kidney, followed by, liver, spleen, lymph nodes,
adrenals, thyroid, GIT, heart etc.

· In systemic amyloidosis, any organ can show deposition of Amyloid. Among all organs, kidneys
are most commonly and most seriously get affected by Amyloidosis.

QUESTION 251. MTQ3MStTQVNJUkVLSEEgUCtzYXNpcmVraGEzMTRAZ21haWwuY29tKzc3MDgwMDE4NzdR


lPTiAyNTA=
False statement about Thyroglossal cyst is?

a) Remnant of 2nd branchial arch


b) May contain total thyroid tissue in 10% cases
c) Papillary cancer be found in the cyst
d) Attached to base of tongue with a tract that goes through body of hyoid
Correct Answer: A
Your Answer: A
Explanation

Ans. A. Remnant of 2nd branchial arch

· A thryoglossal duct cyst is the most common of the clinically important anomalies of thyroid
development

Thyroglossal duct cysts

· Both cysts and sinus tracts can develop along the course of the thyroglossal duct.

· Normally the thyroglossal duct becomes obliterated early in embryonic life but occasionally it
may persist as a cyst or a draining sinus tract.

· Cysts often become infected and may rupture spontaneously

· Removal of the cyst or sinus usually requires excision of the central part of the hyoid bone and
dissection of the tract to the base of the tongue (the sistrunk operation), if recurrence is to be
minimized.

Thyroglossal fistula

Score:
DBMCI CLT MAINS 2018 755.00 /
Page 161 1200
CLT MAINS - 2018
Exam Title :
(Final)
Email : sasirekha314@gmail.com
Contact : 7708001877

· It is never congenital

· It follows infection as inadequate removal of thyroglossal cyst.

· It follows infection as inadequate removal of thyroglossal cyst

Treatment:

· As thyroglossal tract is closely related to the body of hyoid bone, the central part must be
excised together with the cyst or fistula (Sistrunk operation)

QUESTION 252. MTQ3MStTQVNJUkVLSEEgUCtzYXNpcmVraGEzMTRAZ21haWwuY29tKzc3MDgwMDE4NzdR


lPTiAyNTE=
False regarding Hutchinson syndrome:

a) Seen in congenital syphilis


b) Child is irritable
c) Skeletal system is involved
d) None of the above
Correct Answer: A
Your Answer: Unanswered
Explanation

Ans. A. Seen in congenital syphilis

Hutchinson syndrome is seen in Neuroblastoma. In this , there is irritability and tenderness due
to involvement of skeletal system .

QUESTION 253. MTQ3MStTQVNJUkVLSEEgUCtzYXNpcmVraGEzMTRAZ21haWwuY29tKzc3MDgwMDE4NzdR


ElPTiAyNTI=
All are seen in below condition except:

Score:
DBMCI CLT MAINS 2018 755.00 /
Page 162 1200
CLT MAINS - 2018
Exam Title :
(Final)
Email : sasirekha314@gmail.com
Contact : 7708001877

a) Glaucoma
b) Hemiparesis
c) MR
d) CT head shows candle wax dripping appearance
Correct Answer: D
Your Answer: D
Explanation

Ans. D. CT head shows candle wax dripping appearance

The Image represents Sturge Weber syndrome. CT head shows tram track appearance in this
syndrome .Candle wax dripping appearance is seen in Tuberous sclerosis.

QUESTION 254. MTQ3MStTQVNJUkVLSEEgUCtzYXNpcmVraGEzMTRAZ21haWwuY29tKzc3MDgwMDE4NzdR


PTiAyNTM=
According to WHO classification of ovarian tumours Brenner tumor of ovary belongs
to:-

a) Epithelial tumours
b) Sex cord stromal tumours
c) Germ cell tumours
d) Metastatic tumours
Correct Answer: A
Your Answer: A
Explanation

Ans. A. Epithelial tumours

Cancer or ovary

Ovarian cancer Arises from Types Age group

Serous, mucinous
endometrioid clear Peri/postmenopausal
Epithelial Coelomic epithelium
cell Brenner (45+year)
undifferentiated

Granulosa cell tumor, Reproductive (20-40


Sex cord stromal Gonadal stromal
sertoil-leyding tumor years)

Dysgerminoma,
Prepuberal pubertal
Germs cell tumours Primitive germ cells endodermal teratoma
(15-20 years)
chorio carcinoma

Others Metastatic Krukenberg

Score:
DBMCI CLT MAINS 2018 755.00 /
Page 163 1200
CLT MAINS - 2018
Exam Title :
(Final)
Email : sasirekha314@gmail.com
Contact : 7708001877

QUESTION 255. MTQ3MStTQVNJUkVLSEEgUCtzYXNpcmVraGEzMTRAZ21haWwuY29tKzc3MDgwMDE4NzdR


lPTiAyNTQ=
Which endocrine disorder is associated with Epiphyseal dysgenesis : -

a) Hypothyroidism
b) Cushing’s syndrome
c) Addison’s disease
d) Hypoparathyroidism
Correct Answer: A
Your Answer: A
Explanation

Ans. A. Hypothyroidism

FEATURES OF CHILDHOOD HYPOTHYROIDISM (CRETINISM)

· Delayed skeletal maturation

· Fragmented stippled epiphyses, i.e., Epiphyseal dysgenesis

· Delayed dentition

· Delayed/decreased pneumatization of sinuses and mastoids

· Dense vertebral margins

· Demineralization

· Hypertelorism

· Hypoplastic phalanges of 5th finger

· Wide fontanels/sutures with delayed closure

QUESTION 256. MTQ3MStTQVNJUkVLSEEgUCtzYXNpcmVraGEzMTRAZ21haWwuY29tKzc3MDgwMDE4NzdR


lPTiAyNTU=
True colour of cholesterol stone is?

a) Black
b) Brown
c) Dark yellow
d) Pale yellow
Correct Answer: D
Your Answer: D
Explanation

Ans. D. Pale yellow

MORPHOLOGY OF GALL STONES

Score:
DBMCI CLT MAINS 2018 755.00 /
Page 164 1200
CLT MAINS - 2018
Exam Title :
(Final)
Email : sasirekha314@gmail.com
Contact : 7708001877

Cholesterol stones:

· Cholesterol stones arise exclusively in the gallbladder and are composed of cholesterol,
ranging from 100% pure (which is rare) down to around 50% Pure cholesterol stones are pale
yellow, round to ovoid, and have a finely granular, hard external surface which on transaction
reveals a glistening radiating crystalline palisade.

· Stones composed largely of cholesterol are radiolucent; sufficient calcium carbonate is found
in 10% to 20% of cholesterol stones of render them radiopaque

QUESTION 257. MTQ3MStTQVNJUkVLSEEgUCtzYXNpcmVraGEzMTRAZ21haWwuY29tKzc3MDgwMDE4NzdR


lPTiAyNTY=
Histo-pathological finding shown in the Image. What is the most likely causative agent
?

a) Human papilloma virus


b) Pox virus
c) Coxsackie virus
d) Parvovirus
Correct Answer: B
Your Answer: Unanswered
Explanation

Ans. B. Pox virus

· Pox virus is causative organism for Molluscum contagiosum.

Score:
DBMCI CLT MAINS 2018 755.00 /
Page 165 1200
CLT MAINS - 2018
Exam Title :
(Final)
Email : sasirekha314@gmail.com
Contact : 7708001877

· Numerous intracytoplasmic, eosinophilic inclusion bodies, so-called handerson Paterson


(molluscum) bodies, can be seen forming in the lower epidermis. They increase in size as they
move toward the surface.

QUESTION 258. MTQ3MStTQVNJUkVLSEEgUCtzYXNpcmVraGEzMTRAZ21haWwuY29tKzc3MDgwMDE4NzdR


ElPTiAyNTc=
Vaginal epithelium is derived from:

a) Endoderm of urogenital sinus


b) Mesoderm of urogenital sinus
c) Endoderm of genital ridge
d) Mesoderm of genital ridge
Correct Answer: A
Your Answer: A
Explanation

Ans. A. Endoderm of urogenital sinus

Vagina is developed mainly from the Mullerian ducts and party from the urogenital sinus.

Upper 1/3 of vagina develops from the fused uterovaginal canal of the Mullerian ducts.

· Mucous membrane is developed from the endoderm of the canalized (vaginal plate)
sinovaginal bulb(urogenital sinus).

· The musculature is developed from the mesoderm of the fused caudal vertical part of
the Mullerian ducts.

· The hymen is developed from the junction of the Mullerian tubercle (mesodermal) and the
urogenital sinus (endodermal).

· Lower 2/3 is developed entirely from the endoderm of the urogenital sinus.

· Vaginal introitus is developed from the ectoderm of the genital folds after rupture of the
bilaminar urogenital membrane

QUESTION 259. MTQ3MStTQVNJUkVLSEEgUCtzYXNpcmVraGEzMTRAZ21haWwuY29tKzc3MDgwMDE4NzdR


lPTiAyNTg=
Bochdalek hernia is seen through?

a) Mediastinum
b) Urogenital diaphragm
c) Diaphragm
d) Dura
Correct Answer: C
Your Answer: C
Explanation

Score:
DBMCI CLT MAINS 2018 755.00 /
Page 166 1200
CLT MAINS - 2018
Exam Title :
(Final)
Email : sasirekha314@gmail.com
Contact : 7708001877

Ans. C. Diaphragm

· In infants with Bochdalek type congenital diaphragmatic hernia the severity of pulmonary
hypoplasia and the resultant pulmonary hypertension are key determinants of survival.

· Barotraumas and hypoxia should be avoided.

QUESTION 260. MTQ3MStTQVNJUkVLSEEgUCtzYXNpcmVraGEzMTRAZ21haWwuY29tKzc3MDgwMDE4NzdR


lPTiAyNTk=
The most common electrolyte abnormality found in patients with normal anion gap
metabolic acidosis is?

a) Hyperchloremia
b) Hypochloremia
c) Hyperkalemia
d) Hypokalemia
Correct Answer: A
Your Answer: A
Explanation

Ans. A. Hyperchloremia.

In an normal anion gap metabolic acidosis there is by definition a rise in the serum chloride
equivalent to the fall in the serum bicarbonate such that the anion gap (Na - Cl - HCO3) remains
unchanged.

QUESTION 261. MTQ3MStTQVNJUkVLSEEgUCtzYXNpcmVraGEzMTRAZ21haWwuY29tKzc3MDgwMDE4NzdR


ElPTiAyNjA=
Which of the following are true except?

a) Surgical blade is kept over lower lateral cartilage

Score:
DBMCI CLT MAINS 2018 755.00 /
Page 167 1200
CLT MAINS - 2018
Exam Title :
(Final)
Email : sasirekha314@gmail.com
Contact : 7708001877

b) Surgery is rhinoplasty
c) Columella of nose is formed by medial crus of both lower lateral cartilage and septa
cartilage
d) Surgery is extensive septoplasty
Correct Answer: D
Your Answer: C
Explanation

Ans. D. Surgery is extensive septoplasty

Image is of rhinoplasty

QUESTION 262. MTQ3MStTQVNJUkVLSEEgUCtzYXNpcmVraGEzMTRAZ21haWwuY29tKzc3MDgwMDE4NzdR


ElPTiAyNjE=
In which of the following conditions, cortical mastoidectomy is indicated:

a) Malignant disease of middle ear


b) CSOM without cholesteatoma
c) Acute coalescent mastoiditis
d) CSOM with cholesteatoma
Correct Answer: C
Your Answer: C
Explanation

Ans. C. Acute coalescent mastoiditis

For all type of mastoiditis surgery of choice is cortical mastoidectomy.

QUESTION 263. MTQ3MStTQVNJUkVLSEEgUCtzYXNpcmVraGEzMTRAZ21haWwuY29tKzc3MDgwMDE4NzdR


ElPTiAyNjI=
Which of the following is not the new entity added in the proposed ICD-11?

a) Olfactory reference disorder


b) Compulsive sexual behaviour disorder
c) Complex post-traumatic stress disorder
d) Schizophreniform disorder
Correct Answer: D
Your Answer: Unanswered
Explanation

Ans. D. Schizophreniform disorder

· Olfactory Reference Disorder is characterized by persistent preoccupation with the belief that
one is emitting a perceived foul or offensive body odour or breath that is either unnoticeable or
only slightly noticeable to others.

Score:
DBMCI CLT MAINS 2018 755.00 /
Page 168 1200
CLT MAINS - 2018
Exam Title :
(Final)
Email : sasirekha314@gmail.com
Contact : 7708001877

· Individuals experience excessive self-consciousness about the perceived odour, often with
ideas of reference (i.e., the conviction that people are taking notice, judging, or talking about
the odour).

QUESTION 264. MTQ3MStTQVNJUkVLSEEgUCtzYXNpcmVraGEzMTRAZ21haWwuY29tKzc3MDgwMDE4NzdR


lPTiAyNjM=
Which treatment is most likely to result in a complete remission in seriously ill
patients with Thrombotic Thrombocytopenic Purpura:

a) Methylprednisolone
b) Dipyridamole
c) Vincristine
d) Plasmapheresis
Correct Answer: D
Your Answer: D
Explanation

Ans. D. Plasmapheresis

· Today, the treatment of Thrombotic Thrombocytopenic Purpura involves a daily single plasma
volume exchange (plasmapheresis), which is continued until the platelet count normalizes and
there is minimal hemolysis.

· Seriously ill patients may receive fresh frozen plasma infusions (without exchange) for several
hours until plasmapheresis can be arranged.

· The plasma exchange must sometimes be continued for 10 days or longer to obtain a complete
remission in seriously ill patients - Thrombotic Thrombocytopenic Purpura Treatment is first
tapered and later discontinued if normal laboratory parameters are maintained.

QUESTION 265. MTQ3MStTQVNJUkVLSEEgUCtzYXNpcmVraGEzMTRAZ21haWwuY29tKzc3MDgwMDE4NzdR


ElPTiAyNjQ=
The following test is administered to assess:

a) Maturation levels in children


b) Organic dysfunction

Score:
DBMCI CLT MAINS 2018 755.00 /
Page 169 1200
CLT MAINS - 2018
Exam Title :
(Final)
Email : sasirekha314@gmail.com
Contact : 7708001877

c) Visual and motor coordination


d) All of the above
Correct Answer: D
Your Answer: C
Explanation

Ans. D. All of the above

· The Bender Visual Motor Gestalt Test, devised by the American neuropsychiatrist Lauretta
Bender in 1938, is a technique that consists of nine figures that are copied by the subject.

· It is administered as a means of evaluating maturation levels in children and organic


dysfunction. Its chief applications are to determine retardation, loss of function, and organic
brain defects in children and adults. The designs are presented one at a time to the subject,
whose asked to copy them onto a sheet of paper.

· The subject is then asked to copy the designs from memory, thus, the Bender designs can be
used as a test of both visual and motor coordination and immediate visual memory.

QUESTION 266. MTQ3MStTQVNJUkVLSEEgUCtzYXNpcmVraGEzMTRAZ21haWwuY29tKzc3MDgwMDE4NzdR


ElPTiAyNjU=
Most common type of porphyria is :

a) Porphyria Cutanea Tarda


b) Acute Intermittent Porphyria
c) Erythropoietic porphyria
d) Hereditary Coproporphyria
Correct Answer: A
Your Answer: A
Explanation

Ans. A. Porphyria Cutanea Tarda

· Porphyrias are basically classified into Hepatic and Erythropoietic depending on whether
enzyme deficiency occurs in liver or Bone marrow.

· In Porphyria Cutanea Tarda (the commonest type of porphyria), there is hepatic and bone
marrow involvement, though it is classified as a hepatic porphyria.

· Erythropoietic porphyrias present with cutaneous features, ulcers, photosensitivity.

· Hepatic Porphyrias presents with neuropsychiatric features.

· Photosensitivity in porphyrias is as a result of porphyrin mediated formation of superoxide


radicals from O2.

QUESTION 267. MTQ3MStTQVNJUkVLSEEgUCtzYXNpcmVraGEzMTRAZ21haWwuY29tKzc3MDgwMDE4NzdR


ElPTiAyNjY=
In diabetic foot, which amputation is contraindicated?

Score:
DBMCI CLT MAINS 2018 755.00 /
Page 170 1200
CLT MAINS - 2018
Exam Title :
(Final)
Email : sasirekha314@gmail.com
Contact : 7708001877

a) Ray amputation
b) Forefoot amputation
c) Syme’s amputation
d) Below knee amputation
Correct Answer: C
Your Answer: C
Explanation

Ans. C. Syme’s amputation

· For any amputation to heal the local residual stump needs good vascularity, in Syme’s
amputation it is essential to preserve the blood supply to the heel flap by meticulous clean
dissection of the calcaneus.

· The tibia and fibula are sectioned as low as possible to the top of the joint.

· This type of procedure is rarely suitable for severely ischemic athermanous limbs because of
the poor healing of the heel flap.

· Hence not suitable for diabetic foot so it is contraindicated.

QUESTION 268. MTQ3MStTQVNJUkVLSEEgUCtzYXNpcmVraGEzMTRAZ21haWwuY29tKzc3MDgwMDE4NzdR


ElPTiAyNjc=
Risk of subsequent preterm delivery with previous two preterm deliveries at < 34
weeks is?

a) 15%
b) 20%
c) 30%
d) 40%
Correct Answer: D
Your Answer: Unanswered
Explanation

Ans. D. 40%

Recurrent spontaneous preterm births According to prior outcome

Second Birth
Birth outcome
≤34 weeks (%)

First birth ≥35 weeks 5

First birth ≤ 34 weeks 16

First and second birth ≤ 34 weeks 41

Score:
DBMCI CLT MAINS 2018 755.00 /
Page 171 1200
CLT MAINS - 2018
Exam Title :
(Final)
Email : sasirekha314@gmail.com
Contact : 7708001877

QUESTION 269. MTQ3MStTQVNJUkVLSEEgUCtzYXNpcmVraGEzMTRAZ21haWwuY29tKzc3MDgwMDE4NzdR


ElPTiAyNjg=
Sentinel lymph node biopsy is most useful for which of the following gynecological
malignancies:

a) Carcinoma endometrium
b) Carcinoma cervix
c) Carcinoma vulva
d) Carcinoma vagina
Correct Answer: C
Your Answer: C
Explanation

Ans. C. Carcinoma vulva

Vulvar carcinoma is the most promising site for sentinel lymph node biopsy strategy. Studies
suggest that a sentinel node can be identified in most patients. Sentinel node is always located
in the groin making vulva an ideal site for sentinel node biopsy'. 'The detection of SLN is
currently a standard component of the surgical treatment of malignant melanoma, breast
cancer and is a promising staging technique for patients with vulvar cancer. In cervical cancer
SLN mapping is more recent and preliminary studies indicate that it may be a feasible
technique' - 'Trends in Cervical Cancer' by Varaj Sentinel Lymph Node (SLN) Biopsy for
Gynaecological Malignancies

a. The sentinel lymph node is defined as the first draining lymph node to receive lymphatic
drainage from the tumor site. A negative sentinel lymph node would thus predict the absence of
lymph node metastasis in the entire draining basin.

b. Sentinel lymph node evaluation in female genital tract cancers is relatively new and a
number of promising studies are under evaluation.

c. The concept has shown most promising results in vulvar cancer followed by cervical cancer.

d. (SLN is however not yet accepted as a standard procedure for either of these). For
endometrial cancer the concept of sentinel lymph node biopsy has been studied by small studies
and requires further larger studies. Results from current studies are favorable but not to the
same extent as vulvar and cervical cancers.

e. For vaginal cancer the concept of sentient lymph node biopsy is very new and studies are
limited mainly to case reports.

QUESTION 270. MTQ3MStTQVNJUkVLSEEgUCtzYXNpcmVraGEzMTRAZ21haWwuY29tKzc3MDgwMDE4NzdR


ElPTiAyNjk=
Which response is not seen in newborns as a measure of Thermogenesis?

a) Shivering
b) Breakdown of brown fat
c) Universal flexion

Score:
DBMCI CLT MAINS 2018 755.00 /
Page 172 1200
CLT MAINS - 2018
Exam Title :
(Final)
Email : sasirekha314@gmail.com
Contact : 7708001877

d) Cutaneous vasoconstrictions
Correct Answer: A
Your Answer: A
Explanation

Ans. A. Shivering

· “Newborn cannot produce heat by shivering.

· Mechanisms for Thermogenesis in new born are mainly due to rapid increase in circulating
catecholamines due to cold stress

· Lipolysis and re-esterification of brown fat

· Peripheral vasoconstriction

· Increase heart rate.

QUESTION 271. MTQ3MStTQVNJUkVLSEEgUCtzYXNpcmVraGEzMTRAZ21haWwuY29tKzc3MDgwMDE4NzdR


lPTiAyNzA=
A 34-week female fetus was born as shown below. Identify the congenital anomaly:

a) Anencephaly
b) Iniencephaly
c) Aneniniencephaly
d) Complete craniospinal rachischisis
Correct Answer: B
Your Answer: A
Explanation

Ans. B. Iniencephaly

Score:
DBMCI CLT MAINS 2018 755.00 /
Page 173 1200
CLT MAINS - 2018
Exam Title :
(Final)
Email : sasirekha314@gmail.com
Contact : 7708001877

A rare neural tube defect characterized by extreme retroflexion of the head with the absence of
neck due to spinal deformities.

QUESTION 272. MTQ3MStTQVNJUkVLSEEgUCtzYXNpcmVraGEzMTRAZ21haWwuY29tKzc3MDgwMDE4NzdR


lPTiAyNzE=
Cardiotoxicity induced by Doxorubicin can be reduced by :-

a) Dexrazoxane
b) Amifostine
c) Leucovorin
d) Bisphosphonates
Correct Answer: A
Your Answer: A
Explanation

Ans. A. Dexrazoxane

· Cardiotoxicity induced by Doxorubicin is an important side effect which can be modified by the
use of Dexrazoxane.

· Dexrazoxane : Iron chelator that decreases free radical formation ~ Protects myocardium from
anthracycline induced injury.

· Amifostine Radio protective agent (Reduces renal toxicity caused by cisplatin) Leucovorin:
Reduces bone marrow & mucosal toxicity caused by methotrexate other measures [(Vigorous
hydration, bicarbonate loading) Asparaginase, Carboxypeptidase G2] Mesna: Reduces bladder
toxicity caused by Cyclophosphamide.

· Bisphosphonate: Reduces bone pain & frequency of new skeletal events in metastatic breast
cancer & Multiple myeloma.

QUESTION 273. MTQ3MStTQVNJUkVLSEEgUCtzYXNpcmVraGEzMTRAZ21haWwuY29tKzc3MDgwMDE4NzdR


ElPTiAyNzI=
Type of placenta insertion shown is:

Score:
DBMCI CLT MAINS 2018 755.00 /
Page 174 1200
CLT MAINS - 2018
Exam Title :
(Final)
Email : sasirekha314@gmail.com
Contact : 7708001877

a) Battledore Placenta
b) Velamentous Insertion
c) Normal
d) Circumvallate
Correct Answer: A
Your Answer: A
Explanation

Ans. A. Battledore Placenta

· The battledore placenta can be defined as an abnormal insertion of the umbilical cord. The
condition is also known as the marginal cord insertion.

· Battledore placenta can be found in association with other conditions, such as: monochorionic
twin pregnancy, intrauterine growth retardation, pre-term birth and low-weight at birth.

Score:
DBMCI CLT MAINS 2018 755.00 /
Page 175 1200
CLT MAINS - 2018
Exam Title :
(Final)
Email : sasirekha314@gmail.com
Contact : 7708001877

QUESTION 274. MTQ3MStTQVNJUkVLSEEgUCtzYXNpcmVraGEzMTRAZ21haWwuY29tKzc3MDgwMDE4NzdR


PTiAyNzM=
Which of the following is correct for the calculation of Pearl index?

a) (No. of accidental pregnancies x 1200) / (No. of patients observed x months of use)


b) (No. of accidental pregnancies x 1200) / (No. of patients observed x 2400)
c) (No. of patients observed x months of use) / (No. of accidental pregnancies)
d) (No. of patients observed) / (No. of accidental pregnancies x 2400)
Correct Answer: A
Your Answer: A
Explanation

Ans. A. (No. of accidental pregnancies x 1200) / (No. of patients observed x months of


use)

Pearl Index: Failure rate of any contraceptive is calculate din terms of


pregnancy rate per hundred women years of use. It is calculated by
following formula:

Score:
DBMCI CLT MAINS 2018 755.00 /
Page 176 1200
CLT MAINS - 2018
Exam Title :
(Final)
Email : sasirekha314@gmail.com
Contact : 7708001877

Number of accidental pregnancy x 1200

Pearl index = --------------------------------------------------

Number of patients observed x months of use

Here 1200 is the number of months in 100 years

QUESTION 275. MTQ3MStTQVNJUkVLSEEgUCtzYXNpcmVraGEzMTRAZ21haWwuY29tKzc3MDgwMDE4NzdR


lPTiAyNzQ=
Keloid formation is not seen over?

a) Ear
b) Face
c) Eyelid
d) Neck
Correct Answer: C
Your Answer: C
Explanation

Ans. C. Eyelid

· Keloids are defined as scars that grow beyond the borders of the original wounds, and these
scars rarely regress with time. Keloids are more prevalent in patients with more darkly
pigmented skin; they develop in 15% to 20% of African Americans, Asians, and Hispanics.

· Keloids appear to have a genetic predisposition. Keloid scars tend to occur above the clavicles,
on the trunk, on the upper extremities, and on the face. They cannot be prevented at this time
and are often refractory to medical and surgical intervention.

· Hypertrophic scars, in contrast, are raised scars that remain within the confines of the original
wound and frequently regress spontaneously.

· A hypertrophic scar can occur anywhere on the body.

· Keloids most frequently develop on the upper back and chest, neck shoulders, jaw, ear lobes
and legs. These areas all share the quality of high tension on healing wounds.

· Keloids rarely appear on the eyelids, genitals, palms and soles.

QUESTION 276. MTQ3MStTQVNJUkVLSEEgUCtzYXNpcmVraGEzMTRAZ21haWwuY29tKzc3MDgwMDE4NzdR


lPTiAyNzU=
Treatment of presbyopia:

a) LASIK
b) Concave lens
c) Convex lens

Score:
DBMCI CLT MAINS 2018 755.00 /
Page 177 1200
CLT MAINS - 2018
Exam Title :
(Final)
Email : sasirekha314@gmail.com
Contact : 7708001877

d) Radial keratotomy
Correct Answer: C
Your Answer: B
Explanation

Ans. C. Convex lens

Presbyopia is not an error of, refraction but condition physiological insufficiency of


accommodation leading to failing vision of near. It is treated by prescribing convex lens .

QUESTION 277. MTQ3MStTQVNJUkVLSEEgUCtzYXNpcmVraGEzMTRAZ21haWwuY29tKzc3MDgwMDE4NzdR


lPTiAyNzY=
Choreic movements are :-

a) Slow and writhing


b) Shock like asymmetrical
c) Brief, jerky and irregular
d) Rhythmic and oscillatory
Correct Answer: C
Your Answer: C
Explanation

Ans. C. Brief, jerky and irregular

· Chorea is an abnormal involuntary movement derived from the Greek word “dance”. It is
characterized by brief, abrupt, irregular, unpredictable, non-stereotyped movements.

· In milder cases, they may appear purposeful; the patient often appears fidgety and clumsy.

· There is a wide range of seemingly unrelated causes, from pregnancy (chorea gravidarum) to
inherited forms such as Huntington’s disease and benign hereditary chorea, infection/immune-
related such as Sydenham’s chorea and systemic lupus erythematosus, focal vascular lesions in
the basal ganglia, drugs such as levodopa, neuroleptics and oral contraception, various
metabolic and endocrinological disorders such as hyperthyroidism, hypo/hyperparathyroidism
and hypo/hyperglycemia.

QUESTION 278. MTQ3MStTQVNJUkVLSEEgUCtzYXNpcmVraGEzMTRAZ21haWwuY29tKzc3MDgwMDE4NzdR


ElPTiAyNzc=
The deposits shown in the Image are localised to which layer of cornea ?

Score:
DBMCI CLT MAINS 2018 755.00 /
Page 178 1200
CLT MAINS - 2018
Exam Title :
(Final)
Email : sasirekha314@gmail.com
Contact : 7708001877

a) Epithelium
b) Bowman’s layer
c) Posterior stroma
d) Descemet’s membrane
Correct Answer: D
Your Answer: D
Explanation

Ans. D. Descemet’s membrane

Image shows a Kayser- Fletcher ring in the cornea, which represents deposition of copper in the
posterior lamella of Descemet’s membrane.

QUESTION 279. MTQ3MStTQVNJUkVLSEEgUCtzYXNpcmVraGEzMTRAZ21haWwuY29tKzc3MDgwMDE4NzdR


lPTiAyNzg=
Salt and pepper fundus occurs in:

a) Toxoplasma
b) Toxocara
c) Rubella
d) Scurvy
Correct Answer: C
Your Answer: C
Explanation

Ans. C. Rubella

Congenital Rubella (German measles)

1. Retinopathy - visually insignificant, subtle 'salt and pepper' pigmentary disturbance. Most
marked at the macula, later develop

choroidal neovascularization.

Score:
DBMCI CLT MAINS 2018 755.00 /
Page 179 1200
CLT MAINS - 2018
Exam Title :
(Final)
Email : sasirekha314@gmail.com
Contact : 7708001877

2. Pearly nuclear cataract

3. Microphthalmos

4. Glaucoma - resulting in buphthalmos

5. Miscellaneous - stromal keratopathy, iritis, iris atrophy, and extreme refractive errors,
pendular nystagmus and strabismus

Toxoplasma retinitis

i) "Satellite lesion"

ii) "Headlight in the fog" appearance

CMV retinitis - "Brushfire-like" extension along the retinal blood vessels.

Toxocariasis - "Snow-banking" in pars planitis

QUESTION 280. MTQ3MStTQVNJUkVLSEEgUCtzYXNpcmVraGEzMTRAZ21haWwuY29tKzc3MDgwMDE4NzdR


lPTiAyNzk=
Non Absorbable suture among the following is?

a) Polydioxanone
b) Polyester
c) Polyglactin
d) Monocryl
Correct Answer: B
Your Answer: B
Explanation

Ans. B. Polyester

Non absorbable
Absorbable sutures Raw material Raw material
suture

Surgical Gut Derived from healthy Organic protein


Silk suture
suture beef and sheep called fibroin

Monocryl suture Copolymer of Long-chain aliphatic


(Poliglecaprone glycolide and Ethilon Nylon suture polymers nylon 6 or
25) epsiloncaprolactone nylon 6,6

Copolymer of lactide
Coated Vicryl
and glycolide coated Mersilene polyester Plyethylene
suture (polyglactin
with 370 and calcium fiber suture terephthalate
910)
stearate

Score:
DBMCI CLT MAINS 2018 755.00 /
Page 180 1200
CLT MAINS - 2018
Exam Title :
(Final)
Email : sasirekha314@gmail.com
Contact : 7708001877

Isotactic crystalline
PDS II Suture Prolene
Polyester polymer stereoisomer of
(Polydioxanone) polypropylene suture
polypropylene

QUESTION 281. MTQ3MStTQVNJUkVLSEEgUCtzYXNpcmVraGEzMTRAZ21haWwuY29tKzc3MDgwMDE4NzdR


lPTiAyODA=
On inspection of the oral cavity of a patient the faucial pillars, and soft palate can be
visualized, but the uvula is masked by the base of the tongue. The patient would be
classified as?

a) Mallampati I
b) Mallampati II
c) Mallampati III
d) Mallampati IV
Correct Answer: B
Your Answer: B
Explanation

Ans. B. Mallampati II.

· Mallampati I - faucial pillars, soft palate, and uvula can be visualized

· Mallampati II - faucial pillars, soft palate can be visualized, but the uvula is masked by the
base of the tongue

· Mallampati III - only the base of the uvula can be visualized

· Mallampati IV - none of the three structures can be visualized.

QUESTION 282. MTQ3MStTQVNJUkVLSEEgUCtzYXNpcmVraGEzMTRAZ21haWwuY29tKzc3MDgwMDE4NzdR


lPTiAyODE=
Identify the given below airway device:

Score:
DBMCI CLT MAINS 2018 755.00 /
Page 181 1200
CLT MAINS - 2018
Exam Title :
(Final)
Email : sasirekha314@gmail.com
Contact : 7708001877

a) I-Gel
b) BASKA
c) LMA Supreme
d) Classic LMA
Correct Answer: B
Your Answer: A
Explanation

Ans. B. BASKA

· It's BASKA airway device.

· 3rd generation supraglottic airway device.

QUESTION 283. MTQ3MStTQVNJUkVLSEEgUCtzYXNpcmVraGEzMTRAZ21haWwuY29tKzc3MDgwMDE4NzdR


lPTiAyODI=
AFP raised in :

a) Teratoma
b) Yolk sac tumor
c) Choriocarcinoma
d) Dysgerminoma
Correct Answer: B
Your Answer: B
Explanation

Ans. B. Yolk sac tumor

Score:
DBMCI CLT MAINS 2018 755.00 /
Page 182 1200
CLT MAINS - 2018
Exam Title :
(Final)
Email : sasirekha314@gmail.com
Contact : 7708001877

Alpha Feto protein (AFP) level are elevated in non- seminomatous germ cell tumors
(NSGCT) including embryora. a carcinoma, yolk sac tumor and mixed germ cell
tumors.

Germ Cell Tumors

Non seminomatous Germ Cell K Tumors


Seminomatous germ Cell Tumors
(NSGCT)

AFP
Tumor AFP hCG Tumor
hCG

Embryonal carcinoma

Yolk sac tumor +ve +ve


Seminoma (male)
-ve +/- +ve -ve
(Endodermal sinus
Dysgerminoma tumor)
-ve +/- -ve +ve
(female)
Choriocarcinoma
-ve -ve
Teratoma

QUESTION 284. MTQ3MStTQVNJUkVLSEEgUCtzYXNpcmVraGEzMTRAZ21haWwuY29tKzc3MDgwMDE4NzdR


PTiAyODM=
Medications least used in treatment of following skin pathology include the following?

Score:
DBMCI CLT MAINS 2018 755.00 /
Page 183 1200
CLT MAINS - 2018
Exam Title :
(Final)
Email : sasirekha314@gmail.com
Contact : 7708001877

a) Emollients
b) Topical glucocorticoids
c) Dosulepin
d) Doxepin
Correct Answer: C
Your Answer: Unanswered
Explanation

Ans. C. Dosulepin

· Hyperpigmentation, lichenification, and scaling in the antecubital fossae are seen in this
patient suggesting the diagnosis of atopic dermatitis. Therapy for AD should include avoidance
of cutaneous irritants, adequate moisturizing through the application of emollients, judicious
use of topical anti-inflammatory agents, and prompt treatment of secondary infection.

· Low to mid-potency topical glucocorticoids are employed in most treatment regimens for AD.

· Control of pruritus is essential for treatment because AD often represents “an itch that
rashes”. Antihistamines are most often used to control pruritus. Diphenhydramine (25 mg every
4-6 h), hydroxyzine (10-25mg every 6 h), or doxepin (10-25 mg at bedtime) are useful primarily
due to their sedating action.

· Treatment with systemic glucocorticoids should be limited to severe exacerbations


unresponsive to topical therapy. Dosulepin is tricyclic antidepressant used in long term
treatment of depression.

QUESTION 285. MTQ3MStTQVNJUkVLSEEgUCtzYXNpcmVraGEzMTRAZ21haWwuY29tKzc3MDgwMDE4NzdR


PTiAyODQ=
The commonest cause of metabolic alkalosis is:

a) Cancer stomach
b) Pyloric stenosis
c) Small-bowel obstruction
d) Diuretics
Correct Answer: B
Your Answer: Unanswered
Explanation

Ans. B. Pyloric stenosis

Metabolic Alkalosis is the most common acid base disturbance in the surgical patients usually
result from excessive vomiting (as in pyloric stenosis) and excessive administration of sodium
bicarbonate during resuscitation.

QUESTION 286. MTQ3MStTQVNJUkVLSEEgUCtzYXNpcmVraGEzMTRAZ21haWwuY29tKzc3MDgwMDE4NzdR


PTiAyODU=
'Chancre redux' is a clinical feature of:

Score:
DBMCI CLT MAINS 2018 755.00 /
Page 184 1200
CLT MAINS - 2018
Exam Title :
(Final)
Email : sasirekha314@gmail.com
Contact : 7708001877

a) Early relapsing syphilis


b) Late syphilis
c) Chancroid
d) Recurrent herpes simplex infection
Correct Answer: A
Your Answer: B
Explanation

Ans. A. Early relapsing syphilis

· Excoriated secondary syphilitic papules in women can be confused with multiple small
chancres. Chancre Redux is a recurrence of the primary sore at its original site.

· Pseudochancre Redux describes gummatous (tertiary stage) recurrence at the site of the
primary chancre.

QUESTION 287. MTQ3MStTQVNJUkVLSEEgUCtzYXNpcmVraGEzMTRAZ21haWwuY29tKzc3MDgwMDE4NzdR


lPTiAyODY=
Recognize the contraceptive shown in the Image:

a) Progestasert
b) Mirena
c) Lippes loop
d) Vaginal ring
Correct Answer: C
Your Answer: C

Score:
DBMCI CLT MAINS 2018 755.00 /
Page 185 1200
CLT MAINS - 2018
Exam Title :
(Final)
Email : sasirekha314@gmail.com
Contact : 7708001877
Explanation

Ans. C. Lippes loop

• 1st generation / non - dedicated / inert IUD contraceptives


• Consists of a thin plastic wire bent in a series of S shapes
• Needs to be straightened when it is being inserted into the uterus but resumes its shape
once inside
• Intended for long term use until menopause
• No Longer used in India
• IUD with highest failure rate

QUESTION 288. MTQ3MStTQVNJUkVLSEEgUCtzYXNpcmVraGEzMTRAZ21haWwuY29tKzc3MDgwMDE4NzdR


lPTiAyODc=
The efficacy of which Anticonvulsant decreases when used with combined oral
contraceptives:

a) Phenytoin
b) Carbamazepine
c) Topiramate
d) Lamotrigine
Correct Answer: D
Your Answer: B
Explanation

Ans. D. Lamotrigine

Anticonvulsants (Phenytoin, carbamazepine, barbiturates, primidone, topiramate,


oxcarbazepine) : Use of these anticonvulsants may decrease the effectiveness of COCs.
Although the interaction of these anticonvulsants with COCs, minipill(P) or Combined vaginal
ring (CVR) is not harmful to women, it is likely to reduce the effectiveness of COCs, P or CVR.

Use of other contraceptives should be encouraged for women who are long-term users of any of
these drugs. When a COC is chosen, a preparation containing a minimum of 30 µg of ethinyl
estradiol (EE) should be used.

Lamotrigine : Pharmacokinetic studies show levels of lamotrigine decrease significantly


during COC use and increase significantly during the pill-free interval. Some women who used
both COCs and lamotrigine have been shown to experience increased seizure activity.

QUESTION 289. MTQ3MStTQVNJUkVLSEEgUCtzYXNpcmVraGEzMTRAZ21haWwuY29tKzc3MDgwMDE4NzdR


lPTiAyODg=
Commonest cause of pyogenic liver abscess:

a) Biliary Sepsis
b) Biliary Colic
c) Appendicitis

Score:
DBMCI CLT MAINS 2018 755.00 /
Page 186 1200
CLT MAINS - 2018
Exam Title :
(Final)
Email : sasirekha314@gmail.com
Contact : 7708001877

d) Sigmoid Diverticulitis
Correct Answer: A
Your Answer: A
Explanation

Ans. A. Biliary Sepsis

“ The majority of hepatic abscesses were used to result from portal spread of Intra-abdominal
infections (e.g. appendicitis, diverticulitis, colitis) but with approved management of these
conditions spread now occur primarily through the biliary tree or arterial supply as in patients
suffering from cholangitis or after biliary tract manipulation.”

QUESTION 290. MTQ3MStTQVNJUkVLSEEgUCtzYXNpcmVraGEzMTRAZ21haWwuY29tKzc3MDgwMDE4NzdR


lPTiAyODk=
Menorrhagia ('heavy periods') is defined as a blood loss of greater than:

a) 35 mL per period
b) 55 mL per period
c) 80 mL per period
d) 120 mL per period
Correct Answer: C
Your Answer: C
Explanation

Ans. C. 80 mL per period

· Menorrhagia ('heavy periods') is defined as a blood loss of greater than 80 mL per period.

· This definition is rather arbitrary, but represents the level of blood loss at which a fall in
haemoglobin and hematocrit concentration commonly occurs.

QUESTION 291. MTQ3MStTQVNJUkVLSEEgUCtzYXNpcmVraGEzMTRAZ21haWwuY29tKzc3MDgwMDE4NzdR


lPTiAyOTA=
Condition that can give rise to the following appearance include:

Score:
DBMCI CLT MAINS 2018 755.00 /
Page 187 1200
CLT MAINS - 2018
Exam Title :
(Final)
Email : sasirekha314@gmail.com
Contact : 7708001877

a) Superior limbic keratoconjunctivitis


b) Sjogren's syndrome
c) Thygeson keratitis
d) Adenoviral keratitis
Correct Answer: B
Your Answer: Unanswered
Explanation

Ans. B. Sjogren's syndrome

· The Rose Bengal stains the inferior cornea showing inferior superficial punctate keratitis. This
is seen in dry eyes and exposure keratitis.

· Superior limbic keratoconjunctivitis typically shows staining in the superior cornea. Whereas
Thygeson and adenoviral keratitis tends to have staining which involve the whole cornea and
are less dense.

QUESTION 292. MTQ3MStTQVNJUkVLSEEgUCtzYXNpcmVraGEzMTRAZ21haWwuY29tKzc3MDgwMDE4NzdR


lPTiAyOTE=
Which of the following agents is useful in the treatment of malignant hyperthermia?

a) Baclofen
b) Diazepam
c) Cyclobenzaprine
d) Dantrolene
Correct Answer: D
Your Answer: D
Explanation

Ans. D. Dantrolene

Score:
DBMCI CLT MAINS 2018 755.00 /
Page 188 1200
CLT MAINS - 2018
Exam Title :
(Final)
Email : sasirekha314@gmail.com
Contact : 7708001877

· Malignant hyperthermia (hyperpyrexia), a syndrome that is associated with the use of a


general anesthetic (e.g., halothane) in conjunction with a skeletal muscle relaxant, is
characterized by tachycardia, hyperventilation, arrhythmias, fever, muscular fasciculation, and
rigidity.

· It is caused by a sudden increase in the availability of calcium (Ca) ions in the mycoplasma of
muscle.

· Dantrolene, which interferes with release of Ca ions from the sarcoplasmic reticulum, is
indicated in treatment of the disorder.

· The first three agents are centrally acting skeletal muscle relaxants that are not useful in the
treatment of malignant hyperthermia.

QUESTION 293. MTQ3MStTQVNJUkVLSEEgUCtzYXNpcmVraGEzMTRAZ21haWwuY29tKzc3MDgwMDE4NzdR


ElPTiAyOTI=
Which of the following inhalation anaesthetics is most likely to produce
hepatotoxicity?

a) Isoflurane
b) Enflurane
c) Methoxyflurane
d) Halothane
Correct Answer: D
Your Answer: D
Explanation

Ans. D. Halothane

· Halothane is a substituted alkane general anesthetic.

· It undergoes significant metabolism in humans with about 20% of the absorbed dose
recovered as metabolites.

· Halothane can cause postoperative jaundice and hepatic necrosis with repeated administration
in rare instances.

QUESTION 294. MTQ3MStTQVNJUkVLSEEgUCtzYXNpcmVraGEzMTRAZ21haWwuY29tKzc3MDgwMDE4NzdR


PTiAyOTM=
The burrows in scabies is in the-

Score:
DBMCI CLT MAINS 2018 755.00 /
Page 189 1200
CLT MAINS - 2018
Exam Title :
(Final)
Email : sasirekha314@gmail.com
Contact : 7708001877

a) Stratum basale
b) Stratum granulosum
c) Stratum corneum
d) The dermis
Correct Answer: C
Your Answer: C
Explanation

Ans. C. Stratum corneum

Scabies, also known as the seven-year itch, is a contagious skin infestation by the mite
Sarcoptes scabiei. Scabies mite burrows into stratum corneum to live and deposit eggs. These
burrows are visible clinically as irregular grey- brown lines.

QUESTION 295. MTQ3MStTQVNJUkVLSEEgUCtzYXNpcmVraGEzMTRAZ21haWwuY29tKzc3MDgwMDE4NzdR


lPTiAyOTQ=
Which of the following is not a feature of primary hyperparathyroidism?

a) Hypercalcemia
b) Hyperphosphatemia
c) Hyperchloremia
d) Increased Alkaline phosphatase.
Correct Answer: B
Your Answer: B
Explanation

Ans. B. Hyperphosphatemia.

Score:
DBMCI CLT MAINS 2018 755.00 /
Page 190 1200
CLT MAINS - 2018
Exam Title :
(Final)
Email : sasirekha314@gmail.com
Contact : 7708001877

· It causes hypophosphatemia.

· Hyperphosphatemia is seen in secondary hyperthyroidism.

QUESTION 296. MTQ3MStTQVNJUkVLSEEgUCtzYXNpcmVraGEzMTRAZ21haWwuY29tKzc3MDgwMDE4NzdR


lPTiAyOTU=
Indication of liver transplantation are all of following except :

a) Biliary atresia
b) Sclerosing cholangitis
c) Hepatitis A
d) Cirrhosis
Correct Answer: A
Your Answer: C
Explanation

Ans. A. Biliary atresia

Indications of Liver Transplantation

Children Adult

Score:
DBMCI CLT MAINS 2018 755.00 /
Page 191 1200
CLT MAINS - 2018
Exam Title :
(Final)
Email : sasirekha314@gmail.com
Contact : 7708001877

Biliary atresia

Neonatal hepatitis Primary biliary cirrhosis

Congenital hepatic fibrosis Secondary biliary cirrhosis

Alagille’s disease Primary sclerosing cholangitis

Byler’s disease Caroli’s disease

α 1 Antitrypsin deficiency Cryptogenic cirrhosis

Chronic hepatitis with cirrhosis


Inherited disorders of
metabolism Hepatic vein thrombosis
Wilson’s disease, Tyrosinemia Alcoholic cirrhosis
Glycogen storage disease Chronic viral hepatitis
Lysosomal storage disase Primary hepatocellular malignancies
Protoporphyria, Hepatic adenomas
Crigler-Najjar disease Type- I, Non alcoholic steatohepatitis
Familial hypercholesterolemia, Familial amyloid polyneuropathy
Hereditary oxalosis, Hemophilia

QUESTION 297. MTQ3MStTQVNJUkVLSEEgUCtzYXNpcmVraGEzMTRAZ21haWwuY29tKzc3MDgwMDE4NzdR


lPTiAyOTY=
A 5 yr old male child has multiple hyperpigmented macules over the trunk. On rubbing
the lesion with rounded end of pen, he developed urticarial wheal, confined to border
of lesion, most likely diagnosis is:

a) Fixed drug eruption


b) Lichen planus
c) Urticaria pigmentosa
d) Urticarial vasculitis
Correct Answer: C
Your Answer: C
Explanation

Ans. C. Urticaria pigmentosa

· It is urticaria pigmentosa.

· It is cutaneous mastocytosis presenting with pigmented lesions.

· On rubbing the lesions mast cells degranulate releasing histamine.

· This produces the wheal and flare of the lesion. It is known as Darier’s sign.

Score:
DBMCI CLT MAINS 2018 755.00 /
Page 192 1200
CLT MAINS - 2018
Exam Title :
(Final)
Email : sasirekha314@gmail.com
Contact : 7708001877

Epidermal acantholysis in mid epidermis with “dilapidated or crumbling brick wall”


appearance. Darier’s disease is associated with ATP2A2 defect

QUESTION 298. MTQ3MStTQVNJUkVLSEEgUCtzYXNpcmVraGEzMTRAZ21haWwuY29tKzc3MDgwMDE4NzdR


ElPTiAyOTc=
Identify the prosthesis shown in the Image below:

a) TORP
b) PORP
c) Stapes prosthesis
d) All of the above
Correct Answer: B
Your Answer: C
Explanation

Ans. B. PORP

Porp is kept between tympanic membrane and stapes footplate

QUESTION 299. MTQ3MStTQVNJUkVLSEEgUCtzYXNpcmVraGEzMTRAZ21haWwuY29tKzc3MDgwMDE4NzdR


lPTiAyOTg=
All are true regarding physiology of menstruation except?

Score:
DBMCI CLT MAINS 2018 755.00 /
Page 193 1200
CLT MAINS - 2018
Exam Title :
(Final)
Email : sasirekha314@gmail.com
Contact : 7708001877

a) Early folliculo genesis is independent of gonadotropins


b) LH is required for folliculo genesis
c) Inhibin A levels are increased in the follicular phase
d) Progesterone in low levels causes positive feedback on pituitary to release LH
Correct Answer: C
Your Answer: C
Explanation

Ans. C. Inhibin A levels are increased in the follicular phase

a. Early folliculo genesis is gonadotropin independent. FSH rescues the antral follicles and
selects a cohort of follicles for further development.

b. Early folliculo genesis is dependent on FSH. LH acts on the theca interna cells to secrete
androgens which act as substrate for the synthesis of increased amount of estrogens by the
granulose cells of the developing dominant follicle. Thus LH also contributed to folliculo
genesis. For a healthy dominant follicle, it is important to change the androgenic environment
of the follicle to estrogenic.

c. Granulosa cells in the follicular phase also secrete inhibin B, which along with estradiol
causes negative feedback on the pituitary to decrease FSH levels.

d. Inhibin A is the form of inhibin synthesized by the luteinized granulosa cells of the ovary in
the luteal phase, under the control of LH

e. FSH in the early follicular phase, induces LH receptors on the granulose cells. Thus, in the
late follicular phase, the luteinized granulose cells synthesize low amounts of progesterone, that
cause positive feedback on the pituitary to increase LH secretion, thus, long with estradiol
contributing to LH surge.

QUESTION 300. MTQ3MStTQVNJUkVLSEEgUCtzYXNpcmVraGEzMTRAZ21haWwuY29tKzc3MDgwMDE4NzdR


lPTiAyOTk=
Which of the following sign is indicated in the Image given below?

Score:
DBMCI CLT MAINS 2018 755.00 /
Page 194 1200
CLT MAINS - 2018
Exam Title :
(Final)
Email : sasirekha314@gmail.com
Contact : 7708001877

a) Chvostek sign
b) Troisier sign
c) Trousseau’s sign
d) Lhermitte’s sign
Correct Answer: C
Your Answer: C
Explanation

Ans. C. Trousseau’s sign

The image shows presence of trousseau sign/Carpo-pedal spasm seen in patients of


Hypocalcemia.

Elicited by tapping over anterior border of parotid gland and seen in


Chvostek sign
tetany

Left supraclavicular lymphadenopathy in metastatic abdominal


Troisier sign
malignancy

The barber chair phenomenon is the name which describes an


electric shock-like sensation that occurs on flexion of the neck. This
Lhermitte’s sign
sensation radiates down the spine, often into the legs, arms and
sometimes to the trunk

Score:
DBMCI CLT MAINS 2018 755.00 /
Page 195 1200
CLT MAINS - 2018
Exam Title :
(Final)
Email : sasirekha314@gmail.com
Contact : 7708001877
Review in

Score:
DBMCI CLT MAINS 2018 755.00 /
Page 196 1200

You might also like